PRACTICE TEST 1 - McGraw-Hill Education SAT 2017 Edition (Mcgraw Hill's Sat) (2016)

McGraw-Hill Education SAT 2017 Edition (Mcgraw Hill's Sat) (2016)

CHAPTER 11

PRACTICE TEST 1

  1. Reading Test

65 MINUTES 52 QUESTIONS

  1. Writing and Language Test

35 MINUTES 44 QUESTIONS

  1. Math Test – No Calculator

25 MINUTES 20 QUESTIONS

  1. Math Test – Calculator

55 MINUTES 38 QUESTIONS

  1. Essay (optional)

50 MINUTES 1 QUESTION

ANSWER SHEET

Start with number 1 for each new section. If a section has fewer questions than answer spaces, leave the extra answer spaces blank. Be sure to erase any errors or stray marks completely.

Start with number 1 for each new section. If a section has fewer questions than answer spaces, leave the extra answer spaces blank. Be sure to erase any errors or stray marks completely.

CAUTION Use the answer spaces in the grids below for Section 3 only if you are told to do so in your test book.

Student-Produced Responses ONLY ANSWERS ENTERED IN THE CIRCLES IN EACH GRID WILL BE SCORED. YOU WILL NOT RECEIVE CREDIT FOR ANYTHING WRITTEN IN THE BOXES ABOVE THE CIRCLES.

Start with number 1 for each new section. If a section has fewer questions than answer spaces, leave the extra answer spaces blank. Be sure to erase any errors or stray marks completely.

CAUTION Use the answer spaces in the grids below for Section 4 only if you are told to do so in your test book.

Student-Produced Responses ONLY ANSWERS ENTERED IN THE CIRCLES IN EACH GRID WILL BE SCORED. YOU WILL NOT RECEIVE CREDIT FOR ANYTHING WRITTEN IN THE BOXES ABOVE THE CIRCLES.

SECTION 5: ESSAY

You may wish to remove these sample answer document pages to respond to the practice SAT Essay Test.

SECTION 5: ESSAY

You may wish to remove these sample answer document pages to respond to the practice SAT Essay Test.

SECTION 5: ESSAY

You may wish to remove these sample answer document pages to respond to the practice SAT Essay Test.

SECTION 5: ESSAY

You may wish to remove these sample answer document pages to respond to the practice SAT Essay Test.

Reading Test

65 MINUTES, 52 QUESTIONS

Turn to Section 1 of your answer sheet to answer the questions in this section.

DIRECTIONS

Each passage or pair of passages below is followed by a number of questions. After reading each passage or pair, choose the best answer to each question based on what is stated or implied in the passage or passages and in any accompanying graphics.

Questions 1–12 are based on the following passage.

Passage 1 is adapted from Nicholas Heidorn, “The Enduring Political Illusion of Farm Subsidies .” ©2004 The Independent Institute. Originally Published August 18, 2004 in the San Francisco Chronicle. Passage 2 is ©2015 by Mark Anestis. Since 1922, the U.S. government has subsidized the agricultural industry by supporting the price of crops (commodity subsidies), paying farmers let their fields go fallow (conservation subsidies), helping farmers purchase crop insurance (crop insurance subsidies), and compensating farmers for uninsured losses due to disasters (disaster subsidies). The following passages discuss these programs.

Passage 1

Passage 2

1

Both passages acknowledge the effectiveness of U.S. farm subsidies in

  1. A) stabilizing commodity prices.
  2. B) expanding American exports.
  3. C) assisting smaller farms.
  4. D) increasing agricultural productivity.

2

The first sentence of Passage 1 refers primarily to the author”s belief that

  1. A) the American government is not doing enough to help small farmers.
  2. B) some American farmers are violating the law.
  3. C) a federal agricultural program is unfair and ineffective.
  4. D) American farmers are struggling to compete in international markets.

3

The author of Passage 2 would most likely regard the “taxes” mentioned in line 15 as

  1. A) a worthwhile expenditure.
  2. B) a misplaced priority.
  3. C) a political delusion.
  4. D) a technical misnomer.

4

The author of Passage 1 believes that the GAO report “probably won”t” (line 9) horrify lawmakers because

  1. A) the report indicates that farm subsidies are not as harmful as many suggest.
  2. B) most members of congress do not live in districts that receive farm subsidies.
  3. C) the legislature is too divided along ideological party lines.
  4. D) many members of congress receive benefits from pro-subsidy farm lobbies.

5

Which of the following provides the strongest evidence for the answer to the previous question?

  1. A) Lines 16–18 (“Because of … wheat prices”)
  2. B) Lines 21–24 (“To prevent this … prices again”)
  3. C) Lines 40–43 (“Indeed … family farms”)
  4. D) Lines 53–55 (“Agribusiness … sign of closing”)

6

Unlike Passage 1, Passage 2 emphasizes the danger of

  1. A) corrupt political officials.
  2. B) sudden changes in commodity prices.
  3. C) competition in international markets.
  4. D) onerous public tax burdens.

7

Passage 1 mentions the results of the 2009 poll (lines 40–43) primarily to

  1. A) confirm a general sentiment.
  2. B) refute a misconception.
  3. C) change the focus of the discussion.
  4. D) reveal a surprising finding.

8

If the author of Passage 1 were to use the data in the graph to support his main thesis, he would most likely mention

  1. A) the general decline in total farm subsidies from 2005 to 2012.
  2. B) the overall rate of change in commodity subsidies from 1998 to 2012.
  3. C) the expansion of crop insurance subsidies from the late 1990s to the late 2000s.
  4. D) the sudden spike in disaster subsidies from 2004 to 2005.

9

If the author of Passage 2 were to use the data in the graph to support his main thesis, he would most likely mention

  1. A) the general decline in total farm subsidies from 2005 to 2012.
  2. B) the overall rate of change in commodity subsidies from 1998 to 2012.
  3. C) the expansion of crop insurance subsidies from the late 1990s to the late 2000s.
  4. D) the sudden spike in disaster subsidies from 2004 to 2005.

10

The author of Passage 1 would most likely say that the “benefit” in line 85 is

  1. A) offset by its costs.
  2. B) an exception to a rule.
  3. C) enjoyed only by the wealthy.
  4. D) misrepresented by legislators.

11

Unlike Passage 2, Passage 1 makes a direct appeal to the reader”s

  1. A) sense of humor.
  2. B) distaste for ineptitude.
  3. C) environmental responsibility.
  4. D) fiscal prudence.

12

In line 55, the “floodgates” are controls against

  1. A) environmental destruction.
  2. B) unscrupulous funding.
  3. C) emotional outbursts.
  4. D) necessary capital.

Questions 13–22 are based on the following passage.

This passage is adapted from Marie Myung-Ok Lee, Somebody”s Daughter . ©2006 Beacon Press. The story is about a Korean-American girl adopted by an American family and raised in the Midwest.

13

The narrator characterizes Reverend Jansen primarily as

  1. A) an aloof scholar.
  2. B) a fierce taskmaster.
  3. C) a sympathetic caregiver.
  4. D) a patronizing figure.

14

The narrator”s statement that her mother “had been murdered” (line 2) is best taken to mean that

  1. A) her mother was killed by a negligent driver.
  2. B) the reputation of her mother had been severely impugned.
  3. C) the death of her mother was deliberate.
  4. D) her adoptive family was trying to obliterate all memory of her biological mother.

15

The narrator”s description of the reverend”s “eyes” and “breath” in lines 15–16 primarily convey a sense of

  1. A) empathy.
  2. B) detachment.
  3. C) geniality.
  4. D) severity.

16

Christine believes that Sarah”s ethnicity is

  1. A) a source of pride.
  2. B) an exotic mystery.
  3. C) a sacred blessing.
  4. D) an unfortunate fact.

17

Which choice provides the best evidence for the answer to the previous question?

  1. A) Lines 13–16 (“God called … second”)
  2. B) Lines 24–26 (“That”s why … treasure”)
  3. C) Lines 48–52 (“It was almost … is not real”)
  4. D) Lines 76–80 (“Since the Korean … government coffers”)

18

Lines 26–28 (“God kills … my mother”) are striking for their use of

  1. A) juxtaposition.
  2. B) metaphor.
  3. C) personification.
  4. D) understatement.

19

Lines 36–49 chiefly describe Christine”s

  1. A) cunning deceitfulness.
  2. B) sense of superiority.
  3. C) motherly sympathy.
  4. D) emotional immaturity.

20

In line 45, “charged” most nearly means

  1. A) loaded.
  2. B) entrusted.
  3. C) attacked.
  4. D) demanded.

21

The passage suggests that Uncle Henry”s role in the Thorson family is that of

  1. A) a stern patriarch.
  2. B) a bigoted lout.
  3. C) a pitiable embarrassment.
  4. D) a noble hero.

22

The “cheery smirk” (line 81) is taken by the narrator to indicate Gumbel”s

  1. A) satisfaction with the publicity the Olympics were receiving.
  2. B) admiration for Korea”s economic competitiveness.
  3. C) pleasure that Korean children would be well cared for.
  4. D) happiness that Singapore had finally been defeated.

Questions 23–32 are based on the following passage and any accompanying material.

This passage is adapted from Christopher F. Black, “Baby Pictures of the Universe .” ©2015 by Christopher F. Black and College Hill Coaching.

23

This passage is primarily concerned with

  1. A) chronicling the discoveries yielded by recent satellite telescopes.
  2. B) examining the controversies surrounding a physical theory.
  3. C) discussing the analysis and significance of a cosmological phenomenon.
  4. D) describing similarities between the study of human history and the study of astronomy.

24

The author presents the “Gettysburg Cyclorama” (line 1) primarily as

  1. A) an illustrative analogy.
  2. B) a historical precedent.
  3. C) a quaint anachronism.
  4. D) an accidental success.

25

Lines 11–13 (“Unfortunately … human eyes”) convey the author”s disappointment in

  1. A) the appropriateness of a comparison.
  2. B) an audience”s level of interest.
  3. C) the magnitude of an event.
  4. D) the accessibility of a phenomenon.

26

The quotation marks around the words “invisible” (line 26) and “see” (line 28) serve primarily to

  1. A) draw attention to two relatively recent coinages.
  2. B) imply that the author is speaking speculatively.
  3. C) suggest an irony implicit in conventional terms.
  4. D) indicate a technical usage of common words.

27

The “moment a swaddled one-day-old opens its eyes” (lines 22–23) corresponds to the instant that

  1. A) scientists first discovered the cosmic microwave background radiation.
  2. B) all of the particles and energy in the universe were created in the Big Bang.
  3. C) the cosmic microwave background radiation was first released from the hydrogen plasma.
  4. D) George Gamow first published his theory about the cosmic microwave background radiation.

28

In line 64, “distinctive” most nearly means

  1. A) bizarre.
  2. B) distinguishing.
  3. C) elite.
  4. D) irreconcilable.

29

Which of the following can be inferred about the work that earned Penzias and Wilson the Nobel Prize?

  1. A) It was the product of decades of research.
  2. B) It was the result of an accidental discovery.
  3. C) It depended greatly on the data from the COBE satellite.
  4. D) It provided a more plausible alternative to Gamow”s theory.

30

Which choice provides the best evidence for the answer to the previous question?

  1. A) Lines 60–63 (“His theory … current data”)
  2. B) Lines 72–75 (“Their initial … droppings”)
  3. C) Lines 82–89 (“Since then … Gamow”s guess”)
  4. D) Lines 89–96 (“In addition … early universe”)

31

Figure 1 best confirms which claim made in the passage?

Figure 1. Comparison of COBE radiation data to blackbody curves for 2° K and 3° K

  1. A) “For the first 380,000 years of its life … the universe was “invisible”” (lines 24–26)
  2. B) “the CMB radiation did not originate from just one point in space” (lines 53–54)
  3. C) “the expanding universe would have cooled this radiation to below 5 degrees Kelvin today” (lines 66–67)
  4. D) “CMB radiation … has a nearly perfect blackbody spectrum” (lines 85–87)

32

Figure 2 best confirms which claim made in the passage?

Figure 2. Panoramic map of the cosmic background radiation showing temperatures ranging from 2.7248° K (dark) to 2.7252° K (white)

  1. A) “For the first 380,000 years of its life … the universe was “invisible”” (lines 24–26)
  2. B) “the CMB radiation did not originate from just one point in space” (lines 53–54)
  3. C) “the expanding universe would have cooled this radiation to about 5 degrees Kelvin” (lines 66–67)
  4. D) “CMB radiation … has a nearly perfect blackbody spectrum” (lines 85–87)

Questions 33–42 are based on the following passage.

This passage is from John Adams, “A Dissertation on Canon and Feudal law.” Originally published in 1765.

1 benefit

2 related to church matters

33

The first paragraph is primarily concerned with the right of citizens to

  1. A) pursue academic interests.
  2. B) learn more about their leaders.
  3. C) become proficient in the art of printing.
  4. D) propose helpful legislation.

34

In line 14, “constitute” most nearly means

  1. A) place in power.
  2. B) account for.
  3. C) amount to.
  4. D) be regarded as.

35

The passage indicates that our “forefathers” (line 49) endured all of the following EXCEPT

  1. A) physical deprivation.
  2. B) political oppression.
  3. C) arduous physical labor.
  4. D) a sense of despair.

36

The passage indicates that all people are born with

  1. A) a curious nature.
  2. B) a desire for power.
  3. C) a dread of tyranny.
  4. D) a sense of thrift.

37

Which sentence provides the best evidence for the answer to the previous question?

  1. A) Lines 1–5 (“Liberty … to know”)
  2. B) Lines 10–15 (“And if the cause … trustees”)
  3. C) Lines 20–25 (“The only question … public expenses”)
  4. D) Lines 38–46 (“Let us study … earth and hell”)

38

In line 34, the phrase “every order and degree” refers to

  1. A) an anthology of official declarations.
  2. B) a set of civic responsibilities.
  3. C) the diverse groups within a society.
  4. D) the highest standards of academic achievement.

39

Compared to the first paragraph, the second paragraph is more

  1. A) prescriptive.
  2. B) despondent.
  3. C) critical.
  4. D) ironic.

40

In line 51, “power” refers to

  1. A) a personal ability.
  2. B) a social virtue.
  3. C) a despotic agent.
  4. D) a mysterious spirit.

41

In line 46, “the gates of earth and hell” refer primarily to

  1. A) the privations endured by our forefathers.
  2. B) the superstitions of ancient cultures.
  3. C) the dangers posed by an ignorant populace.
  4. D) the brutality of oppressive leaders.

42

In the second paragraph, the discussion of the “views and ends” (line 48) of our forefathers primarily serves to

  1. A) remind the reader of the importance of liberty.
  2. B) establish a contrast between the past and the present.
  3. C) emphasize the significance of hard work.
  4. D) draw attention to an unfortunate tradition.

Questions 43–52 are based on the following passage and supplementary material.

This passage is from David Biello, “Can Tiny Plankton Help Reverse Climate Change?” ©2015 by David Biello. Originally published in Aeon (http://aeon.co/ ) on July 1, 2014.

NASA satellite image of the largest recorded natural phytoplankton bloom in February 2012, believed to have been caused by the addition of iron dust blown into the sea around Antarctica by strong offshore winds.

43

The characterization of the Southern Ocean in the first paragraph (lines 1–8) primarily serves to emphasize

  1. A) the improbability of Smetacek”s success.
  2. B) the pessimism of Smetacek”s detractors.
  3. C) the boldness of Smetacek”s experiment.
  4. D) the promise of Smetacek”s hypothesis.

44

In line 13, the word “base” most nearly means

  1. A) sordid.
  2. B) precarious.
  3. C) stark.
  4. D) foundational.

45

The passage indicates that the “fertilizer run-off” (line 23) is

  1. A) an unfortunate by-product.
  2. B) an environmental hazard.
  3. C) a potential sustenance.
  4. D) a source of oxygen.

46

The author regards the fertilization of oceans with iron as

  1. A) a well-intentioned but environmentally dangerous activity.
  2. B) a brave but needlessly expensive endeavor.
  3. C) a promising and feasible solution to a global problem.
  4. D) an established and valuable component of the worldwide economy.

47

Which sentence provides the best evidence for the answer to the previous question?

  1. A) Lines 5–6 (“Its churning … ships”)
  2. B) Lines 75–84 (“Activists stoked … crabs and worms”)
  3. C) Lines 90–92 (“Replenishing … the practice”)
  4. D) Lines 94–96 (“Instead, it”s … environment”)

48

Which of the following statements about Smetacek”s research is best supported by Figure 1 ?

  1. A) The iron fertilization from Smetacek”s experiment created a secondary algal bloom nearly as large as the primary bloom.
  2. B) Smetacek”s experiment would likely have been more successful if it were conducted in February, which is the warmest month in the southern hemisphere.
  3. C) Naturally occurring algal blooms in the Southern Ocean can be more than 30 times as large as the one created in Smetacek”s experiments.
  4. D) Algal blooms are likely to get smaller as they move away from the ice shelves that surround Antarctica.

49

The passage suggests that Smetacek regarded the death of the alga bloom described in lines 61–65 as

  1. A) vindication of his theory that iron fertilization can lead to carbon sequestration.
  2. B) an indication of the potential dangers of “dead zones” such as those in the Gulf of Mexico.
  3. C) evidence that there was insufficient oxygen in the Southern Ocean to support large blooms.
  4. D) a disappointment because the diatoms were being removed from the food chain.

50

The passage suggests that iron fertilization could potentially help the whale population primarily by

  1. A) increasing the concentration of oxygen in the ecosphere.
  2. B) decreasing the concentration of carbon dioxide in the atmosphere.
  3. C) supporting an important food source for the whales.
  4. D) reducing the demand for hunting in areas where the whales are endangered.

51

The “route” mentioned in line 91 refers to

  1. A) an experimental procedure.
  2. B) an economic difficulty.
  3. C) an idealistic approach.
  4. D) a mode of persuasion.

52

The tone of the final paragraph is best described as

  1. A) sanguine.
  2. B) awestruck.
  3. C) apprehensive.
  4. D) fatalistic.

STOP

If you finish before time is called, you may check your work on this section only. Do not turn to any other section of the test.

Writing and Language Test

35 MINUTES, 44 QUESTIONS

Turn to Section 2 of your answer sheet to answer the questions in this section.

DIRECTIONS

Each passage below is accompanied by a number of questions. For some questions, you will consider how the passage might be revised to improve the expression of ideas. For other questions, you will consider how the passage might be edited to correct errors in sentence structure, usage, or punctuation. A passage or a question may be accompanied by one or more graphics (such as a table or graph) that you will consider as you make revising and editing decisions.

Some questions will direct you to an underlined portion of a passage. Other questions will direct you to a location in a passage or ask you to think about the passage as a whole.

After reading each passage, choose the answer to each question that most effectively improves the quality of writing in the passage or that makes the passage conform to the conventions of Standard Written English. Many questions include a “NO CHANGE” option. Choose that option if you think the best choice is to leave the relevant portion of the passage as it is.

Questions 1–11 are based on the following passage.

The Carrot or the Stick?

Good teachers want their students to do well, but getting students 1 responding is not always easy. Simple suggestion works occasionally, but not often enough. Reasoning sometimes works, too, but explaining the logical nuances of behavioral standards 2 is often time-consuming and too often falls on deaf ears.

1

  1. A) NO CHANGE
  2. B) to become responsive
  3. C) to respond
  4. D) becoming more responsive

2

  1. A) NO CHANGE
  2. B) are often time-consuming
  3. C) is consuming time
  4. D) consume time

So the practical question becomes: the carrot or the stick? It”s not always easy to choose 3 the potential motivator to consider: by punishment or incentive .

Most educators and psychologists agree that, as a teaching tool, 4 to reward is generally better than punishment, but a growing group of psychologists suggest that rewards can often be as 5 harmful, if not more so, thanpunishment. The introduction of a reward system, like gold stars on an attendance sheet or extra recess time for good behavior, can change the nature not only of the desired behavior, 6 but also of the student-teacher relationship .

Psychologist Edward Deci conducted a study in which people were given a challenging puzzle to solve. Some subjects were offered money as a reward for solving the puzzle, and others were not.

Afterward, both groups were observed secretly after the researcher left the room. Many of those who had not been paid as a reward for their work continued to play with the puzzle, presumably because they found it interesting for its own sake. 7 Those who had received the cash rewards, however, showed significantly less interest in returning to the puzzle .

3

  1. A) NO CHANGE
  2. B) between punishment and incentive when considering potential motivators
  3. C) the potential motivator to consider: either punishment or incentive
  4. D) between punishment and incentive as potential motivators to be considered

4

  1. A) NO CHANGE
  2. B) reward
  3. C) rewarding
  4. D) a reward

5

  1. A) NO CHANGE
  2. B) harmful as, if not more harmful than,
  3. C) harmful, if not more harmful, than
  4. D) equally harmful, if not more harmful than,

6

  1. A) NO CHANGE
  2. B) but also the nature of the student-teacher relationship
  3. C) but the student-teacher relationship as well
  4. D) but the nature of the student-teacher relationship is changed as well

it interesting for its own sake. 7 Those who had received the cash rewards, however, showed significantly less interest in returning to the puzzle.

8 Interpreting these results, the subjects who were paid probably construed the task as being manipulative: the experimenter was trying to get them to do something through bribery. The unpaid subjects, however, could engage the puzzle on their own terms simply because it was fun.

This study and others like it have profound 9 implications for the classroom. Several experiments have demonstrated that “pay-to-read” programs, where students are given money or gift credits to read books, have surprisingly negative effects on literacy. Such programs do get students to “read” more books, but the kind of reading they do is not ideal. Students tend to read superficially and only to get the reward. In follow-up studies, these students show not only lower reading skills but also less desire to read. 10 Nevertheless , the reward system turns reading from a fun activity into drudgery. Students think, if reading is such a rewarding experience, why do they need to pay us to do it?

It would be a mistake to conclude from a few experiments that all rewards are bad. Certainly, honest praise from a respectful teacher can do a great deal to encourage not only good behavior but also intellectual curiosity. Teachers must be aware of their students” need to feel independent and in control. 11

7

The author is considering deleting the final sentence to make the paragraph more concise. Should the author make this change?

  1. A) Yes, because it conveys information that is already implied elsewhere in the paragraph.
  2. B) Yes, because it conveys information that distracts from the discussion of student motivation.
  3. C) No, because it explains why the experiment was so difficult to conduct.
  4. D) No, because it provides information that is essential to this discussion of student motivation.

8

  1. A) NO CHANGE
  2. B) While interpreting these results
  3. C) One interpretation of these results is that
  4. D) In interpreting these results,

9

  1. A) NO CHANGE
  2. B) indications
  3. C) improvisations
  4. D) instigations

10

  1. A) NO CHANGE
  2. B) Evidently
  3. C) However
  4. D) Lastly

11

The final paragraph is notable primarily for its use of which two rhetorical devices?

  1. A) prescription and qualification
  2. B) illustration and quantification
  3. C) anecdote and metaphor
  4. D) irony and humor

Questions 12–22 are based on the following passage.

The Promise of Bio-Informatics

Although scientists have always been interested in data, modern biologists are increasingly becoming “information scientists.” Biological information science is the study of how chemical signals govern life processes. The most familiar biomolecular code is of course DNA, 12 serving as the chemical compound for the blueprint of life. Another biochemical code tells a fertilized egg how to differentiate into scores of unique cell types—heart, muscle, bone, nerve, gland, 13 blood—that assemble themselves into organs, which in turn assemble themselves into a complex organism.

12

  1. A) NO CHANGE
  2. B) this is the chemical compound serving as
  3. C) the chemical compound that serves as
  4. D) which is the chemical compound that is serving as

13

  1. A) NO CHANGE
  2. B) blood, that assemble themselves
  3. C) blood; assembling themselves
  4. D) blood—assembling itself

Yet another code governs 14 how the immune system “reads” the chemical signatures of invading pathogens and then manufactures specialized attack cells to fight infections.

15 Today we are seeing dramatic progress in all three of these areas of biochemistry. The science of genomics is developing better, cheaper, and faster ways to decode our DNA, and doctors are becoming more 16 apropos at using this information to create “personalized medicine.” Other researchers are learning how to turn the most rudimentary human cells, “stem cells,” into specialized tissues 17 for helping to repair damaged human organs. And oncologists—cancer specialists—are now coming to understand how the human immune system can be decoded to provide a crucial weapon against the most dangerous tumors.

14

Which of the following would not be an acceptable replacement for the underlined phrase?

  1. A) NO CHANGE
  2. B) the way of the immune system
  3. C) the way the immune system
  4. D) the way that the immune system

15

  1. A) NO CHANGE
  2. B) Therefore,
  3. C) Nevertheless,
  4. D) Ironically,

16

  1. A) NO CHANGE
  2. B) adept
  3. C) liable
  4. D) essential

17

  1. A) NO CHANGE
  2. B) in helping repair of
  3. C) in order to help repairing
  4. D) to help repair

18 In particular, the success of these new biological technologies 19 depends on our ability in translating vast quantities of chemical information into digital form. Specialized software and hardware 20 is needed to be developed to turn biochemical data into information that doctors and researchers can use to streamline research and make patients” lives better. Fortunately, the progress has so far been good. Since the Human Genome Project was completed in 2003, the National Human Genome Research Institute has monitored the cost of decoding a single human-sized genome. A famous law in computer science, known as “Moore”s Law,” says that the cost of processing a given quantity of information should decline by 50% every two years or so. In fact, with “second generation” techniques developed in 2008, the cost of decoding human genomes has plummeted even faster than Moore”s Law predicted. 21

18

Which choice most effectively establishes the main topic of the paragraph?

  1. A) Some scientists are skeptical about the viability of such radical new therapies.
  2. B) Researchers from all over the world are collaborating in these new discoveries.
  3. C) These new therapies and cures depend heavily on progress in the computer sciences.
  4. D) Many forms of alternative medicine are being combined with traditional therapies to treat a wide range of diseases.

19

  1. A) NO CHANGE
  2. B) depend on our ability to translate
  3. C) depends on the ability of our translating
  4. D) depends on our ability to translate

20

  1. A) NO CHANGE
  2. B) must be developed
  3. C) must develop
  4. D) needs developing

21

Which of the following statements is best supported by the data in Figure 1 ?

  1. A) Since 2003, more people have been taking advantage of genome sequencing technologies, thereby reducing costs.
  2. B) By the start of 2014, the cost per genome was less than 1% what Moore”s Law had predicted.
  3. C) Nevertheless, it still costs more than $10,000 to decode a single genome.
  4. D) The cost of genome sequencing is declining more rapidly than that of any other information-based technology.

This integration of medicine and information technology is perhaps today”s most promising scientific development. Using these new resources, perhaps 22 treatments and even cures for the most intractable diseases can be discovered by researchers .

22

  1. A) NO CHANGE
  2. B) researchers will discover treatments and even cures for the most intractable diseases
  3. C) treatments and even cures will be discovered by researchers for the most intractable diseases
  4. D) researchers have discovered treatments and even cures for the most intractable diseases

Questions 23–33 are based on the following passage.

What is Art?

Look around you. Do you see art in your immediate surroundings? What qualities 23 decide that certain things are art? Definitions of art vary widely, but most tend to fall within general notions 24 that have developed over the centuries. The technical ability of an ancient Egyptian potter to produce a well-made clay vessel defined his “art.” In Europe 600 years ago, trade and professional organizations from shoemaking to banking 25 would hold to this broad definition of art as skill in a particular field. The currently popular notion of the artist as the creator and definer of art—put simply, “Art is what artists create”—is a relatively recent one.

Some items and activities in our environment 26 stand out in a conspicuous way as somehow more “art” than others. The way that the visual elements of particular buildings, chairs, album covers, or athletic performances—their line, color, shape, texture, and other visual elements—combine to please the senses, is so satisfying that we call them beautiful. 27

Prior to the twentieth century, most philosophers of art believed that beauty was the defining feature of art. By the turn of the twentieth century, however, some aestheticians had begun to find this definition insufficient. Some said that the defining characteristic of art was the effective expression of 28 emotion; but others said the effective communication of ideas. One influential group, the formalists, argued that an object or activity qualifies as art 29 when its form is sufficiently compelling or inspiring or beautiful to provoke an intense sensory response. This echoed the ancient Greek definition of aesthetic: “of or pertaining to the senses” or “sensuous perception.”

23

  1. A) NO CHANGE
  2. B) arrange
  3. C) regulate
  4. D) determine

24

  1. A) NO CHANGE
  2. B) developing
  3. C) which are developed
  4. D) as developed

25

  1. A) NO CHANGE
  2. B) hold
  3. C) had held
  4. D) held

26

  1. A) NO CHANGE
  2. B) are conspicuous for how they stand out
  3. C) stand out
  4. D) stand out conspicuously

27

The end of the second paragraph could be best enhanced with a sentence about

  1. A) an alternate theory of beauty
  2. B) why a particular chair is beautiful
  3. C) how to design more beautiful buildings
  4. D) the benefits of art therapy

Aesthetic experiences are not as rare as you might think. If you have ever felt yourself swept away in the sensuous experience of a sports event, a musical performance, a film, a sunset, or a 30 painting: you have had an aesthetic experience. Look around again. Do any objects in your field of vision provoke an aesthetic experience? 31 Is it skill, beauty, expression, communication, compelling form, or all of the above that make these art for you? Or is it some other quality, such as originality or creativity, 32 that makes these objects or activities stand out as art?

28

  1. A) NO CHANGE
  2. B) emotion; others said it was
  3. C) emotion, others said it was
  4. D) emotion; while others said it was

29

  1. A) NO CHANGE
  2. B) if its form sufficiently compels
  3. C) if its form is sufficiently compelling
  4. D) if it”s form is sufficiently compelling

30

  1. A) NO CHANGE
  2. B) painting; you
  3. C) painting—you
  4. D) painting, you

31

  1. A) NO CHANGE
  2. B) Are they
  3. C) Do
  4. D) Are

32

  1. A) NO CHANGE
  2. B) making these objects that stand out
  3. C) that make these objects stand out
  4. D) that stands out in these objects to make them

Does setting matter? Would a sports photo become more “artistic” if it were placed in an art museum? According to George Dickie”s “institutional theory of art,” major art institutions, such as museums, determine what is art in a given culture. 33

Perhaps art is a concept that cannot have a fixed definition. Perhaps, like a living organism, it must evolve.

33

Which of the following sentences serves as the most effective concluding sentence for this paragraph?

  1. A) Dickie, a professor emeritus of philosophy at the University of Illinois, has championed the work of philosopher David Hume.
  2. B) Nearly every major city has museums dedicated to the display of works of fine art such as paintings, sculptures, and performance art.
  3. C) Other institutions, such as schools and governments, also provide definitions for concepts like education and public value.
  4. D) This theory forces us to ask: is art truly in the eye of the beholder, or is it in the eye of the artist, the curator, or some critical mass of the consuming public?

Questions 34–44 are based on the following passage.

The Little Tramp

Few people have had as strong an impact on an industry 34 as the impact that Charlie Chaplin had on the world of film. 35 Born in 1889 into an impoverished London family, Chaplin crossed the Atlantic and became a pioneer in silent comedic movies. 36 Early in his film career, Chaplin developed his signature character, the “Little Tramp,” who amused audiences repeatedly with his clever physical comedy and endearing sensitivity. Modest yet clearly intelligent, shy yet always at the center of action, the 37 Tramp”s embodiment was the genius of Chaplin”s artistry .

34

  1. A) NO CHANGE
  2. B) as what Charlie Chaplin
  3. C) than Charlie Chaplin
  4. D) as Charlie Chaplin

35

  1. A) NO CHANGE
  2. B) He was born in 1889 into
  3. C) Being born in 1889 into
  4. D) He was born in 1889 of

36

The author is considering inserting the following sentence at this point in the paragraph.

Charlie”s mother suffered from severe mental illness and was institutionalized for a significant part of Charlie”s young life.

Do you think this is appropriate?

  1. A) Yes, because it helps to explain how Chaplin became a pioneer in film.
  2. B) Yes, because it provides an important detail about health care in 19th-century London.
  3. C) No, because it detracts from the discussion of Chaplin”s impact on the film industry.
  4. D) No, because it diminishes the humorous tone of the paragraph.

37

  1. A) NO CHANGE
  2. B) genius of Chaplin”s artistry was embodied by the Tramp
  3. C) Tramp embodied the genius of Chaplin”s artistry
  4. D) Tramp”s embodiment was of the genius of Chaplin”s artistry

38 Being writer, director, and editing his own work , Chaplin faced a daunting challenge with the rise of “talkie” films, which drew audiences away from silent stars like the Tramp. Chaplin responded by taking on the additional role of composer, writing beautiful scores to accompany his films and 39 thus allowing the Tramp to remain speechless. Chaplin managed to defy the odds and maintain a remarkable level of popularity and success in the face of technological advancement. 40 Not just a master of the craft of acting and filmmaking, but also the face of a character that resonated deeply with those suffering through the Depression.

A vocal liberal in a time of conservative domination, 41 he became a target for Senator Joseph McCarthy and his House Un-American Activities Committee . While he managed to avoid being named to McCarthy”s Hollywood Ten, a list of black listed entertainment industry figures suspected of Communist connections, he drew the ire of J. Edgar Hoover 42 in the messages imbedded within his films.

Chaplin saw the dangers in Hitler”s rise to power before most of the world had heard of the dictator. He also believed that the development of the atomic bomb was a crime. Outraged at what 43 they viewed as subversive propaganda created by an immoral man, the United States government 44 eradicated Chaplin”s reentry visa during a trip to London in 1952. Sixty-three years old and tired of fighting against a force unwilling to hear his message, Chaplin agreed to exile rather than going back to America and facing interrogation and lived the rest of his years in Europe. He returned twenty years later to receive an Academy Award for lifetime achievement.

38

  1. A) NO CHANGE
  2. B) Writing, directing, and being editor of his own work,
  3. C) Writing his own work, as well as directing and editing it too,
  4. D) As the writer, director, and editor of his own work,

39

  1. A) NO CHANGE
  2. B) therefore he allowed the Tramp to remain
  3. C) allowing the Tramp thus remaining
  4. D) he allowed the Tramp thus to remain

40

  1. A) NO CHANGE
  2. B) Besides being
  3. C) He was not only
  4. D) In addition to being

41

  1. A) NO CHANGE
  2. B) Senator Joseph McCarthy and his house Un-American Activities Committee targeted him
  3. C) the House Un-American Activities Committee of Senator Joseph McCarthy targeted him
  4. D) he became targeted for Senator Joseph McCarthy”s House Un-American Activities Committee.

42

  1. A) NO CHANGE
  2. B) by
  3. C) because of
  4. D) from

43

  1. A) NO CHANGE
  2. B) it
  3. C) would have been
  4. D) were

44

  1. A) NO CHANGE
  2. B) revoked
  3. C) excluded
  4. D) abolished

STOP

If you finish before time is called, you may check your work on this section only. Do not turn to any other section of the test.

Math Test – No Calculator

25 MINUTES, 20 QUESTIONS

Turn to Section 3 of your answer sheet to answer the questions in this section.

DIRECTIONS

For questions 1–15 , solve each problem, choose the best answer from the choices provided, and fill in the corresponding circle on your answer sheet. For questions 16–20, solve the problem and enter your answer in the grid on the answer sheet. Please refer to the directions before question 16 on how to enter your answers in the grid. You may use any available space in your test booklet for scratch work.

NOTES

  1. The use of a calculator is NOT permitted.
  2. All variables and expressions used represent real numbers unless otherwise indicated.
  3. Figures provided in this test are drawn to scale unless otherwise indicated.
  4. All figures lie in a plane unless otherwise indicated.
  5. Unless otherwise indicated, the domain of a given functionf is the set of all real numbers for which f (x ) is a real number.

REFERENCE

The number of degrees of arc in a circle is 360.

The number of radians of arc in a circle is 2π.

The sum of the measures in degrees of the angles of a triangle is 180.

1

If 8x + 6 = 6m , what is the value of 4x + 3 in terms of m ?

  1. A) 2m– 3
  2. B) 2m
  3. C) 3m– 3
  4. D) 3m

2

3x + 4y = 18

Which of the following ordered pairs (x , y ) is a solution of the system of equations above?

  1. A) (2, 3)
  2. B) (3, 2.25)
  3. C) (4, 1.5)
  4. D) (4, 6)

3

Which of the following is equivalent to

  1. A)
  2. B)
  3. C)
  4. D)

4

If x − 3 is a factor of the expression x 2 + kx + 12, what is the value of k ?

  1. A) −7
  2. B) −5
  3. C) 5
  4. D) 7

5

Note: Figure not drawn to scale.

In the figure above, P is the center of a circle and AC is its diameter. What is the value of x ?

  1. A) 60
  2. B) 50
  3. C) 40
  4. D) 30

6

The n th term of a sequence is given by the expression bn + 4, where b is a positive constant. Which of the following is necessarily equal to b ?

  1. A) the value of the first term
  2. B) the difference between the fourth term and the third term
  3. C) the average (arithmetic mean) of the first three terms
  4. D) the ratio of the second term to the first term

7

If , where n > 0, what is the value of m in terms of n ?

  1. A)
  2. B)
  3. C)
  4. D)

8

One bag of grass seed can cover 5,000 square feet of new lawn. If each bag costs p dollars, which of the following expressions gives the cost, in dollars, to cover a new rectangular lawn that measures a feet by b feet?

  1. A)
  2. B)
  3. C)
  4. D) 5,000abp

9

If , where m > 0, what is the least possible value of m ?

  1. A) 6.5
  2. B) 7
  3. C) 7.5
  4. D) 8

10

If f (x ) = 3x + n , where n is a constant, and f (2) = 0, then f (n ) =

  1. A) −24
  2. B) −18
  3. C) −12
  4. D) 12

11

If s, t, u , and v are the coordinates of the indicated points on the number line above, which of the following is greatest?

  1. A) |sv |
  2. B) |st |
  3. C) |s+v |
  4. D) |u+v |

12

How many solutions to the equation 4 cos x = 1 lie between x = 0 and x = 3π?

  1. A) Two
  2. B) Three
  3. C) Four
  4. D) Six

13

If , which of the following is NOT equal to i 3 + i ?

  1. A) (2i)2+ 4
  2. B) 2 − 2i4
  3. C) 2i2− 2
  4. D) i4− 1

14

If m > 1, which of the following could be the graph of y = – (x + m )2 + m in the xy -plane?

A)

B)

C)

D)

15

x − 3y = −2

The values of x that satisfy the system of equations above also satisfy which of the following equations?

  1. A) (x− 5)(x+ 3) = 0
  2. B) (x− 3)(x+ 5) = 0
  3. C) (x− 2)(x− 5) = 0
  4. D) (x+ 2)(x+ 5) = 0

DIRECTIONS

For questions 16–20, solve the problem and enter your answer in the grid, as described below, on the answer sheet.

  1. Although not required, it is suggested that you write your answer in the boxes at the top of the columns to help you fill in the circles accurately. You will receive credit only if the circles are filled in correctly.
  2. Mark no more than one circle in any column.
  3. No question has a negative answer.
  4. Some problems may have more than one correct answer. In such cases, grid only one answer.
  5. Mixed numberssuch as must be gridded as 3.5 or .

(If is entered into the grid as , it will be interpreted as , not ).

  1. Decimal answers:If you obtain a decimal answer with more digits than the grid can accommodate, it may be either rounded or truncated, but it must fill the entire grid.

16

If , and b = 4, what is the value of a ?

17

What is the smallest integer value of x such that is less than 1?

18

If the area of the figure above is square units, what is its perimeter?

19

What is one possible solution to the equation

20

In the figure above, triangle ABC has an area of 19. What is the value of tan θ ?

STOP

If you finish before time is called, you may check your work on this section only. Do not turn to any other section of the test.

Math Test – Calculator

55 MINUTES, 38 QUESTIONS

Turn to Section 4 of your answer sheet to answer the questions in this section.

DIRECTIONS

For questions 1–30, solve each problem, choose the best answer from the choices provided, and fill in the corresponding circle on your answer sheet. For questions 31–38, solve the problem and enter your answer in the grid on the answer sheet. Please refer to the directions before question 31 on how to enter your answers in the grid. You may use any available space in your test booklet for scratch work.

NOTES

  1. The use of a calculator is permitted.
  2. All variables and expressions used represent real numbers unless otherwise indicated.
  3. Figures provided in this test are drawn to scale unless otherwise indicated.
  4. All figures lie in a plane unless otherwise indicated.
  5. Unless otherwise indicated, the domain of a given functionf is the set of all real numbers for which f (x ) is a real number.

REFERENCE

The number of degrees of arc in a circle is 360.

The number of radians of arc in a circle is 2π.

The sum of the measures in degrees of the angles of a triangle is 180.

1

The fraction is equal to 0.8. What is the value of n ?

  1. A) 4
  2. B) 8
  3. C) 12
  4. D) 16

2

The median of the numbers x , 10, and 12 is 12. Which of the following CANNOT be the value of x ?

  1. A) 8
  2. B) 12
  3. C) 16
  4. D) 20

3

Based on the ordered pairs in the table above, which of the following could express a relationship between x and y ?

  1. A) y=x + 4
  2. B) y= 2x
  3. C) y= 2x+ 2
  4. D) y= 2x+ 4

4

The average (arithmetic mean) of a set of 3 positive integers is m . If the number 24 is added to this set, what is the average (arithmetic mean) of the new set of numbers?

  1. A)
  2. B)
  3. C) m+ 8
  4. D)

5

If , what is the value of x ?

  1. A) −3
  2. B) −2
  3. C)
  4. D)

6

The Municipal Electric Company charges each household $0.15 per kilowatt-hour of electricity plus a flat monthly service fee of $16. If a household uses 30 kilowatt-hours of electricity and is charged $P in a given month, which of the following equations is true?

  1. A) 0.15(30) + 16 =P
  2. B) 0.15P+ 16 = 30
  3. C)
  4. D)

7

Alyssa determines that a floating balloon is 1,200 meters away from her at an angle of 42° from the ground, as in the figure above. What is the height, h , of the balloon from the ground? (sin 42° = 0.669, cos 42° = 0.743, tan 42° = 0.900)

  1. A) 802.8 meters
  2. B) 891.6 meters
  3. C) 1,080 meters
  4. D) 1,793 meters

8

In the figure above, line segments and intersect at point E . What is the value of x ?

  1. A) 60°
  2. B) 65°
  3. C) 70°
  4. D) 75°

Questions 9 and 10 are based on the graph below.

9

A university surveyed 24 economics majors and asked them how many credits they received the previous semester. The results are represented in the graph above. What percentage of these students received 15 or more credits that semester?

  1. A) 29%
  2. B)
  3. C)
  4. D) 54%

10

What is the median number of credits these students received the previous semester?

  1. A) 10.5
  2. B) 11.5
  3. C) 12
  4. D) 12.5

11

If a and b are the coordinates of two points on the number line, then which of the following is equivalent to the statement that the absolute distance from a to b is greater than the absolute distance from −2 to 6?

  1. A) |a| > −2 and |b| > 6
  2. B) |ab | > −8
  3. C) |a+ 2| > |b− 6|
  4. D) |ab | > 8

12

Note: Figure not drawn to scale.

In the figure above, rectangle ABCD is inscribed in the circle with center O . What is the area of the circle?

  1. A) 26π
  2. B) 121π
  3. C) 144π
  4. D) 169π

13

Everyone in Niko”s class has a different birth date. If Niko is both the 8th oldest person and the 12th youngest person in his class, how many students are in Niko”s class?

  1. A) 18
  2. B) 19
  3. C) 20
  4. D) 21

14

If , which of the following is equivalent to (2 − i )(3 − 2i )?

  1. A) 8 − 7i
  2. B) 6 + 2i
  3. C) 6 − 6i
  4. D) 4 − 7i

15

If f (x ) = (x 2 )−2b and f (3) = 3, what is the value of b ?

  1. A)
  2. B)
  3. C)
  4. D)

16

In a survey of 80 students, 55 students stated that they play a varsity sport, and 35 stated that they are taking at least one AP level course. Which of the following statements must be true?

  1. A) At least 10 of these students are both playing a varsity sport and taking at least one AP level course.
  2. B) Less than half of the students who play a varsity sport are also taking at least one AP level course.
  3. C) The number of students who do not play a varsity sport is greater than the number of students who do not take at least one AP level course.
  4. D) At least one student who takes an AP level course does NOT play a varsity sport.

17

Note: Figure not drawn to scale.

In the figure above, AB = BC . If has a slope of m and has a slope of n , what is the value of mn ?

  1. A) −9
  2. B)
  3. C)
  4. D) 9

18

The functions f , g , and h are defined by the equations f (x ) = x 2 , g (x ) = x , and . Which of the following must be true?

  1. A)
  2. B)
  3. C)
  4. D)

19

Which of the following scatterplots provides the strongest evidence in support of the hypothesis that y varies inversely as the square of x ?

A)

B)

C)

D)

20

The bird department of a pet store has 12 canaries, 30 finches, and 18 parrots. If the pet store purchased n more finches, then 80% of its birds would be finches. Which of the following equations must be true?

  1. A)
  2. B)
  3. C)
  4. D)

21

Let function f (x ) be defined by the equation f (x ) =x 2 − 1. If b is a positive real number, then

  1. A)
  2. B)
  3. C)
  4. D)

22

The value of y varies with x according to the equation y = kx 2 , where k > 0. When the value of x increases from 3 to 12, which of the following best describes the behavior of y ?

  1. A) It increases by 81.
  2. B) It increases by 135.
  3. C) It is multiplied by 4.
  4. D) It is multiplied by 16.

23

If the function f is defined by the equation f (x ) = k (x + 6)(x − 1), where k > 5, then which of the following is equivalent to f (7)?

  1. A) f(−78)
  2. B) f(−12)
  3. C) f(−2)
  4. D) f(78)

24

After its initial offering, the price of a stock increased by 20% in the first year, decreased by 25% in the second year, then increased by 10% in the third year. What was the net change in the stock price over the entire three-year period?

  1. A) It increased by 5%.
  2. B) It increased by 1%.
  3. C) It decreased by 1%.
  4. D) It decreased by 5%.

25

If y = x 2 , where x ≠ 0, and w = y 6 , which of the following expresses the value of in terms of x ?

  1. A) x2
  2. B) x4
  3. C) x5
  4. D) x6

26

With the exception of the shaded squares in the first row and first column, every square in the table above is to be filled in with a number equal to the sum of the number directly above it and the number directly to its left. For instance, the number 7 in the second row is the sum of 3 in the square above it and 4 in the square directly to its left. What is the value of x ?

  1. A) 16
  2. B) 84
  3. C) 96
  4. D) 112

27

3x 2 = 4x + c

In the equation above, c is a constant. If x = −1 is a solution of this equation, what other value of x satisfies the equation?

  1. A)
  2. B)
  3. C)
  4. D) 7

28

Note: Figure not drawn to scale.

The figure above shows two concentric circles with center O . If OD = 3, DB = 5, and the length of arc AB is 5π, what is the length of arc CD ?

  1. A)
  2. B)
  3. C) 3π
  4. D)

Questions 29 and 30 refer to the following graph.

29

Four different classes at Corbett Elementary School participated in two fund-raisers last year, one in February and another in May. The rates of participation for each class are recorded in the graph above. Which class had the greatest change in percent participation from the February fund-raiser to the May fund-raiser?

  1. A) Class A
  2. B) Class B
  3. C) Class C
  4. D) Class D

30

If there were 20 students each in Class A and Class C, and 30 students each in Class B and Class D, how many students participated in the May fund-raiser?

  1. A) 71
  2. B) 72
  3. C) 74
  4. D) 76

DIRECTIONS

For questions 31–38, solve the problem and enter your answer in the grid, as described below, on the answer sheet.

  1. Although not required, it is suggested that you write your answer in the boxes at the top of the columns to help you fill in the circles accurately. You will receive credit only if the circles are filled in correctly.
  2. Mark no more than one circle in any column.
  3. No question has a negative answer.
  4. Some problems may have more than one correct answer. In such cases, grid only one answer.
  5. Mixed numberssuch as must be gridded as 3.5 or .

(If is entered into the grid as , it will be interpreted as , not .)

  1. Decimal answers:If you obtain a decimal answer with more digits than the grid can accommodate, it may be either rounded or truncated, but it must fill the entire grid.

31

If , what is the value of

32

In the xy -plane, the graph of the equation y = 3x 2kx – 35 intersects the x -axis at (5, 0). What is the value of k ?

33

In the figure above, triangle FGH is inscribed in the circle with center P . If the area of the circle is π, what is the area of triangle FGH ?

34

If , what is one possible value of 6t ?

35

If cos (x – π) = 0.4, what is the value of sin2 x ?

36

If one pound of grain can feed either 5 chickens or 2 pigs, then ten pounds of grain can feed 20 chickens and how many pigs?

Questions 37 and 38 are based on the following information

The table above shows information about the tickets sold for a recent performance by a theater troupe. The total revenue in ticket sales for this performance was $15,000.

37

If 15 more tickets were sold in the second mezzanine than in the first mezzanine, what is the total number of tickets that were sold for this performance?

38

Before the tickets for this performance went on sale, a consultant for the theater had predicted that n , the number of tickets sold per section, would vary with p , the price in dollars for a ticket in that section, according to the formula . By how many tickets did this model underestimate the actual total number of tickets sold?

STOP

If you finish before time is called, you may check your work on this section only. Do not turn to any other section of the test.

Essay

50 MINUTES, 1 QUESTION

DIRECTIONS

As you read the passage below, consider how Ellis Parker Butler uses

  • evidence, such as facts or examples, to support his claims
  • reasoning to develop ideas and connect claims and evidence
  • stylistic or persuasive elements, such as word choice or appeals to emotion, to add power to the ideas expressed

Adapted from Ellis Parker Butler, “On Spelling.” Originally published in 1906.

1 My own opinion of the spelling profession is that it has nothing to do with genius, except to kill it. I know that Shakespeare was a promiscuous sort of speller, even as to his own name, and no one can deny that he was a greater genius than Noah Webster. The reason America so long lagged behind Europe in the production of genius is that America, for many decades, was the slave of the spelling-book. No man who devotes the fiery days of his youth to learning to spell has time to be a genius.

2 My wife, Serena, says, and I agree with her, that it is the jealousy of a few college professors who are trying to undermine the younger writers. They know that it is excusable to spell incorrectly now, but they want this new phonetic spelling brought into use so that there shall be no excuse for bad spelling, and that then, Serena says, self-made authors like me, who never can spell but who simply blaze with genius, will be hooted out of the magazines to make room for a stupid sort of literature that is spelled correctly. Serena looks upon the whole thing as a direct, personal stab at me. I look at it more philosophically.

3 To me it seems that the spelling reformers are entirely on the wrong track. Their proposed changes are almost a revolution, and we Americans do not like sudden changes. We like our revolutions to come about gradually. Think how gradually automobiles have come to pass. If, in our horse age, the streets had suddenly been covered with sixty horsepower snorters going thirty miles an hour and smelling like an eighteenth-century literary debate, and killing people right and left, we Americans would have arisen and destroyed every vestige of the automobile. But the automobile came gradually—first the bicycle, then the motorcycle, and so, by stages, to the present monsters. So slowly and progressively did the automobile increase in size and number that it seemed a matter of course. We take to being killed by the automobile quite naturally now.

4 Of course, the silent letters in our words are objectionable. They are lazy letters. We want no idle class in America, whether tramp, aristocrat, or silent letter, but we do not kill the tramp and the aristocrat. We set them to work, or we would like to. My theory of spelling reform is to set the idle letters to work.

5 Take that prime offender, although . Altho does all the work, and ugh sits on the fence and whittles. I would put ugh to work. Ugh is a syllable in itself. I would have the ugh follow the pronounced altho as a third syllable. Doubtless the asthmatic islanders who concocted our English language actually pronounced it so.

6 I propose to have some millionaire endow my plan, and Serena and I will then form a society for the reforming of English pronunciation. I will not punch out the i of any chief, nor shall any one drag me from any programme, however dull. I will pronounce programme as it should be pronounced—programmy —and, as for chief , he shall be pronounced chy-ef .

7 The advantage of this plan is manifest. It is so manifest that I am afraid it will never be adopted.

8 Serena”s plan is, perhaps, less intellectual, but more American. Serena”s plan is to ignore all words that contain superfluous letters. She would simply boycott them. Serena would have people get along with such words as are already phonetically spelled. Why should people write although , when they can write notwithstanding that , and not have a silent letter in it? I have myself often written a phrase twelve words long to stand instead of a single word I did not know how to spell. In fact, I abandoned my Platonic friendship for Serena, and replaced it with ardent love, because I did know how to spell sweetheart , but could not remember whether she was my friend or freind .

Write an essay in which you explain how Ellis Parker Butler builds an argument to persuade his audience that American English spelling conventions of 1906 need to be reformed. In your essay, analyze how Butler uses one or more of the features listed in the box above (or features of your own choice) to strengthen the logic and persuasiveness of his argument. Be sure that your analysis focuses on the most relevant features of the passage.

Your essay should NOT explain whether you agree with Butler”s claims, but rather explain how Butler builds an argument to persuade his audience.

SAT PRACTICE TEST 1 ANSWER KEY

Section 1: Reading

  1. A
  2. C
  3. A
  4. D
  5. D
  6. B
  7. A
  8. C
  9. A
  10. A
  11. B
  12. B
  13. D
  14. C
  15. B
  16. D
  17. C
  18. A
  19. D
  20. A
  21. C
  22. B
  23. C
  24. A
  25. D
  26. D
  27. C
  28. B
  29. B
  30. B
  31. D
  32. B
  33. B
  34. A
  35. D
  36. A
  37. A
  38. C
  39. A
  40. C
  41. D
  42. A
  43. C
  44. D
  45. C
  46. C
  47. D
  48. C
  49. A
  50. C
  51. D
  52. A

Total Reading Points (Section 1)

Section 2: Writing and Language

  1. C
  2. A
  3. B
  4. B
  5. B
  6. A
  7. D
  8. C
  9. A
  10. B
  11. A
  12. C
  13. A
  14. B
  15. A
  16. B
  17. D
  18. C
  19. D
  20. B
  21. B
  22. B
  23. D
  24. A
  25. D
  26. C
  27. B
  28. B
  29. C
  30. D
  31. A
  32. A
  33. D
  34. D
  35. A
  36. C
  37. C
  38. D
  39. A
  40. C
  41. A
  42. C
  43. B
  44. B

Total Writing and Language Points (Section 2)

Section 3: Math (No Calculator)

  1. D
  2. A
  3. C
  4. A
  5. C
  6. B
  7. A
  8. B
  9. C
  10. A
  11. A
  12. B
  13. C
  14. D
  15. B
  16. 4.5 or 9/2
  17. 7
  18. 8.4 or 42/5
  19. 5 or 7
  20. 8/13 or .615

Total Math Points (Section 3)

Section 4: Math (Calculator)

  1. D
  2. A
  3. C
  4. B
  5. C
  6. A
  7. A
  8. C
  9. C
  10. D
  11. D
  12. D
  13. B
  14. D
  15. B
  16. A
  17. A
  18. D
  19. A
  20. C
  21. B
  22. D
  23. B
  24. C
  25. D
  26. D
  27. C
  28. B
  29. C
  30. C
  31. 0.8 or 4/5
  32. 8
  33. .96
  34. 4.29 ≤ x ≤ 4.79
  35. .84
  36. 12
  37. 371
  38. 25

Total Math Points (Section 4)

SCORE CONVERSION TABLE

Scoring Your Test

  1. Use the answer key to mark your responses on each section.
  2. Total the number of correct responses for each section:
  3. Add the raw scores for sections 3 and 4. This is yourMath Raw Score : ____________
  4. Use theTable 1 to calculate your Scaled Test and Section Scores (10–40) .
  5. Add theReading Test Scaled Score and the Writing and Language Test Scaled Score and multiply this sum by 10 to get your Reading and Writing Test Section Score (20–80) .

Table 1: Scaled Section and Test Scores (10–40)

SAT PRACTICE TEST 1 DETAILED ANSWER KEY

Section 1: Reading

1 . A

Detail

In lines 10–25, the author of Passage 1 discusses how farm subsidies are used both to lower food prices (by subsidizing farmers) when prices get too high, and to raise them (by paying farmers to leave their land fallow) when prices get too low. Although he disputes that these efforts to stabilize prices are worth the cost, he does indicate that they work. In the first paragraph of Passage 2 (lines 60–73), the author indicates that farm subsidies protect consumers from price spikes (lines 69–70).

2 . C

Interpretation

In the first paragraph of Passage 1, the statement that something is rotten down on the farm (line 1) introduces the author”s discussion of the U.S. farm subsidies program, which he claims gives away millions of taxpayer dollars to farmers who are actually ineligible for the program (lines 6–8) and is rife with inefficiencies (line 16).

3 . A

Cross-Textual Inference

The thesis of Passage 2 is that U.S. farm subsidies are vital (line 61) to both farmers and American consumers. Therefore, he regards the taxes (line 15) we pay for these subsidies to be a worthwhile expenditure .

4 . D

Inference

In lines 8–9, the author of Passage 1 states that the report about corruption and incompetence in the U.S. farm subsidies program should horrify lawmakers, but it probably won”t . He explains why in the last paragraph (lines 51–59): Rich corporate farmers are an enormously powerful lobby in American politics , contributing nearly $100 million into political campaigns every year .

5 . D

Textual Evidence

As the explanation to question 4 explains, the evidence for this answer is found in the last paragraph, particularly lines 53–55.

6 . B

Passage Comparison

Although the author of Passage 1 does not think that the stabilization of commodity prices is worth the cost of higher taxes (lines 19–25), the author of Passage 2 indicates that price spikes (line 69) can be devastating to both farmers and consumers.

7 . A

Specific Purpose

The 2009 poll cited in lines 40–43 indicates that most Americans support farm subsidies for small family farms, confirming the author”s statement that Americans feel that we need these subsidies to save the small family farmer . Choice (B) is incorrect because, although the author himself goes on to refute this misconception, the results of the poll do not. Rather, they confirm a general sentiment . Choice (C) is incorrect because the poll does not indicate any shift away from the discussion about the ineffectiveness of the U.S. farm subsidies program. Choice (D) is incorrect because the word Indeed (line 40) indicates that this result is unsurprising to him.

8 . C

Data Analysis

The thesis of Passage 1 is that something is rotten down on the farm (line 1), namely, the fact that, in a recent seven-year period, the U.S. Congress has doled out more than $114 billion to farmers (lines 10–11) through a program that uses administrators who are ill-trained and poorly monitored (lines 5–6) and that implements programs that are not worthwhile to taxpayers, that are actually harming American exporters (lines 33–34) and that make it much harder for the small family farmers to compete (lines 47–48). The graph in Figure 1 , however, shows about a 40% decline in these subsidies from 2000 to 2012, perhaps undercutting the author”s claim that these subsidies are an overall burden on the American taxpayer.

He would most likely, then, choose to focus on the component of these subsidies that has grown significantly in the 15 years indicated on this graph, namely, crop insurance subsidies, which have expanded at a fairly steady rate and grew by about 500% from 1998 to 2012. As the introduction to the passage indicates, this program takes money from taxpayers to help farmers to buy crop insurance, thereby providing direct entitlements not only to farmers but also to insurance companies.

Choice (A), the general decline in total farm subsidies from 2005 to 2012 , does not help the author make the point that these subsidies are a burden to American taxpayers. Similarly, choice (B), the overall rate of change in commodity subsidies from 1998 to 2012 does not help his thesis, because after the first several years, the trend is generally downward. Choice (D), the sudden spike in disaster subsidies from 2004 to 2005 , also does not support his thesis, because he does not make any particular claims about the benefit of disaster subsidies.

9 . A

Data Analysis

The thesis of Passage 2 is that farm subsidies in the United States are vital (line 61) and not as burdensome to American taxpayers as the critics claim (lines 62–63). Therefore, the author of Passage 2 would most likely cite evidence that the total cost of the subsidies program is declining.

10 . A

Cross-Textual Inference

The author of Passage 1 indicates that we taxpayers will pay more in taxes than we will ever get back in lower corn or wheat prices (lines 16–18), thereby indicating that the benefit of lower prices (line 85) is offset by its costs .

11 . B

Cross-Textual Comparison

The author of Passage 1 mentions that the U.S. farm subsidy programs use administrators who give away millions of taxpayer dollars to farmers who are actually ineligible for the program (lines 6–8) and are rife with inefficiencies(line 16) to make the argument that they are not worthwhile to taxpayers. This is an appeal to the reader”s distaste for ineptitude (incompetence).

Although this could also be seen as an appeal to the reader”s fiscal prudence (sense of responsibility), the author of Passage 2 makes the same kind of appeal when he indicates that these subsidies are vital (line 61) to preventing price spikes (line 69) and are not as burdensome to American taxpayers as the critics claim (lines 62–63). Since the question asks us to find an appeal that is NOT also found in Passage 2, choice (D) is incorrect.

12 . B

Interpretation

When the author of Passage 1 states that Agribusiness and farm insurance lobbies pump nearly $100 million into political campaigns every year, and the floodgates show no sign of closing (lines 53–55), he suggests that there seem to be no controls against this unscrupulous (unethical) funding of political campaigns by those who benefit from the decisions of those politicians.

13 . D

Tone and Characterization

The narrator says that Reverend Jansen bent down in a cloud of Aqua Velva (lines 5–6) and told her not to worry (line 12). He then describes to the narrator why God called [her] Korean parents home (line 13). All of these descriptions work together to portray someone who is acting in a condescending and patronizing manner to a young child.

14 . C

Interpretation

The statement that the narrator”s mother had been murdered (line 2) is later explained to refer to the narrator”s interpretation of the fact that she was told that “God called [her] Korean parents home” (line 13) and that “It was all part of His plan” (line 18), in other words, her death was deliberate . At first, choice (D) may seem plausible, because in lines 36–43, Sarah”s mother does not want to talk about Sarah”s biological mother. However, the passage makes it clear that the narrator attributed the “murder” to a divine plan (God kills, I thought then , line 27) rather than to any intention of her adoptive family.

15 . B

Tone and Diction

The description of Reverend Jansen”s eyes and breath in lines 13–17 indicates that he is somewhat emotionally detached (his eyes sliding sideways , lines 15–16) and that Sarah is likewise emotionally detached from him and his profound claims, instead distracted by his breath that smelled vaguely of toast (lines 16–17). These descriptions surprise us, because they are so incongruent with the expectation of respect for and contemplation of the reverend”s deep spiritual pronouncements.

16 . D

Interpretation

The narrator has Korean heritage, yet she grew up in a house in which Korea had always been the oddly charged word, never to be mentioned in connection with [Sarah], the same way [they] never said “Uncle Henry” and “alcoholic” in the same sentence (lines 44–47). The narrator”s mother, Christine, thought [Sarah] needed to protected from (line 49) her ethnicity. In other words, she regarded Sarah”s ethnicity as an unfortunate fact .

17 . C

Textual Evidence

As the explanation to question 16 makes clear, the best evidence for the previous answer is in lines 48–52.

18 . A

Literary Device

The contrast between murder and Christmas and the Easter Bunny (line 28) is a classic example of juxtaposition , the act of placing together two images with highly contrasting effects.

19 . D

Interpretation

The paragraph states that Christine begins her reply patiently (line 36), which might suggest that she is demonstrating motherly sympathy . However, sympathy means “a feeling of common understanding,” and the rest of Christine”s reply suggests that she is disappointed (line 43) with Sarah rather than sympathetic with her. The point of the paragraph is that Christine is not emotionally ready (it makes me sad , line 40) to discuss something that her eight-year-old adopted daughter clearly wants to discuss, that is, she is emotionally immature .

20 . A

Word in Context

When the narrator states that Korea was the oddly charged word (lines 44–45), she means that it was a word that was never to be mentioned (lines 45–46), because it was associated with potentially negative feelings. That is, it was an emotionally loaded word.

21 . C

Interpretation

In lines 44–60, the narrator describes her Uncle Henry as an “alcoholic” (line 47) who sat drinking at family cookouts at the far corner of our yard, away from everyone (line 60). This is treatment appropriate to a pitiable embarrassment rather than a stern patriarch or noble hero . There is also no indication, despite Sarah”s parents” discomfort with discussing her heritage, that Uncle Henry is a bigoted lout .

22 . B

Tone and Inference

The reference to Bryant Gumbel”s cheery smirk (line 81) follows the description of his television segment during the Olympic games about how Korea had become one of the economic miracle countries (line 74). According to the narrator, the cheery smirk seemed to say Top that, Singapore! thereby indicating that he admired Korea”s ability to compete economically with other strong countries.

23 . C

General Purpose

The passage as a whole describes the spectacular cyclorama (line 9) that is known as the cosmic microwave background (CMB) radiation, a 13 billion year-old panoramic snapshot of the universe as it appeared the moment it first released its primordial photons (lines 14–18). It then goes on to discuss the precise measurements that scientists have taken of this radiation and what they tell us about the early universe. In other words, the passage as a whole is discussing the analysis and significance of a cosmological phenomenon .

24 . A

Specific Purpose

The description of the Gettysburg Cyclorama in the first paragraph is used to draw an analogy between two cataclysmic historical event[s] (lines 9–10), one of which we can see with our own eyes and one of which we can only detect with special tools. The answer is not (B), because although this Cyclorama depicts a historic battle, it is not itself a historical precedent (an event that serves as a model for future similar events). Choice (C) is incorrect because the painting is depicted neither as quaint nor anachronistic (out of historical order). Choice (D) is incorrect because although the passage later indicates that the discovery of the CMB was somewhat accidental , the Cyclorama was not.

25 . D

Interpretation

The author indicates that to appreciate [the] full splendor[of the Cosmic Background Radiation], you would have to be able to see microwaves (lines 11–13). In other words, the disappointment is in the fact that we can”t see the spectacular cyclorama (line 8) that is the cosmic microwave background; it is an inaccessible phenomenon , at least to our naked eyes.

26 . D

Specific Purpose

The discussion in lines 23–28 concerns the emergence of the first photons (light particles) in the early universe. In saying that the universe was “invisible” (lines 25–26), the author means that photons—the particles that are required for us to be able to detect something visually—did not yet exist. Calling the early universe “invisible” is somewhat inappropriate, since there were no eyes to see it anyway during that stage in its development, so the quotes are drawing attention to the fact that these terms are being used to make a technical point a bit clearer by using common words that correspond with our everyday experience.

27 . C

Interpretation

The moment a swaddled one-day-old opens its eyes (lines 22–23) refers to the moment [the early universe] first released its primordial photons (lines 17–18) which we now refer to as the cosmic microwave background radiation. The discussion in the next paragraph (lines 24–36) explains that these early photons were previously trapped in an opaque fog of hydrogen plasma (line 29). Choices (A) and (D) are incorrect because this moment describes when the photons were released, not when they were first discovered by humans. Choice (B) is incorrect because these particles, as it is explained in the third paragraph, were released 380,000 years after the Big Bang.

28 . B

Word in Context

The distinctive spectrum (line 64) refers to the precise “blackbody” curve for 2.75° Kelvin as shown in Figure 1 . It is the particular set of wavelength intensities that distinguish blackbody radiation from ordinary radiation, and confirm Gamow”s theory about the origin of the signals detected at Murray Hill.

29 . B

Inference

The passage states that Penzias and Wilson were initially troubled (line 71) by the signals that turned out to be from the CMB radiation, and in fact mistakenly attributed them to pigeon droppings (lines 74–75). This indicates that they were not looking for these signals, nor did they know how to interpret them. The work they did to receive the Nobel prize, therefore, was the result of an accidental discovery .

30 . B

General Structure

As the explanation of question 29 indicates, lines 72–75 indicate that Penzias and Wilson did not understand the nature of the signals they were receiving, atttributing them erroneously to pigeon droppings .

31 . D

Data Analysis

Figure 1 shows the blackbody spectrum for various temperatures, and compares these to the measurements taken of the cosmic microwave background, showing that the CMB radiation has a nearly perfect blackbody spectrum .

32 . B

Word in Context

Figure 2 shows a panoramic map of the cosmic background radiation, showing that it did not originate from just one point in space , but rather from every direction.

33 . B

General Purpose

The first paragraph states that the people have a right . . . to that most dreaded and envied kind of knowledge of the characters and conduct of their rulers (lines 5–9). In other words, they have the right to learn about who their leaders are and what they do. Choice (A) is incorrect because the right to pursue academic interests is discussed somewhat in the second paragraph (Let us dare to read, think, speak, and write , line 33) but not in the first. Choice (C) is incorrect, because although Adams says that the art of printing should be encouraged (line 28), this is not the primary point of the paragraph. Rather, it is secondary to the point that citizens should be well informed. Choice (D) is incorrect because although the first passage mentions the right of citizens to revoke the authority (line 13) of their leaders, it does not discuss the right of citizens to propose legislation themselves.

34 . A

Word in Context

The statement that the people have a right to revoke the authority that they themselves have deputed, and to constitute abler and better agents, attorneys and trustees (lines 12–15) means that the people have the right to place in power better leaders to replace those whose authority has been revoked.

35 . D

Interpretation

The passage indicates that our forefathers endured physical deprivation in the form of the hunger, the nakedness, [and] the cold (line 54), political oppression in the form of domestic tyrants and usurpers (lines 44–45), and arduous physical labor in the form of the severe labors of clearing their grounds, building their houses, [and] raising their provisions (lines 55–57). It does not mention, however, that they endured any feelings of despair . In fact, it says that they endured these with the hopes and expectations which constantly supported and carried them through all hardships with patience and resignation (lines 60–63).

36 . A

Interpretation

The very first sentence states that all people have a desire to know (line 5), that is, a curious nature . Choice (B) is incorrect, because although the passage discusses at length the people”s right to revoke the authority of those in power, it does not claim that people themselves have a desire for power. Choice (C) is incorrect, because although the passage discusses the right of the people to revoke the authority of bad rulers, and mentions the inherent rights of mankind against foreign and domestic tyrants and usurpers (lines 43–45), it does not state specifically that the people have any dread of tyranny. Choice (D) is incorrect because the passage does not discuss thrift (resourcefulness with money).

37 . A

Textual Evidence

As the explanation to question 36 explains, the best evidence for the previous answer is found in the very first sentence of the passage.

38 . C

Interpretation

The phrase every order and degree among the people (line 34) refers to the entire society that Adams is addressing throughout the second paragraph.

39 . A

Structural Comparison

The second paragraph is characterized primarily by its use of the imperative mood: Let us dare … Let every order … Let them … Let us study … Let us read … Let us examine … These sentences therefore have a much more urgent and suggestive diction than do the sentences in the first paragraph. While the first paragraph is primarily descriptive of the rights of free citizens, the second is prescriptive of their corresponding duties.

40 . C

Interpretation

When Adams says Let us examine the nature of that power (lines 50–51) he is referring to the cruel power that drove [our forefathers] from their homes (line 52), that is, the domestic tyrants (line 44) that made it difficult for them to remain in their native countries. Clearly, then power refers to a despotic (tyrannical) agent .

41 . D

Interpretation

Although this paragraph does discuss the privations endured by our forefathers in the form of the hunger, the nakedness, [and]the cold (line 54) and does implicitly warn against the dangerous posed by an ignorant populace because it strongly encourages us to read, think, speak, and write (line 33), this particular sentence is referring specifically to arbitrary kings and cruel priests (line 45). So the phrase the gates of earth and hell (line 46) is referring to the brutality of oppressive leaders who persecute us in our worldly existence and about an otherworldly existence.

42 . A

Purpose

The sentence Let us read and recollect and impress upon our souls the views and ends of our own more immediate forefathers in exchanging their native country for a dreary, inhospitable wilderness (lines 46–50) invites us to learn about the beliefs and motivations of our forefathers who came to America to escape oppression. In other words, Adams wants to remind the reader of the importance of liberty .

43 . C

Specific Purpose

The first paragraph characterizes the Southern Ocean as a foreboding place by evoking images of its chilly current (lines 2–3), the dangerous icebergs [that] hide in its gloom (lines 4–5), and the churning swells [that] sometimes serve up freak waves that can easily flip ships (lines 5–6). Such images might be used to make a case for the improbability of Smetacek”s success or the pessimism of Smetacek”s detractors , but his portion of the passage contains no such pessimism. Rather goes on directly to explain the promise of Smetacek”s work. This description, therefore, must be regarded as emphasizing the boldness of Smetacek”s experiment .

44 . D

Word in Context

The phrase the base layer of the food chain refers to plankton”s role in the global ecosystem, specifically how it serves as the foundation of the food chain.

45 . C

Interpretation

Although many environmentalists may well regard fertilizer run-off from farm fields (line 23) as an unfortunate by-product of farming, or an environmental hazard , the author here presents it as supplying some of the key nutritional elements (line 21) for cyanobacteria. Therefore, it is a potential sustenance (nourishment).

46 . C

Characterization and Tone

The passage as a whole characterizes Smetacek”s experiments in iron fertilization to promote oceanic cyanobacterial blooms as a demonstration of the potential for large-scale manipulation of the planetary environment (lines 94–96) to remove planet-heating CO 2 from the atmosphere (lines 29–30). Therefore, according to the author, this fertilization is a promising and feasible solution to a global problem .

47 . D

Textual Evidence

As the explanation to question 46 explains, the best evidence for this answer is found in lines 94–96.

48 . C

Data Analysis

Figure 1 depicts a satellite image of the largest recorded natural phytoplankton bloom in February 2012, believed to have been caused by the addition of iron dust blown into the sea around Antarctica by strong offshore winds (from the caption beneath Figure 1 ). Choice (A) cannot be correct, because Smetacek”s experiment took place in 2004, not 2012. Choice (B) cannot be correct, because the figure does not indicate anything about the relationship between algal bloom size and time of year. Choice (D) cannot be correct, because the figure does not contain any information about the relationship between bloom size and distance from the Antarctic ice shelves. The correct answer is (C) because the figure clearly shows a bloom that is well over 5,000 square kilometers (over 100 km long and over 50 km wide) in area, which is more than 30 times larger than Smetacek”s 167 square kilometer (line 53) bloom.

49 . A

Inference

According to the passage, Smetacek”s theory was that iron fertilization of plankton could siphon more and more planet-heating CO 2 from the atmosphere (lines 29–30) and then sequester as much as one billion metric tons of carbon dioxide annually, keeping it deep in the ocean for centuries (lines 34–37) by [dying] and [sinking] to the sea floor, thereby providing natural carbon sequestration (lines 49–50). Therefore, the death of the algal bloom described in lines 61–65 is vindication of his theory that iron fertilization can lead to carbon sequestration .

50 . C

Inference

The passage states that plankton serves as the base layer of the global food chain (line 13) and therefore fertilizing phytoplankton with iron would promote blooms to help sea life thrive all the way up the food chain, even to whale populations (lines 44–46). Therefore, iron fertilization helps the whale population by supporting an important food source for the whales .

51 . D

Interpretation

The second to last paragraph (lines 88–92) discusses a way to motivate further research into iron fertilization (lines 89–90) therefore the route to broader acceptance of the practice (lines 91–92) is a mode of persuasion . Choice (A) is incorrect because, although the research itself probably involves an experimental procedure , the route is not part of the research itself, but rather a means to gain support for that research. Choice (B) is incorrect because, although an appeal to the needs of commerce (line 88) shows an appreciation for economic concerns, the route is not itself an economic difficulty . Choice (C) is incorrect because appealing to the needs of commerce is not an idealisticapproach, but rather a pragmatic one.

52 . A

Tone

The last paragraph is sanguine (hopeful) about the potential for iron fertilization of the oceans to undo a great deal of the damage we have already done (lines 98–99).

Section 2: Writing and Language

1 . C

Idiom

Although gerunds like hiking are often interchangeable with infinitives like to hike (for instance, saying I like hiking is essentially the same as saying I like to hike ), often the conventions of idiom dictate a preference for one form over the other in a particular context. In this case, the phrase getting students to respond is proper idiom, whereas getting students responding is not proper idiom. Choice (B) uses an infinitive form, but the phrase to become responsive inappropriately changes the meaning of the sentence.

2 . A

Subject-Verb Agreement

The subject-verb core of this clause is explaining … is . Notice that this subject and verb agree in number, whereas choices (B) and (D) would introduce subject-verb disagreement. Choice (C) is not idiomatic, so the original phrasing is best.

3 . B

Coordination

The original phrasing misuses the colon, which should be used only to precede an explanatory clause or an explanatory list. Choice (C) is incorrect for the same reason. Choice (D) is incorrect because it commits a number shift: choosing between punishment and incentive is choosing a single motivator, not motivators . Choice (B) avoids these errors and conveys the idea clearly and concisely.

4 . B

Parallelism/Logical Comparison

The phrase better than signals that this sentence is making a comparison, which must be both parallel and logical. In the original phrasing, to reward (infinitive) is being compared with punishment (abstract class noun), and since these are different parts of speech, it violates the law of parallelism. The only choice that provides another abstract class noun is (B) reward . Choice (C) is incorrect because rewarding is a gerund, not a class noun, and choice (D) is incorrect because a reward represents an event-instance, not a class of actions.

5 . B

Modifier error/Idiom

Remember that any sentence must retain its grammatical integrity even when its modifying phrases are “trimmed” away. The phrase if not more so is an interrupting modifier, but when it is removed, the sentence reads … as harmful … than punishment , which is of course not idiomatic. The only choice that avoids this problem is choice (B).

6 . A

Parallelism

This sentence contains the comparative idiom not only A but also B . When we use such idioms, we must make sure that we use the precise phrasing and that the words or phrases that replace A and B are parallel. The original phrasing is both idiomatic and parallel, because both phrases that replace A and B are prepositional phrases. Choice (B) is not parallel, and choices (C) and (D) are neither parallel nor idiomatic.

7 . D

Coherence

The passage as a whole is discussing the use of rewards as a teaching tool, so the underlined sentence is important because it indicates their ineffectiveness in that role.

8 . C

Dangling Participles

In the original phrasing, as well as in choices (B) and (D), the participle interpreting dangles: its subject does not match the subject of the main clause, subjects . Choice (C) does not have this problem, and conveys the idea clearly and concisely.

9 . A

Diction

The original word choice is best. Choice (B) is incorrect because although the results of the study may indicate that changes be made in the classroom, the phrase have profound indications for the classroom is not idiomatic, because indicate is a transitive verb and so requires a direct object. Choice (D) has a similar problem, since the verb instigate is also a transitive verb. Choice (C) is incorrect because improvisations are performances without preparation, which studies cannot do.

10 . B

Transitions

This paragraph is discussing the evidence regarding the ability of rewards to incentivize learning. This particular sentence mentions a possible interpretation of that evidence; therefore, the adverb evidently is the most logical sentence modifier. Choices (A) and (C) are incorrect because they inappropriately indicate a contrast. Choice (D) is incorrect because this point is not the last of a sequence of points.

11 . A

Rhetorical Devices

The final paragraph contains a prescription (strong suggestion) in the last sentence: Teachers must be aware of their students” need to feel independent and in control . It also contains a qualification (a statement that moderates a previous claim) in the statement it would be a mistake to conclude that all rewards are bad . Choice (B) is incorrect because the paragraph contains no quantification (numerical measurement). Choice (C) is incorrect because it provides neither anecdote (illustrative story) nor metaphor (comparison that equates to things that are not literally equivalent). Choice (D) is incorrect because the paragraph contains no irony (reversal of reader expectations) or attempts at humor .

12 . C

Coordination

The original phrasing is illogical because DNA does not serve as a chemical compound , it is a chemical compound. (Although someone can both serve as a nurse and be a nurse, this is because nursing is a service. Molecules do not perform services in the way that human professionals do.) Choice (B) is incorrect because it produces a comma splice, and choice (D) is incorrect because it is needlessly wordy, and because the present progressive form is servingincorrectly implies a current action rather than a general function. Choice (C) avoids these errors and conveys the idea clearly and effectively in the form of an appositive phrase.

13 . A

Punctuation/Pronoun Agreement

Since the sentence uses the em-dash (—) to introduce the list of examples, it must likewise use the em-dash to close this list. Any interrupting phrase must start and end with the same punctuation marks: either commas or em-dashes. Although choice (D) uses the em-dash, it is incorrect because the pronoun itself does not agree in number with the antecedent cell types .

14 . B

Diction

Choice (B) includes an illogical use of the preposition of .

15 . A

Transitions

The original phrasing is best. Choice (B) is incorrect because the sentence does not indicate any logical consequence. Choice (C) is incorrect because the sentence does not indicate any contrast. Choice (D) is incorrect because the sentence does not indicate any irony.

16 . B

Diction

This sentence describes the ability of doctors to use biological information to create “personalized medicine .” One who is particularly skilled is adept . Choice (A) is incorrect because apropos means appropriate to a given situation. Choice (B) is incorrect because liable means likely or legally responsible . Choice (D) is incorrect because the phrase essential at is neither logical nor idiomatic.

17 . D

Idiom

This sentence discusses using stem cells to repair damaged organs. The most concise and idiomatic way to express this functional relationship is with the infinitive to help . Choices (A), (B), and (C) all use nonidiomatic phrases.

18 . C

Logical Cohesiveness/Transitions

This paragraph discusses the task of translating vast quantities of chemical information into digital form , and indicates that good progress has been made, thanks to progress in specialized hardware and software . Therefore, the best introductory sentence is (C), which focuses on progress in the computer sciences .

19 . D

Subject-Verb Agreement/Idiom

The original phrasing is incorrect because the phrase our ability in translating is not idiomatic. Choice (B) is incorrect because the verb depend does not agree in number with the subject success . Choice (C) is incorrect because the phrase the ability of our translating is illogical. Choice (D) avoids these errors and expresses the idea clearly and concisely.

20 . B

Idiom/Voice/Mood

The original phrasing is not idiomatic. Choice (C) is incorrect because its use of the active voice is illogical. Choice (D) is also not idiomatic. Only choice (B) conveys the proper mood (necessity) idiomatically and concisely.

21 . B

Data Analysis

Choice (A) is incorrect because the graph does not indicate anything about the reason for the declining costs in gene sequencing. Choice (B) is correct because in 2014 the actual cost per genome decoded was approximately $8,000, whereas Moore”s Law predicted a cost of over $1,000,000 in 2014. This actual cost is less than 1% of the predicted cost. Choice (C) is incorrect because the cost to decode a single genome has been under $10,000 ever since 2012. Choice (D) is incorrect because the graph makes no direct comparison of the cost of genome sequencing to that of other information-based technologies.

22 . B

Dangling Participles/Verb Tense

The original phrasing is incorrect because the participle using dangles: it does not share its subject with the main clause. Choice (C) is incorrect for the same reason. Choice (D) is incorrect because, although it corrects the dangling participle, the present perfect form of the verb, have discovered , is illogical.

23 . D

Diction

The original word choice is illogical since qualities are incapable of deciding anything. Choice (B) and (C) are incorrect for similar reasons: anything that arranges or regulates must have a mind and intention, which qualities lack. Choice (D) is the only reasonable choice, since qualities can determine (that is, play a deciding role in an outcome ) whether or not something is art.

24 . A

Verb Aspect

The original phrasing is best. The present perfect form have developed is appropriate because the status of the notions is the consequence of how they developed over the centuries . Recall that the perfect (or consequential ) aspectis used to indicate a status-as-consequence (see Chapter 4 , Lesson 23).

25 . D

Verb Tense

This sentence indicates a historical fact, so the simple present tense is best.

26 . C

Redundancy

The original phrasing is redundant, since being conspicuous is the same thing as standing out . Choice (C) is the only one that avoids the redundancy.

27 . B

Logical Cohesiveness

The paragraph is about the fact that some things in our environment stand out obviously as “art.” It would be reasonable, then, to follow this point with an explanation of why a particular object, such as a chair, qualifies as art.

28 . B

Coordination

The original phrasing is incorrect because the clause that follows the semicolon is not independent, and the transitive verb said lacks a logical direct object. Choice (C) is incorrect because it creates a comma splice. Choice (D) is incorrect because the clause that follows the semicolon is not independent. Choice (B) avoids these problems.

29 . C

Punctuation/Coordination/Parallelism

The original phrasing is illogical because the adverb when incorrectly implies that whether or not an object qualifies for art is a time-specific event, rather than a general criterion. Choice (B) is incorrect because it creates a non-parallel list: compels … inspiring … beautiful . Choice (D) is incorrect because it”s = it is .

30 . D

Coordination/Punctuation

The sentence is a compound sentence joining a dependent clause If you have … and an independent clause you have had … . The original phrasing is incorrect because a colon should be used only to join two independent clauses in which the second explains the first. Choice (B) is incorrect because a semicolon should only be used to join independent clauses. Choice (C) is incorrect because an em dash should only be used to separate an independent clause from an interrupting modifier. Choice (D) is best because a simple comma is most effective at joining a dependent clause and an independent one.

31 . A

Verb Form

The original phrasing is best. Choice (B) is incorrect because the pronoun they disagrees in number with its antecedent skill, beauty, or expression . (The conjunction or implies that only one of these serves as the subject; therefore, it is singular.) Choice (C) is incorrect because it does not coordinate with the phrase that gives . Choice (D) is incorrect because it has the disagreement problem of (B) as well as the coordination problem of (C).

32 . A

Coordination

The original phrasing is best. Choice (B) is illogical. Choice (C) creates subject-verb disagreement. Choice (D) is awkward and creates an unidiomatic phrase: to make them as art .

33 . D

Logical Cohesiveness

This paragraph introduces and defines the “institutional theory of art,” presumably because the author regards it as an interesting theory of aesthetics. Choice (D) is best because it poses an intriguing question that follows directly from that definition. Choices (A), (B), and (C), while true statements, are irrelevant to a discussion of this theory.

34 . D

Comparative Idiom/Logical Comparison

This sentence uses the comparative idiom as strong as , so we must check that the comparison is idiomatic, parallel, and logical. The original phrasing and the phrasing in choice (B) are incorrect because they create illogical comparisons: they compare people to an impact . Choice (C) is incorrect because it is not idiomatic. Choice (D) provides an idiomatic, parallel, and logical comparison.

35 . A

Coordination/Idiom

The original phrasing of the participial phrase is best: notice that the subject of the past participle born is also the subject of the main clause: Chaplin . Choices (B) and (D) are incorrect because they form comma splices. Choice (C) is incorrect because the present participle being implies that Chaplin was born at the same time that he crossed the Atlantic, which is illogical.

36 . C

Logical Cohesiveness

Although this sentence introduces a true and interesting fact, it is inappropriate to this paragraph, which is about Chaplin”s impact on the film industry.

37 . C

Coordination

The sentence begins with two adjectival phrases that modify the Tramp . Therefore, these modifiers will dangle unless the subject of the main clause is the Tramp . The only choice that avoids this dangling is (C).

38 . D

Parallelism

The original phrasing includes a list that is not parallel: writer, director, and editing . The only choice that does not violate the Law of Parallelism is (D).

39 . A

Parallelism

The original phrasing is best because it creates the parallel phrasing taking on … writing … and thus allowing .

40 . C

Sentence Fragments/Comparative Idiom

The original phrasing is incorrect because it creates a sentence fragment. Choices (B) and (D) commit the same error. Only choice (D) forms a sentence with an independent clause. Note also that is correctly applies the comparative idiom not only A but also B .

41 . A

Dangling Modifiers

The original phrasing is best because it coordinates with the appositive phrase that begins the sentence. Choices (B) and (C) allow this appositive to dangle. Choice (D) is incorrect because the phrase became targeted for is not idiomatic.

42 . C

Logical Coordination/Idiom

This sentence describes the reason that Chaplin drew the ire of J. Edgar Hoover . Choice (C) provides the most logical phrase to coordinate this state of being and its cause: because of . The prepositional phrases in the original phrasing and in choices (B) and (D) do not convey this logical relationship.

43 . B

Pronoun Agreement

The original phrasing is incorrect because the definite pronoun they disagrees in number with the antecedent government . Choice (C) is incorrect because the subjunctive form would have been incorrectly implies that this clause in counterfactual. Choice (D) is incorrect because propaganda , although it sounds plural, is singular.

44 . B

Diction

In this context, eradicated does not work because it means destroy completely, as a scourge , which does not accurately modify a visa. Choice (B), revoked (officially invalidated) works nicely. Choice (C) is illogical because the visa is not disallowed entry into a group, as excluded would imply. Choice (D) is illogical because abolish more properly describes the formal termination of an institution, practice, or system.

Section 3: Math (No Calculator)

1 . D

Algebra (solving equations) EASY

8x + 6 = 6m

To solve in one step, just divide both sides by 2:

4x + 3 = 3m

2 . A

Algebra (linear systems) EASY

To determine which ordered pair is a solution to the system, just “plug in” the values for x and y and choose the one that satisfies both equations. Notice that x = 2 and

y = 3 is a solution because 3(2) + 4(3) = 18, and .

3 . C

Algebra (algebraic expressions) EASY

Distribute:

Simplify:

4 . A

Advanced Mathematics (polynomials) EASY

There are several ways to approach this question. Perhaps the simplest is to use the Factor Theorem: If xc is a factor of a polynomial, then x = c is a zero of that polynomial. Therefore, if x − 3 is a factor of our polynomial, x = 3 must be a zero:

x 2 + kx + 12 = (3)2 + 3k + 12 = 0

Simplify:

9 + 3k + 12 = 0

Subtract 21:

3k = –21

Divide by 3:

k = –7

Alternately, you might try to find the other factor of the quadratic. Since the constant term in the quadratic is 12, the constant term in the other binomial factor must be 12 ÷ –3 = −4.

(x − 3)(x − 4) = x 2 + kx + 12

FOIL:

x 2 − 7x + 12 = x 2 + kx + 12

Subtract x and 12:

–7x = kx

Divide by x :

–7 = k

5 . C

Additional Topics (circles and triangles) MEDIUM

Since PA and PB are both radii of the circle, they are congruent, and so triangle APB is isosceles. By the Isosceles Triangle Theorem, then, angle A must also be 20°. From here, you might simply notice that the angle we”re looking for, CPB , is the external angle to this triangle, and so it has a measure equal to the sum of the two remote interior angles: 20° + 20° = 40°. Alternately, you could notice that angle APB must have a measure of 140° (since all angles in a triangle have a sum of 180°), and since AC is a straight line, angle CPB = 180° − 140° = 40°.

6 . B

Advanced Mathematics (sequences) MEDIUM

Let”s choose a value, like b = 2, for our positive constant. This gives us an expression of 2n + 4 for the n th term of the sequence. Substituting n = 1, n = 2, n = 3, etc. gives us a sequence of 6, 8, 10, 12, 14, and so on. Choice (A) is clearly incorrect, because the first term of this sequence is not 2. Choice (C) is also incorrect because the average of the first three terms is (6 + 8 + 10)/3 = 8, not 2. Choice (D) is also incorrect because the ratio of the second term to the first is 8/6 = 4/3. Only choice (B), the difference between the fourth term and the third term, 12 – 10, gives us a value of 2.

7 . A

Advanced Mathematics (radical and exponential equations) MEDIUM

For this question, we need to know two Laws of Exponentials from Chapter 9 : Law #8 and Law #9. First, we use Law #9 to translate the radicals into exponents.

Given equation:

Apply Law of Exponentials #9:

Apply Law of Exponentials #9 again:

Apply Law of Exponentials #8:

Raise to the power:

Apply Law of Exponentials #8 again:

8 . B

Algebra (word problems) MEDIUM

Perhaps the most straightforward way to approach this question is to regard it as a conversion from a given area of lawn (in square feet) to cost (in dollars).

Area of rectangular lawn:

A = bh = ab square feet

Convert using given conversion factors:

Make sure to check this calculation by noticing that all units “cancel” as common factors, except for the unit we want, dollars, which remains in the numerator.

9 . C

Advanced Mathematics (rational inequalities) MEDIUM

Original inequality:

Multiply by 3m (since m > 0, we don”t “flip” the inequality):

15 ≤ 2m

Divide by 2:

7.5 ≤ m

Therefore, the least possible value of m is 7.5.

10 . A

Algebra (linear functions) MEDIUM-HARD

Given function:

f (x ) = 3x + n

Substitute f (2) = 0:

f (2) = 3(2) + n = 0

Simplify:

6 + n = 0

Subtract 6:

n = –6

Therefore, the function is f (x ) = 3x − 6.

Evaluate f (n ):

f (n ) = f (–6) = 3(–6) – 6 = –18 – 6 = –24

11 . A

Algebra (absolute values) MEDIUM-HARD

First, we should notice that each choice can be interpreted as a distance between two points on the number line.

(A) |sv | = the distance between s and v

(B) |st | = the distance between s and t

(C) |s + v | = |s − (–v )| = the distance between s and –v

(D) |u + v | = |u − (–v )| = the distance between u and –v

Thinking this way gives us a very straightforward way to solve the problem without doing any calculation. First we need to locate –v on the number line by just reflecting v over the origin at 0. (Recall that multiplication by –1 is equivalent to reflecting a point on the number line over the origin at 0.) This makes it easy to see the distances the problem is asking us to compare:

Clearly, the greatest of these distances is (A).

12 . B

Special Topics (trigonometry) MEDIUM-HARD

In order to solve this without a calculator, we need to know how to analyze this problem in terms of the unit circle. First, let”s solve for cos x :

4 cos x = 1

Divide by 4:

What does the mean in terms of the unit circle? Recall from Chapter 10 , Lesson 9, that the cosine of any angle corresponds to the x -coordinate of the corresponding point for that angle on the unit circle:

Notice that there are exactly two points on the unit circle that have an x -coordinate of 1/4. Now let”s think about the angle. We are told that x goes from 0 to 3π. Remember that a full trip around the circle is 2π radians; therefore, a journey from x = 0 to x = 3π is 1.5 trips around the circle counterclockwise starting from the positive x -axis. If you trace with your finger 1.5 times around the circle starting from the point (1, 0), you”ll hit our “points of interest” exactly three times.

13 . C

Additional Topics (complex numbers) HARD

To solve this without a calculator, you must be able to evaluate a few low powers of i . Recall from Chapter 10 , Lesson 10, that i 0 = 1, i 1 = i , i 2 = –1, i 3 = –i , and i 4 = 1. Therefore i 3 + i = –i + i = 0. Now, it”s just a matter of finding the choice that does NOT equal 0.

(A) (2i )2 + 4 = –4 + 4 = 0

(B) 2 – 2i 4 = 2 – 2 = 0

(C) 2i 2 – 2 = –2 – 2 = –4

(D) i 4 – 1 = 1 – 1 = 0

Therefore, the correct answer is (C).

14 . D

Algebra (graphs of quadratic equations) HARD

Recall from Chapter 9 , Lesson 6, that any equation in the form y = a (xh )2 + k has a vertex at (h , k ) and is open up if a > 0 and down if a < 0. In the equation y = –(x + m )2 + m ; therefore, the vertex is (–m, m ), and a = –1. Since m > 1, this means that the vertex of the parabola has a negative x -coordinate and a positive y -coordinate, which means the vertex is in quadrant II. And since a < 0, the parabola is open down. The only graph among the choices that is an open down parabola with a vertex in the second quadrant is the graph in choice (D).

15 . B

Advanced Mathematics (linear and nonlinear systems) HARD

First, notice that the question is only asking us to find values of x , so it”s a good idea to substitute in order to eliminate y from the system.

x − 3y = –2

Substitute :

Multiply by x and simplify:

x 2 − 15 = –2x

Add 2x :

x 2 + 2x − 15 = 0

Factor using Sum-Product Method:

(x − 3)(x + 5) = 0

Therefore, the values of x that satisfy the original system also satisfy the equation

(x − 3)(x + 5) = 0.

16 . 4.5 or 9/2

Algebra (linear equations) EASY

Original equation:

Substitute b = 4:

Simplify:

Subtract 2:

Multiply by :

17 . 7

Advanced Mathematics (rational equations) EASY

Given inequality:

Multiply by 2x :

12 + 1 < 2x

Simplify:

13 < 2x

Divide by 2:

6.5 < x

The smallest integer that is greater than 6.5 is 7.

18 . 8.4 or 42/5

Additional Topics (perimeters and area) MEDIUM-HARD

First, drawing a line as shown in the diagram shows that the figure is composed of two rectangles, but the height of the smaller one is unknown. Let”s call it x . The area of the larger rectangle is (3)(1) = 3, and the area of the smaller rectangle is (1)(x ) = x . Clearly, the area of the figure must be the sum of these two areas

Subtract 3:

Therefore, the perimeter of the figure is just the sum of the lengths of its sides. If we travel around the figure clockwise from the leftmost side, we get a perimeter of

.

19 . 5 or 7

Algebra (rational equations) MEDIUM-HARD

Original equation:

Multiply by 4(x + 1)(x – 1):

We do this because 4(x + 1)(x – 1) is the least common multiple of the denominators, so multiplying both sides by this will eliminate the denominators and simplify the equation.

Cancel common factors:

24(x – 1) – 12(x + 1) = (x + 1)(x – 1)

Distribute and FOIL:

(24x – 24) – (12x + 12) = x 2 – 1

Collect like terms:

12x – 36 = x 2 – 1

Subtract 12x and add 36:

0 = x 2 – 12x + 35

Factor:

0 = (x – 5)(x – 7)

Solve using Zero Product Property:

x = 5 or 7

20 . 8/13 or .615

Special Topics (trigonometry) HARD

Find AD with Pythagorean Theorem:

(AD )2 + 42 = 52

Simplify:

(AD )2 + 16 = 25

Subtract 16:

(AD )2 = 9

Take square root:

AD = 3

Or, even better, just notice that triangle ADB is a 3-4-5 right triangle.

Use triangle area formula to find AC :

Simplify:

2(AC ) = 19

Divide by 2:

Find DC :

Find tan θ :

Section 4: Math (Calculator)

1 . D

Algebra (solving equations) EASY

Multiply by 20:

n = 0.8(20) = 16

2 . A

Data Analysis (central tendency) EASY

The median of three numbers is the one in the middle when they are listed in order. If two of the numbers are 10 and 12, with 12 as the median, then the third number must be greater than or equal to 12, otherwise 12 would not be in the middle. Of the choices, only (A) 8 is not greater than or equal to 12.

3 . C

Algebra/Data Analysis (expressing relationships) EASY

The first ordered pair, x = 0 and y = 2, does not satisfy the equations in (A), (B), or (D), so those choices can be eliminated. You should also confirm that the equation in (C), y = 2x + 2, is satisfied by all four ordered pairs.

4 . B

Data Analysis (central tendency) EASY

Let”s call the 3 positive integers a , b , and c . If the average of these numbers is m , then

Multiply by 3:

a + b + c = 3m

New average when 24 is included in the set:

Substitute a + b + c = 3m :

5 . C

Algebra (rational equations) EASY

Multiply by x :

6 + 3x = –x

Subtract 3x :

6 = –4x

Divide by –4:

6 . A

Algebra (representing quantities) EASY

The cost for a month”s worth of energy is the cost per kilowatt-hour times the total number of kilowatt-hours used: ($0.15/kWh)(30 kWh). The total monthly charge, P , must also include the service fee: P = 0.15(30) + 16.

7 . A

Advanced Mathematics (triangle trigonometry) EASY

Remember the definitions of the basic trigonometric functions: SOH CAH TOA. Since the “side of interest” (h ) is the opposite side to the given angle (42°), and since we know the length of the hypotenuse (1,200), we should use SOH.

Plug in the values:

Substitute sin 42° = 0.669:

Multiply by 1,002:

(1,200)(0.669) = 802.8 = h

8 . C

Special Topics (polygons) EASY

The sum of the measures if the interior angles of a triangle is 180°, therefore mBED + 90° + 50° = 180°, and so mBED = 40°. Since ∠AEC is vertical to ∠BED , it must also have a measure of 40°, and so

40 + x + x = 180

Simplify:

40 + 2x = 180

Subtract 40:

2x = 140

Divide by 2:

x = 70

9 . C

Data Analysis (histogram) MEDIUM

According to the histogram, 7 students received 15 credits, 1 student received 16 credits, and 1 student received 18 credits, for a total of 9 students who received 15 or more credits. This is 9/24 of the total, or 37.5%

10 . D

Data Analysis (histogram/central tendency) MEDIUM

Data set: 8, 9, 12, 12, 12, 12, 12, 12, 12, 12, 12, 12, 13, 13, 13, 15, 15, 15, 15, 15, 15, 15, 16, 18

The median of a set of numbers is the “middle” number of the set when the numbers are listed in order. If the set contains an odd number of numbers, the median is the middle number, but if the set contains an even number of numbers, it is the average of the two middle numbers. Since this set contains 24 numbers, the median is the average or the 12th and the 13th numbers. The 12th number in the set is 12, and the 13th number in the set is 13, the median is 12.5.

11 . D

Algebra (absolute values) EASY

The absolute distance from a to b is |ab | and the absolute distance from –2 to 6 is |–2 − 6| = 8. Therefore, |ab | > 8.

12 . D

Special Topics (circles) MEDIUM

Since ABCD is a rectangle, we can find the length of its diagonal using the Pythagorean Theorem: 102 + 242 = d 2 . Even better, we can notice that the two legs are in a 5:12 ratio, and therefore triangle BCD is a 5-12-13 triangle. In either case, we find that DB = 26. Since DB is also a diameter of the circle, the radius of the circle is 26/2 = 13, and therefore, the area of the circle is πr 2 = π(13)2 = 169π.

13 . B

Problem Solving/Data Analysis (enumeration of data) MEDIUM

If Niko is the 8th oldest person in the class, then there are 7 students older than he is. If he is the 12th youngest person, then there are 11 students younger than he is. Therefore, there are 18 students in addition to him, for a total of 19 students.

14 . D

Additional Topics (complex numbers) MEDIUM

(2 – i )(3 – 2i )

FOIL:

6 – 4i – 3i + 2i 2

Substitute i 2 = –1:

6 – 4i – 3i + 2(–1)

Combine like terms:

4 – 7i

15 . B

Advanced Mathematics (exponentials) MEDIUM

f (3) = (32 )–2b = 3

Exponential Law #8

(from Chapter 9 , Lesson 9):

3–4b = 31

Exponential Law #10

(from Chapter 9 , Lesson 9)

–4b = 1

Divide by –4:

16 . A

Data Analysis (probability) MEDIUM

Since the sum of 55 and 35 is 90, which is 10 greater than 80, there must be at least 10 in the overlap between the two sets. Statement (B) is not necessarily true, because it is possible that all 35 students taking AP courses are also varsity athletes, which is more than half of 55. Statement (C) is not true because 80 – 55 = 25 students do not play varsity sports, and 80 – 35 = 45 students do not take at least one AP course. Statement (D) is not necessarily true, because 35 students take at least one AP course and 25 students do not play a varsity sport, and this sum, 35 + 25 = 60, is less than the total number of students, so it is possible that there is no overlap between these two sets.

17 . A

Algebra (slopes) MEDIUM

If AB = BC , then triangle ABC is isosceles and therefore the two base angles are congruent and the triangle has a vertical axis of symmetry at the line x = 3. This implies that the slopes of lines and are opposites. We can calculate the slope of from its endpoints:

Therefore, the slope of is 3, and so mn = (3)(–3) = –9.

18 . D

Advanced Mathematics (functions)

MEDIUM-HARD

To answer this question, we must evaluate each of the three functions for an input of ½:

Therefore, .

19 . A

Data Analysis (graphing data) MEDIUM-HARD

If y varies inversely as the square of x , then the variables are related by the equation , where k is a positive constant. The graph of such an equation in the xy -plane looks like this:

This most closely resembles the scatterplot in choice (A).

20 . C

Algebra (expressing relationships) MEDIUM-HARD

The portion of the birds that are finches is just the number of finches divided by the total number of birds. Since there are already 30 finches, adding n finches makes 30 + n finches. Since there are already 12 + 30 + 18 = 60 total birds, adding n finches makes 60 + n total birds.

Since .

21 . B

Advanced Mathematics (functions) MEDIUM-HARD

f (x ) = x 2 – 1

Substitute :

Simplify:

Get common denominator:

Subtract fractions:

Factor numerator:

22 . D

Advanced Mathematics (quadratics)

MEDIUM-HARD

Since k can be any number greater than 0, let”s pick k = 1 for convenience. If x = 3, then y = (1)(3)2 = 9, and if x = 12, then y = (1)(12)2 = 144. In this case, both statement (B) and statement (D) are true, since 9 + 135 = 144 and 9(16) = 144; therefore, we can eliminate choices (A) and (C). Now let”s choose k = 2. If x = 3, then y = (2)(3)2 = 18, and if x = 12, then y = 2(12)2 = 288. Since 18 + 135 ≠ 288, but 18(16) = 288, the correct answer is (D).

Notice, also, that since y varies directly as the square of x , then when x is multiplied by n , y is multiplied by n 2 . Since x is being multiplied by 4 (to go from 3 to 12), then y must be multiplied by 42 = 16.

23 . B

Advanced Mathematics (analyzing quadratics) HARD

One way to tackle this question is simply to simplify the expression for f (7), and then see which choice gives the same expression.

f (7) = k (7 + 6)(7 − 1) = k (13)(6) = 78k

Evaluate (A):

f (–78) = k (–78 + 6)(–78 − 1) = k (–72)(–79) = 5,688k

Evaluate (B):

f (–12) = k (–12 + 6)(–12 − 1) = k (–6)(–13) = 78k

Evaluate (C):

f (–2) = k (–2 + 6)(–2 − 1) = k (4)(–3) = –12k

Evaluate (D):

f (78) = k (78 + 6)(78 − 1) = k (84)(77) = 6,468k

This shows that f (–12) is equal to f (7). Alternately, you might just make a quick sketch of the parabola and take advantage of the symmetry:

24 . C

Problem Solving (percentages) MEDIUM-HARD

Let p = the initial price per share of the stock. After the first year, its price increased by 20%, so its price was (1.20)p . After the second year, this price declined 25%, so its price was (0.75)(1.20)p . After the second year, this price increased by 10% so its price was (1.10)(0.75)(1.20) p = 0.99p , which means that overall the price decreased by 1%.

25 . D

Algebra (exponentials) MEDIUM-HARD

Expression to be evaluated:

Substitute w = y 6 :

Simplify with Exponential Law #6 (from Chapter 9 , Lesson 9):

y 3

Substitute y = x 2 :

(x 2 )3

Simplify with Exponential Law #8 (from Chapter 9 , Lesson 9):

x 6

26 . D

Data Analysis (tables) MEDIUM-HARD

Although we don”t need to fill in the entire table, it”s interesting to note that it has a “diagonal symmetry” when it is completed. Just following the rule and moving systematically toward x reveals that it is 56 + 56 = 112.

27 . C

Advanced Mathematics (quadratics) HARD

We can find the value of c by just substituting x = –1 into the equation.

Given equation:

3x 2 = 4x + c

Substitute x = –1:

3(–1)2 = 4(–1) + c

Simplify:

3 = –4 + c

Add 4:

7 = c

Therefore, the equation is:

3x 2 = 4x + 7

Subtract 4x and 7:

3x 2 – 4x – 7 = 0

Factor using Sum-Product Method:

(x + 1)(3x – 7) = 0

(Notice that the factor (x + 1) corresponds to the fact that x = –1 is a solution to the quadratic.)

Use Zero Product Property to find other solution:

3x − 7 = 0

Add 7:

3x = 7

Divide by 3:

x = 7/3

28 . B

Special Topics (arcs) HARD

First, we should make sure we mark up the diagram with the measurements we know: OD = 3 and DB = 5. This means that the radius of the small circle is 3 and the radius of the large circle is 8. Notice that sectors AOB and CODshare a central angle, and therefore are similar. So the measures of arc CD and arc AB are

in a ratio of 3:8.

Cross multiply:

Divide by 8:

29 . C

Data Analysis (graphs) MEDIUM

Since there are only four data points, it”s not hard to list the February-May ordered pairs. Notice that the February axis is vertical, and the May axis is horizontal, so the typical x-y relationship is reversed:

Class A: February: 60, May: 60

Class B: February: 80, May: 70

Class C: February: 50, May: 70

Class D: February: 90, May: 90

Notice that the only class that saw an increase in percent participation is Class C.

30 . C

Data Analysis (graphs) HARD

We just need to tally the number of students who participated from each class.

Class A: 60% of 20 students = 12 students

Class B: 70% of 30 students = 21 students

Class C: 70% of 20 students = 14 students

Class D: 90% of 30 students = 27 students

12 + 21 + 14 + 27 = 74 students

31 . 0.8 or 4/5

Algebra (radical equations) EASY

Given equation:

Subtract 4:

Therefore, .

32 . 8

Advance Mathematics (quadratics) EASY

Given equation:

y = 3x 2kx – 35

Substitute x = 5 and y = 0:

0 = 3(5)2k (5) – 35

Simplify:

0 = 75 – 5k – 35

Simplify:

0 = 40 – 5k

Add 5k :

5k = 40

Divide by 5:

k = 8

33 . .96

Additional Topics (circles/triangles) MEDIUM-HARD

When looking for the area of the triangle, remember that there are two basic methods: the direct method and the indirect method. With the direct method, we simply plug the base and height measurements into the formula , and with the indirect method, we find the area as the sum or difference of other areas. In this case, since we know the lengths of one of the sides, the direct method is probably best. But we will need to find the height as well.

Area of the circle is π:

πr 2 = π

Divide by π:

r 2 = 1

Take square root:

r = 1

Now let”s mark up the diagram with this information. Since the radius of the circle is 1, the diameter FH has a length of 2. Now we can use the Pythagorean Theorem to find the length of GH , which is the height of the triangle if FGis taken as the base.

(1.6)2 + (GH )2 = 22

Simplify:

2.56 + (GH )2 = 4

Subtract 2.56:

(GH )2 = 1.44

Take square root:

GH = 1.2

(Notice that this is in fact a 3-4-5 triangle: if we multiply 3-4-5 by 0.4, we get 1.2-1.6-2.)

Plug into area formula:

34 . 4.29 ≤ x ≤ 4.79

Algebra (solving inequalities) HARD

Multiply by –3 and “flip” inequalities:

Add 3:

Divide to get decimal form:

4.80 > 6t > 4.2857

Therefore any decimal value between 4.29 and 4.79, inclusive, is acceptable.

35 . .84

Advanced Mathematics (trigonometry) MEDIUM-HARD

You may find it helpful to make a quick sketch of the unit circle on the xy -plane, as we discussed in Chapter 10 . Subtracting π radians (or 180°) from an angle just means rotating the terminal ray of that angle 180° clockwise. It should be clear, then, that in the xy -plane, the angle with measure x – π points in the opposite direction of the angle with measure x . Recall that the cosine of an angle is just the x -coordinate of the point where its terminal ray intersects the unit circle. Since cos (x – π) = 0.4 (that is, its cosine is positive), its terminal ray must be in a quadrant where the x -coordinates are positive: either quadrant I or quadrant IV. Let”s just put it in quadrant I. This means that the angle with measure x is in quadrant III, and so it has the opposite cosine:

cos x = –0.4

Recall Pythagorean Identity from Chapter 10 :

sin2 x + cos2 x = 1

Substitute cos x = 0.4:

sin2 x + (–0.4)2 = 1

Simplify:

sin2 x + 0.16 = 1

Subtract 0.16:

sin2 x = 0.84

36 . 12

Problem Solving/Data Analysis (word problem) MEDIUM-HARD

This one is a bit trickier than it looks. We have 10 pounds of grain and have used it to feed 20 chickens. Since one pound of grain feeds 5 chickens, proportionally we need 4 pounds of grain to feed 20 chickens. This leaves us 10 − 4 = 6 pounds of grain to feed the pigs. Since 1 pound of grain can feed 2 pigs, proportionally 6 pounds of grain can feed 12 pigs.

37 . 371

Problem Solving (extended thinking) HARD

The total revenue from the tickets sold is $60(50) + $50(60) + $40x + $35y + $30(100). If the total revenue was $15,000, then 3,000 + 3,000 + 40x + 35x + 3,000 = 15,000

Subtract 9,000:

40x + 35y = 6,000

Divide by 5:

8x + 7y = 1,200

If 15 more tickets were sold in the second mezzanine than the first mezzanine:

y = x + 15

Substitute y = x + 15 in previous equation:

8x + 7(x + 15) = 1,200

Distribute:

8x + 7x + 105 = 1,200

Subtract 105:

15x = 1,095

Divide by 15:

x = 73

Substitute to find y :

y = x + 15 = 73 + 15 = 88

Therefore, the total number of tickets sold is 50 + 60 + 73 + 88 + 100 = 371.

38 . 25

Problem Solving (extended thinking) HARD

The mathematical model has embedded in

it the predicted revenue per section: np = revenue per section = $2,800. Notice that this prediction is $200 less than the actual average revenue per section of $3,000, so clearly the model underestimated the number of tickets sold per section.

If we want to analyze this situation in detail, we can compare the predicted tickets sold to the actual tickets sold by adding a new column to the table entitled “predicted sold,” which we can fill in using the calculations from our model. Also, it might be helpful to also add columns for “total revenue” for each situation.

You might notice that the predicted number of tickets sold in the Front Orchestra and the Third Mezzanine are fractions, which seems strange. (Of course we can”t sell a fraction of a ticket!) But even if we round these predictions to the nearest whole numbers, 47 and 93, the total number of tickets is the same: 346, which underestimates the number of tickets sold by 25.

Section 5: Essay

Sample Response

Reading Score: 8 out of 8

Analysis Score: 8 out of 8

Writing Score: 8 out of 8

As Ellis Parker Butler sees it, American English spelling at the turn of the 20th century is a disaster. It defies logic and common sense, and it even destroys reason itself. Or so it seems. In fact, Butler”s essay is not so much about American English spelling rules as it is about American intellectual culture at the turn of the 20th century: it is inclined toward the petty and the self-important. In his essay, Butler builds his argument with humor and charm, and proposes tongue-in-cheek alternatives to traditional standards of, and more recent revisions to, American English spelling. In so doing he pokes ample fun at how Americans think and behave, delving into commentary on politics, technology, and cultural expectations. He uses metaphor, personification, anecdote, and sharp irony to skewer not only the American elite but also himself.

Butler begins his essay with mock-anger at “the spelling profession.” He claims it is “the reason America so long lagged behind Europe in the production of genius,” noting that, while Shakespeare (a Brit) was clearly a genius, Noah Webster (the American lexicographer) was less so. He mounts his high horse with indignant braggadocio: “… self-made authors like me, who never can spell but who simply blaze with genius, will be hooted out of the magazines to make room for a stupid sort of literature that is spelled correctly.” Butler”s juxtaposition of overconfidence and incompetence (at least in the arena of spelling) establish his wry and ironic tone, which he maintains consistently throughout the essay.

Butler then lashes the “spelling reformers” who want “this new phonetic spelling brought into use” for doing what “we Americans do not like,” namely, advocating for sudden change. He illustrates this cultural foible with the example of automobiles, “sixty horsepower snorters” that smell “like an eighteenth-century literary debate” that became acceptable because they were introduced gradually. In fact, Butler says, “we take to being killed by the automobile quite naturally now.” At this point, the reader is probably wondering if this essay is really about spelling after all.

Then he turns his gaze to the problem at hand: silent letters. But to Butler, unlike the “spelling reformers,” the problem is not practical but moral: we despise silent letters in our words because “we want no idle class in America, whether tramp, aristocrat, or silent letter.” His solution? To “set the idle letters to work.” Butler then announces that he, perhaps as a lone insurgent, will commence pronouncing them: “although” will now be a three-syllable word with an audible “ugh” at the end.

At this point, the reader probably recognizes Butler”s shift in topic to be a feint: in fact, the object of his satire is not spelling at all, but those who are obsessed with the idea of reform. The preposterousness of his theory is intended to reflect the preposterousness of those who spend so much time and mental energy on silly rules. His satire even extends to the peculiarly American obsession with committee meetings and reform projects: “I propose to have some millionaire endow my plan, and Serena (Butler”s wife) and I will then form a society for the reforming of English pronunciation.”

The greatest disadvantage of Butler”s plan, he admits, is that it is too commonsensical: it”s advantage is “so manifest that I am afraid it will never be adopted.” Here, we readers can”t help but wonder if Butler is making a wry commentary on bureaucratic incompetence in government and business.

Butler concludes by considering a “Plan B” offered by his wife, Serena: boycott words with silent letters. To Butler, this plan is less intellectual, but “more American”: “Why should people write “although” when they can write “notwithstanding that,” and not have a silent letter in it?” By calling Serena”s plan “more American,” he seems to be criticizing the fickleness of the American consumer, who will change habits sometimes for nonsensical reasons.

Butler uses ironic humor to skewer the American habit of arguing over silly things. By using himself as a foil, together with skillful use of metaphor, anecdote, and personification (““Although” does all the work, and “ugh” sits on the fence and whittles”), Butler encourages us to laugh at ourselves, and perhaps then move on to more serious matters.

Scoring

Reading—8 (both readers gave it a score of 4)

This response demonstrates thorough comprehension of Butler”s essay through skillful use of summary, paraphrases, and direct quotations. The author summarizes Butler”s central purpose and main idea (Butler”s essay is not so much about American English spelling rules as it is about American intellectual culture at the turn of the 20th century: it is inclined toward the petty and the self-important ) and presents many details from the text, including abundant direct quotations. Each quotation is accompanied by insightful commentary that demonstrates that this author has thoroughly comprehended Butler”s central idea, as well as his skillful use of devices like irony, metaphor, and personification to support that claim.

Analysis—8 (both readers gave it a score of 4)

This response presents an insightful analysis of Butler”s essay and demonstrates a sophisticated understanding of the analytical task. This author has identified Butler”s primary modes of expression (metaphor, personification, anecdote, and sharp irony ) and has uncovered the core message of the essay by exploring Butler”s use of these devices (he pokes ample fun at how Americans think and behave, delving into commentary on politics, technology, and cultural expectations ). For example, the author identifies the devices that establish the essay”s overall tone (Butler”s juxtaposition of overconfidence and incompetence (at least in the arena of spelling) establish his wry and ironic tone, which he maintains consistently throughout the essay ) as well as elements that establish layers of meaning (we readers can”t help but wonder if Butler is making a wry commentary on bureaucratic incompetence in government and business ) and shift focus to more subtle ideas (the reader probably recognizes Butler”s shift in topic to be a feint: in fact, the object of his satire is not spelling at all, but those who are obsessed with the idea of reform ). This response explores Butler”s essay systematically, from introduction to conclusion, with thoughtful commentary throughout.

Writing—8 (both readers gave it a score of 4)

This response shows a masterful use of language and sentence structure to establish a clear and insightful central claim (Butler”s essay is not so much about American English spelling rules as it is about American intellectual culture at the turn of the 20th century: it is inclined toward the petty and the self-important . … He uses metaphor, personification, anecdote, and sharp irony to skewer not only the American elite but also himself ). The response maintains a consistent focus on this central claim, and supports it with a clear and deliberate analysis of Butler”s essay. The author shows particular skill in verb choice (defies logic … builds his argument … delving into commentary … establish his wry and ironic tone … lashes the “spelling reformers” ), strength in using parallel structures (not only the American elite but also himself … while Shakespeare (a Brit) was clearly a genius, Noah Webster (the American lexicographer) was less so … the problem is not practical but moral ), and thoughtfulness in diction and phrasing ([h]e mounts his high horse with indignant braggadocioperhaps as a lone insurgent … perhaps as a lone insurgent … [h]is satire even extends to the peculiarly American obsession with committee meetings and reform projects ). Largely free of grammatical error, this response demonstrates strong command of language and advanced proficiency in writing.

PRACTICE TEST 2

  1. Reading Test

65 MINUTES 52 QUESTIONS

  1. Writing and Language Test

35 MINUTES 44 QUESTIONS

  1. Math Test – No Calculator

25 MINUTES 20 QUESTIONS

  1. Math Test – Calculator

55 MINUTES 38 QUESTIONS

  1. Essay (optional)

50 MINUTES 1 QUESTION

ANSWER SHEET

Start with number 1 for each new section. If a section has fewer questions than answer spaces, leave the extra answer spaces blank. Be sure to erase any errors or stray marks completely.

Start with number 1 for each new section. If a section has fewer questions than answer spaces, leave the extra answer spaces blank. Be sure to erase any errors or stray marks completely.

CAUTION Use the answer spaces in the grids below for Section 3 only if you are told to do so in your test book.

Student-Produced Responses ONLY ANSWERS ENTERED IN THE CIRCLES IN EACH GRID WILL BE SCORED. YOU WILL NOT RECEIVE CREDIT FOR ANYTHING WRITTEN IN THE BOXES ABOVE THE CIRCLES.

Start with number 1 for each new section. If a section has fewer questions than answer spaces, leave the extra answer spaces blank. Be sure to erase any errors or stray marks completely.

CAUTION Use the answer spaces in the grids below for Section 4 only if you are told to do so in your test book.

Student-Produced Responses ONLY ANSWERS ENTERED IN THE CIRCLES IN EACH GRID WILL BE SCORED. YOU WILL NOT RECEIVE CREDIT FOR ANYTHING WRITTEN IN THE BOXES ABOVE THE CIRCLES.

SECTION 5: ESSAY

You may wish to remove these sample answer document pages to respond to the practice SAT Essay Test.

SECTION 5: ESSAY

You may wish to remove these sample answer document pages to respond to the practice SAT Essay Test.

SECTION 5: ESSAY

You may wish to remove these sample answer document pages to respond to the practice SAT Essay Test.

SECTION 5: ESSAY

You may wish to remove these sample answer document pages to respond to the practice SAT Essay Test.

Reading Test

65 MINUTES, 52 QUESTIONS

Turn to Section 1 of your answer sheet to answer the questions in this section.

DIRECTIONS

Each passage or pair of passages below is followed by a number of questions. After reading each passage or pair, choose the best answer to each question based on what is stated or implied in the passage or passages and in any accompanying graphics.

Questions 1–10 are based on the following passage.

This passage is from Ralph Waldo Emerson, “Prudence.” Public domain. First published in 1841.

1

The tone of the first paragraph is best described as

  1. A) self-effacing.
  2. B) pontifical.
  3. C) aspirational.
  4. D) sardonic.

2

The author”s reference to “some other garden” (lines 8–9) primarily suggests that he

  1. A) finds solace in the art of planting.
  2. B) seeks new challenges and experiences.
  3. C) considers arable land to be a valuable resource.
  4. D) lacks the particular skills associated with farming.

3

In line 11, “title” most nearly means

  1. A) ownership.
  2. B) office.
  3. C) authority.
  4. D) publication.

4

The author believes that he is justified in acting as an authority on prudence primarily because of his

  1. A) experience in making decisions.
  2. B) regret for his past mistakes.
  3. C) studies in classical philosophy.
  4. D) yearning for wisdom.

5

Which choice provides the strongest evidence for the answer to the previous question?

  1. A) Lines 6–9 (“I have no skill . . . some other garden”)
  2. B) Lines 13–14 (“We write from . . . as well as from experience”)
  3. C) Lines 20–22 (“Yet it would . . . coarser sound”)
  4. D) Lines 26–28 (“It is content . . . laws of the intellect”)

6

The passage suggests that members of the “third class” (line 46) are superior for their ability to

  1. A) solve important problems.
  2. B) discern sublime qualities.
  3. C) create works of beauty.
  4. D) reason logically.

7

The “houses and barns” (line 54) represent

  1. A) an unwise allegiance to worldly things.
  2. B) the rejection of mere symbols.
  3. C) the nobility of living with nature.
  4. D) the importance of strong belief.

8

In line 58, “base” most nearly means

  1. A) supportive.
  2. B) ignoble.
  3. C) necessary.
  4. D) straightforward.

9

The “disease” mentioned in line 64 is best described as

  1. A) apathy.
  2. B) gluttony.
  3. C) sensuousness.
  4. D) egotism.

10

The passage as a whole characterizes prudence primarily as.

  1. A) the aspiration to wisdom and righteousness.
  2. B) a commitment to aesthetic principles.
  3. C) the pursuit of practical skills and sensory experience.
  4. D) the noble pursuit of spiritual goals.

Questions 11–21 are based on the following passage.

This passage is from Joseph Conrad, The Secret Sharer . It was originally published in 1912. The narrator of this story is the captain of a ship about to begin a voyage.

11

The tone of the first paragraph (lines 1–13) is primarily one of

  1. A) reflective anticipation.
  2. B) anxious dread.
  3. C) unrestrained excitement.
  4. D) objective analysis.

12

The reference to “some glare” (line 14) serves primarily to make the point that

  1. A) the mastheads of another ship were not immediately visible.
  2. B) the weather was about to change.
  3. C) the ocean around the ship was choppy.
  4. D) the crew was eager to get into the open sea.

13

In lines 20–24 (“The tide . . . friend”) the narrator describes

  1. A) signs of impending danger.
  2. B) reflections of his deep inner turmoil.
  3. C) objects of wistful contemplation.
  4. D) the recollection of a tragic experience.

14

The captain is portrayed primarily as

  1. A) self-conscious and diffident.
  2. B) rugged and adventurous.
  3. C) anxious and short-tempered.
  4. D) scholarly yet intimidating.

15

Which choice provides the strongest evidence for the answer to the previous question?

  1. A) Lines 4–5 (“At that moment . . . her decks”)
  2. B) Lines 24–27 (But, with . . . for good”)
  3. C) Lines 48–51 (“In consequence . . . fortnight before”)
  4. D) Lines 63–65 (“But I . . . himself secretly”)

16

In line 55, “bearing” most nearly means

  1. A) direction.
  2. B) demeanor.
  3. C) relevance.
  4. D) endurance.

17

In line 69, “evolve” most nearly means

  1. A) change slowly.
  2. B) ponder strenuously.
  3. C) persuade earnestly.
  4. D) advance randomly.

18

The “truth” to which the narrator refers in lines 57 is his

  1. A) skepticism about his crew”s ability.
  2. B) apprehension about a dangerous voyage.
  3. C) lack of self-confidence.
  4. D) sense that he may be going insane.

19

In line 81, “exercised” most nearly means

  1. A) practiced.
  2. B) strengthened.
  3. C) utilized.
  4. D) disquieted.

20

The “collaboration” (line 67) refers to an act of

  1. A) selfless assistance.
  2. B) deliberate menace.
  3. C) contrived deceit.
  4. D) strained contemplation.

21

The chief mate believed that, compared to the recently discovered ship, the “scorpion” (line 74) was

  1. A) less explicable.
  2. B) more frightening.
  3. C) more ominous.
  4. D) less miserable.

Questions 22–32 are based on the following passages.

Passage 1 is from Lindsay Smith-Doyle, “Thoughts on the Value of Life.” ©2015 by College Hill Coaching. Passage 2 is from Christopher F. Black, “Who”s Afraid of Cloning?” ©2015 by Christopher F. Black. Since 1996, when scientists at the Roslin Institute in England cloned a sheep from the cells of another adult sheep, many have debated the ethics of cloning human cells. These passages are excerpts from arguments on this issue.

Passage 1

Passage 2

22

In line 13, “control” refers specifically to control over

  1. A) the effects of cloning.
  2. B) the development of genetic technologies.
  3. C) the process of conception.
  4. D) the ethical debate about cloning.

23

In Passage 1, the author”s attitude toward “outsourcing” (line 8) is one of

  1. A) grudging approval.
  2. B) blunt disdain.
  3. C) firm support.
  4. D) ironic detachment.

24

The quotations in line 15 and line 78 are similar in that both

  1. A) represent the opinions of cloning opponents.
  2. B) indicate cautious advocacy for genetic engineering.
  3. C) are presented as being insincere.
  4. D) contradict the viewpoints of the respective authors.

25

Jeremy Rifkin (line 63) would most likely advocate

  1. A) the “humility” mentioned in line 7.
  2. B) the “design and control” mentioned in line 13.
  3. C) the “engineering” mentioned in line 16.
  4. D) the “industries” mentioned in line 30.

26

The accompanying diagram best illustrates

  1. A) the “guided purpose” mentioned in line 22.
  2. B) the “assembly line” mentioned in line 24.
  3. C) the “course of human evolution” mentioned in lines 33–34.
  4. D) the “procedures” mentioned in line 38.

27

In line 52, “introducing” refers to an act of

  1. A) explanation.
  2. B) proposition.
  3. C) announcement.
  4. D) injection.

28

The author of Passage 1 would most likely regard the “management” (line 61) as

  1. A) a necessary measure to avoid the abuse of procreative technologies.
  2. B) an acceptable means by which the medical community can find alternatives to cloning.
  3. C) a regrettable invasion of commercial interests into human reproduction.
  4. D) a dangerous impediment to the development of effective cloning techniques.

29

Passage 2 quotes Jeremy Rifkin in line 64 primarily to

  1. A) exemplify an untenable position.
  2. B) illustrate the potential dangers of cloning.
  3. C) reveal the interests of the corporate community.
  4. D) cite a corroborating opinion from an expert.

30

Passage 2 refers to the Twin”s Day Festival in line 68 as an example of

  1. A) a movement that promotes beneficial cloning.
  2. B) a seemingly harmless event that harbors hidden dangers.
  3. C) the innocuousness of genetic duplication.
  4. D) the logical consequences of procreative technologies.

31

The author of Passage 2 would most likely argue that the “procedures” to which the author of Passage 1 objects in line 38 are in fact

  1. A) inconsequential aspects of the cloning debate.
  2. B) necessary contributions to medical progress.
  3. C) not representative of the methods used by real genetic researchers.
  4. D) ways of manipulating public opinion.

32

Which choice provides the strongest evidence for the answer to the previous question?

  1. A) Lines 59–61 (“It is the end . . . management”)
  2. B) Lines 71–73 (“Each identical . . . beliefs”)
  3. C) Lines 80–82 (“It is the reasoning . . . place”)
  4. D) Lines 95–100 (“With genetic . . . organs”)

Questions 33–42 are based on the following passage.

This passage is from Steven Pinker, An Invitation to Cognitive Science (Gleitman, Liberman, and Osherson, eds.) ©1995 by Bradford Book.

33

This passage as a whole is primarily concerned with

  1. A) delineating the general principles of linguistics.
  2. B) comparing the structural qualities of various languages.
  3. C) exploring academic questions about how we learn language.
  4. D) examining the claims of one influential linguist.

34

The “data” mentioned in line 6 most likely include information regarding

  1. A) the literacy levels of various countries.
  2. B) methods for teaching infants to speak.
  3. C) the syntax rules of different languages.
  4. D) the structures of the human cerebral cortex.

35

In line 2, “the two” refers to

  1. A) self and other.
  2. B) thinking and expressing.
  3. C) grammar and syntax.
  4. D) learning and teaching.

36

In line 15, “sticking” most nearly means

  1. A) applying.
  2. B) upholding.
  3. C) piercing.
  4. D) maintaining.

37

The author”s attitude toward Whorf”s “hypothesis” (line 18) is best described as

  1. A) dismissive.
  2. B) supportive.
  3. C) ambivalent.
  4. D) antagonistic.

38

The statement “Babies can think before they can talk” (line 27) is intended to indicate that

  1. A) learning to talk is much more cognitively challenging than most people believe.
  2. B) skills associated with basic reasoning are not dependent on verbal communication.
  3. C) both physical and cognitive skills tend to develop according to rigid timelines.
  4. D) researchers sometimes do not take into account the particular needs of infants.

39

Which if the following best summarizes the author”s view on human language acquisition?

  1. A) Learning a language is a crucial step in learning to think, because thinking is verbal behavior.
  2. B) The structures for learning language seem to be much simpler than what scientists previously thought.
  3. C) Humans are born with very intricate cognitive structures for learning language.
  4. D) Environmental input is more important than heredity in language acquisition.

40

Which choice provides the strongest evidence for the answer to the previous question?

  1. A) Lines 1–3 (“Language is . . . intimately related”)
  2. B) Lines 23–24 (“Language acquisition . . . to talk”)
  3. C) Lines 61–64 (“The same . . . Vietnamese”)
  4. D) Lines 95–97 (“Hence language . . . general intelligence”)

41

In line 62, “structure” refers to

  1. A) the grammatical rules of a language.
  2. B) the functional organization of the mind.
  3. C) the environment in which infants learn.
  4. D) the systems for investigating linguistic claims.

42

The subjects listed in lines 78–80 are given as examples of disciplines that, in 1959,

  1. A) accepted the hypothesis that cognition depends on verbal skills.
  2. B) considered the scientific method inadequate to the study of language acquisition.
  3. C) regarded most of the processes in involved in language acquisitions to be innate.
  4. D) questioned the conventional theories regarding how humans learn language.

Questions 43–52 are based on the following passage and supplementary material.

This passage is adapted from Rick Smolan and Phillip Moffitt, “Medicine”s Great Journey .” ©1992 by Schering Laboratories, Calloway Editions.

43

The passage as a whole serves primarily to

  1. A) outline the various means by which the human immune system fights infection.
  2. B) describe the history and scientific underpinnings of a medical technique.
  3. C) compare the medical practices of different cultures throughout history.
  4. D) identify particular controversies surrounding a therapeutic method.

44

The passage mentions the “Chinese and Turks” (lines 7–8) as examples of cultures that

  1. A) identified the cause of viral infections.
  2. B) employed early forms of vaccination.
  3. C) were misguided in their use of medical remedies.
  4. D) were decimated by deadly infectious diseases.

45

The passage indicates that viruses cause all of the following EXCEPT

  1. A) tuberculosis.
  2. B) cowpox.
  3. C) polio.
  4. D) hepatitis.

46

Which of the following statements about vaccines is best supported by the passage?

  1. A) Some effective remedies for infectious disease were used before their mechanisms were understood.
  2. B) The discovery of bacteria was key to the development of the first successful vaccine.
  3. C) Vaccines consist of antibodies that are introduced into the bloodstream to ward off infections.
  4. D) Vaccines work by thwarting the process that allows viruses to reproduce themselves.

47

Which sentence provides the best evidence for the answer to the previous question?

  1. A) Lines 1–4 (“Vaccination is . . . sickness”)
  2. B) Lines 18–21 (“In 1895 . . . worked”)
  3. C) Lines 24–25 (“In Germany . . . tuberculosis”)
  4. D) Lines 38–42 (“It must . . . benefit”)

48

The discussion in the third paragraph (lines 30–55) suggests that relationship between the virus and the cell is most similar to the relationship between

  1. A) a cowbird and the Eastern phoebe, in which the cowbird lays its eggs in the nest of the phoebe, which raises the young as its own.
  2. B) a grizzly bear and a salmon, in which the bear captures and eats the salmon before it can spawn, thereby reducing the population of the next generation.
  3. C) a tickbird and a rhinoceros, in which the tickbird cleans the parasites that live on the rhinoceros, thereby providing benefit to both animals.
  4. D) a bumblebee and a flower, in which the bumblebee gathers nectar from the flower, while also distributing the pollen of the flower to distant plants.

49

The passage suggests that bacteria are similar to viruses in that they

  1. A) can provide benefit as well as inflict harm.
  2. B) can reproduce even in very harsh environments.
  3. C) have played a major role in the development of vaccines.
  4. D) can be parasitic to their hosts.

50

In line 44, “signature” most nearly means

  1. A) proprietary.
  2. B) deleterious.
  3. C) distinctive.
  4. D) advantageous.

51

Which of the following best describes the function of the final paragraph?

  1. A) It presents a generalization to explain the examples mentioned in the previous paragraph.
  2. B) It answers a question suggested in the previous paragraph.
  3. C) It gives an example illustrating a concept introduced in the previous paragraph.
  4. D) It qualifies the tone of the previous paragraph.

52

The two graphs (Figure 1 and Figure 2) are presented together most likely to illustrate

  1. A) the fact that measles cases were already beginning to decline before the vaccines were licensed.
  2. B) the wide range of variables that can affect the communicability of an infectious disease.
  3. C) the merits of administering a vaccine in a single dose as opposed to over an extended period.
  4. D) the effects of the measles vaccine administered to comparable populations but at different times.

STOP

If you finish before time is called, you may check your work on this section only. Do not turn to any other section of the test.

Writing and Language Test

35 MINUTES, 44 QUESTIONS

Turn to Section 2 of your answer sheet to answer the questions in this section.

DIRECTIONS

Each passage below is accompanied by a number of questions. For some questions, you will consider how the passage might be revised to improve the expression of ideas. For other questions, you will consider how the passage might be edited to correct errors in sentence structure, usage, or punctuation. A passage or a question may be accompanied by one or more graphics (such as a table or graph) that you will consider as you make revising and editing decisions.

Some questions will direct you to an underlined portion of a passage. Other questions will direct you to a location in a passage or ask you to think about the passage as a whole.

After reading each passage, choose the answer to each question that most effectively improves the quality of writing in the passage or that makes the passage conform to the conventions of Standard Written English. Many questions include a “NO CHANGE” option. Choose that option if you think the best choice is to leave the relevant portion of the passage as it is.

Questions 111 are based on the following passage and supplementary material.

Who Really Owns American Media?

In this era of blogging, news websites, and personalized Twitter feeds, most of us believe that we have more choice than ever 1 in how we get our news. But unless you”re particularly 2 apt about the world of journalism, you might be surprised to learn how few choices we really have.

1

  1. A) NO CHANGE
  2. B) with getting
  3. C) of the way we get
  4. D) of getting

2

  1. A) NO CHANGE
  2. B) acute
  3. C) savvy
  4. D) comprehensive

Thirty years ago, 50 different corporations owned 90% of the American broadcast and news media. Today, just 6 large conglomerates 3 have the same control over that media, which is still 90% . These huge corporations have successfully lobbied the U.S. Congress to loosen or dismantle federal antitrust regulations. These regulations were designed to prevent any one corporation from driving out 4 their competition and controlling public discourse. The debate on this issue centers on the balance between liberties and governmental interference. Some argue that a corporation”s freedom to acquire media and voice its opinion trumps any right the public may have to diverse points of view. 5 The other argument would be that our constitutional freedom of the press requires regulation in order to maintain a free market of ideas and an informed citizenry.

3

  1. A) NO CHANGE
  2. B) control that same 90% of all media
  3. C) control the same media, all 90% of it
  4. D) are in the same 90% control of all media

4

  1. A) NO CHANGE
  2. B) the competition they have
  3. C) its competition
  4. D) it”s competition

5

  1. A) NO CHANGE
  2. B) Others argue
  3. C) Others would argue
  4. D) Another being

According to data from 2007, the American media does not quite look like America. Although fully 33% of the American population was minority, 6 only 3.2% of American broadcast television outlets were controlled by minorities .

One potent antidote 7 regarding media consolidation is the Internet. 8 With some research, it reveals many resources for the curious and intelligent media consumer to hear informed voices from a wide variety of perspectives.

6

Which of the following best represents the information from Figure 1?

  1. A) NO CHANGE
  2. B) only 3.2% of the minority population controlled American broadcast television outlets
  3. C) only 3.2% of the American population included minorities in control of broadcast television outlets
  4. D) only 3.2% of American broadcast television stations were watched by minorities

7

  1. A) NO CHANGE
  2. B) about
  3. C) against
  4. D) to

8

  1. A) NO CHANGE
  2. B) It will reveal with some research
  3. C) Some research will reveal
  4. D) With some research, it will reveal

9 Although the Web abounds with gossip, partisanship, and fear-mongering from many major outlets, and conspiracy theorists on the fringe , the careful viewer can also find thoughtful analysis and civilized debate of the issues. Sites like ProPublica, FactCheck.org, and NPR provide in-depth, nonprofit, public-supported journalism that is less influenced by any corporate or political agenda.

10 Therefore , sensationalism sells, and the media conglomerates have mastered the art. As the first great American media mogul, William Randolph Hearst, said, “If you want the public in sufficient numbers, construct a highway. Advertising is that highway.” Without large advertising and lobbying budgets, these nonpartisan 11 instances of journalism will have a difficult time competing with the big boys.

9

The author wants to introduce this sentence with a representation of modern media that contrasts with the ideal of “civilized debate.” Does this introduction accomplish this task?

  1. A) Yes, because it suggests that controversial matters are ignored in modern media.
  2. B) Yes, because it refers to relatively unsophisticated modes of conversation.
  3. C) No, because it focuses on entertainment rather than any examination of issues.
  4. D) No, because it refers to hypothetical situations rather than real ones.

10

  1. A) NO CHANGE
  2. B) Still
  3. C) Lastly
  4. D) In summary

11

  1. A) NO CHANGE
  2. B) patterns
  3. C) receptacles
  4. D) repositories

Questions 1222 are based on the following passage.

The Dangers of Superstition

Have you ever knocked on wood to dodge a jinx? Do you avoid stepping on cracks in the sidewalk? Do you feel uneasy about the number 13? Most of us realize that these 12 rituals, which are based on ancient and discredited beliefs, but we can”t so easily rid our minds of superstitious thinking. Every culture has its own superstitious beliefs, 13 and now anthropologists and psychologists are beginning to understand why.

Our brains constantly work to find cause-and-effect patterns in the world. When something strange happens that we can”t explain, or seems to 14 collid against what we already believe, we get an uncomfortable feeling known as “cognitive dissonance.” We reflexively fill this gap in knowledge with the explanations that are most easily available to us. 15 Since we are willful beings surrounded by other willful beings, and every conscious moment of our lives is filled with a sense of “agency,” 16 that is: intentional action . Therefore, we imagine tiny beings living in wood, or vaporous spirits roaming the clouds that do strange or harmful things when we displease them. Willful agency is our “default” explanation.

12

  1. A) NO CHANGE
  2. B) rituals, that are based on
  3. C) rituals have been based on
  4. D) rituals are based on

13

  1. A) NO CHANGE
  2. B) for
  3. C) so
  4. D) while

14

  1. A) NO CHANGE
  2. B) contradict
  3. C) disengage
  4. D) go away from

15

  1. A) NO CHANGE
  2. B) Because we
  3. C) We
  4. D) So we

16

  1. A) NO CHANGE
  2. B) that is, intentional action
  3. C) which is what intentional action is
  4. D) which is: intentional action

[1] Our brains are creative. [2] They can design buildings, compose music, and 17 can formulate scientific theories. [3] But this creativity is sometimes hard to discipline, and so we are susceptible to strange thoughts and superstitions. [4] Many of these, like blessing people when they sneeze, are harmless if not quaint. [5] In 2014, villagers in Nigeria brought a goat into a police station, accusing it of being a witch that had attempted to steal a car and then changed into a goat. 18

17

  1. A) NO CHANGE
  2. B) also can formulate
  3. C) have formulated
  4. D) formulate

18

The author is considering adding the following sentence to this paragraph.

Others are sad and bizarre, such as the belief in shape-shifting .

Where should it be placed?

  1. A) before sentence 2
  2. B) before sentence 3
  3. C) before sentence 4
  4. D) before sentence 5

Although superstitious explanations relieve our cognitive dissonance, 19 it might also lead to tragedy. In 2014, people in Paraguay and Tanzania were killed because locals accused them of witchcraft. 20 Some superstitious parents have even beaten or disowned their own children because their strange behavior is attributed to demonic possession. Superstitions are also not harmless when they impede the pursuit of science, placing obstacles in the way of medical and technological breakthroughs that can improve the human condition.

Rituals intended to help your favorite football team score, like dancing or wearing your hat backward, are fun and innocuous. They 21 substitute a craving in our brains for control over situations that otherwise mystify us. 22

19

  1. A) NO CHANGE
  2. B) it can
  3. C) they can
  4. D) they would

20

  1. A) NO CHANGE
  2. B) Some superstitious parents, believing that any strange behavior is a sign of demonic possession, have even beaten or disowned their own children.
  3. C) Even beating or disowning their own children, many superstitious parents attribute their strange behavior to demonic possession.
  4. D) Some superstitious parents, believing that their strange behavior is a sign of demonic possession, have even beaten or disowned their own children.

21

  1. A) NO CHANGE
  2. B) discharge
  3. C) exempt
  4. D) satisfy

22

Which concluding sentence would be most in keeping with the content and tone of the passage as a whole?

  1. A) However, feeling like we have control over a situation is not always the same as understanding it.
  2. B) They represent some of humanity”s greatest accomplishments, and have inspired some of our greatest works of art.
  3. C) Centuries from now, our rituals may become so elaborate that we would scarcely recognize them as such today.
  4. D) Without such rituals, we would not feel as connected to the people or the natural world around us.

Questions 2333 are based on the following passage.

Skepticism and the Scientific Method

Even scientists sometimes forget how essential skepticism, particularly self-skepticism, is to the scientific process. But scientific skepticism is driven by evidence, not agenda. Today, the field of climatology seems to have more than its share of skeptics, debating 23 a warming planet and the things that should be done by us about it, if anything .

24 They are coming from outside of the scientific community, many of these skeptics couch their arguments in political terms. Some claim that global warming is part of a partisan “left-wing” plot or a ploy by the scientific community to ensure funding for yet another “Chicken Little” scare. Others suggest that attempts to reduce greenhouse gas emissions by changing energy or land use policies 25 would provide a needless cost of the American taxpayer of tens to hundreds of billions of dollars annually. Some even suggest that they are really part of an international conspiracy to undermine America”s competitiveness in the global marketplace.

23

  1. A) NO CHANGE
  2. B) what should be done about a warming planet, if we should
  3. C) what, if anything, we should do about a warming planet
  4. D) the things we should do about a warming planet, if we should

24

  1. A) NO CHANGE
  2. B) While coming
  3. C) Their coming
  4. D) Coming

25

  1. A) NO CHANGE
  2. B) would be needless in costing the American taxpayer
  3. C) would needlessly cost the American taxpayer
  4. D) is a needless cost to the American taxpayer of

At the same time, others who legitimately question the data or theories related to climate change are too quickly labeled right-wing “deniers,” even if their concerns are not motivated by any partisan convictions.

In fact, science has, or should have, nothing to do with ideology. Rather, it 26 is a process of identifying significant natural phenomena, gathering evidence about those phenomena, and 27 then we must find the most reliable explanation for that evidence. The preponderance of the evidence suggests that the earth is getting warmer, that the effects of that warming will be problematic, that there are things we can do to prevent or at least mitigate the worst outcomes, and 28 perhaps that many of these things are well worth doing. There is still plenty of uncertainty about the complex systems that make up our planetary climate, but we know enough to be concerned, 29 and to discuss the issue without politicizing it .

26

  1. A) NO CHANGE
  2. B) accounts for
  3. C) represents
  4. D) symbolizes

27

  1. A) NO CHANGE
  2. B) finding the most reliable explanation for
  3. C) then explaining in the most reliable way
  4. D) finding the most reliable way for explaining

28

  1. A) NO CHANGE
  2. B) also that many of these things perhaps may be
  3. C) many of these things perhaps may be
  4. D) that many of these things may be

29

In the context of the passage as a whole, which of the following completes the sentence most appropriately?

  1. A) NO CHANGE
  2. B) and to expose the agendas of those who stand in the way of saving our planet
  3. C) but not enough to risk sacrificing our political or economic security
  4. D) and to create a strong incentive program to transform our national energy policy

The skeptics point out, rightly, that science isn”t about consensus. The fact that 98% of climatologists regard something as true 30 isn”t the same as it being true . After all, only centuries ago the majority of physicians worldwide believed that illnesses were caused not by germs or genetics, 31 but by demons or imbalances in “humors.”

[1] Having an honest and productive conversation about global warming 32 requires an educated public. [2] When we, as public citizens, become more informed about the science of climatology, we become less susceptible to political sniping and to “consensus” as an argument. [3] Most important, perhaps, we become better able to make good decisions about the future of our nation and our planet. 33

30

  1. A) NO CHANGE
  2. B) won”t make that true
  3. C) would not mean it”s that way
  4. D) doesn”t make it so

31

  1. A) NO CHANGE
  2. B) but instead from
  3. C) but from
  4. D) they thought it was by

32

  1. A) NO CHANGE
  2. B) requires the need for
  3. C) requires our being
  4. D) require having

33

The author is considering inserting the following sentence into this paragraph.

Furthermore, we become more adept at evaluating the facts and theories at the heart of the matter.

Where should it be placed?

  1. A) before sentence 1
  2. B) before sentence 2
  3. C) before sentence 3
  4. D) after sentence 3

Questions 3444 are based on the following passage.

The Magic of Bohemia

Bohemia is a landlocked country in central 34 Europe, and until 1918 they were ruled from Vienna by the Austrian Hapsburgs. Today it 35 regards a major part of the modern Czech Republic, and its largest city, Prague, serves as the nation”s capital. Bohemia is also another, less clearly defined country, a country of the mind. This Bohemia in fact derives from misconceptions about the true Bohemia that go back as far as Shakespeare, 36 designating Bohemiaas the land of gypsies and the spiritual habitation of artists.

By 1843, when Michael William Balfe”s opera The Bohemian Girl premiered in London, the term Bohemian 37 would come to mean any wandering or vagabond soul, who need not have been associated with the arts. The Parisian poet Henry Murger clinched the term”s special association with the life of artists.

In November 1849, a dramatized version of Murger”s Latin Quarter tales was staged in Paris with the title La Vie de Bohème . So extraordinarily successful 38 did this prove that the stories themselves were published as Scènes de la Vie de Bohème . The public”s appetite was whetted and a popular cult of the gypsy-artist was underway. Murger”s volume of stories became the textbook for the artistic life throughout the late nineteenth and early twentieth centuries.

34

  1. A) NO CHANGE
  2. B) Europe, until 1918 it was ruled
  3. C) Europe, which, until 1918, was ruled
  4. D) Europe, having been, until 1918, ruled

35

  1. A) NO CHANGE
  2. B) amounts to
  3. C) establishes
  4. D) comprises

36

  1. A) NO CHANGE
  2. B) who designated Bohemia
  3. C) he had designated Bohemia
  4. D) being designated by him

37

  1. A) NO CHANGE
  2. B) had come to mean
  3. C) came to have meant
  4. D) had meant

38

  1. A) NO CHANGE
  2. B) was this proven
  3. C) this was proved
  4. D) this proved

39 What was it that were the basic elements of this Bohemia as it evolved under Murger? To start with, Bohemia belonged to the romantic movements that preached the power of the individual imagination and came to adopt a secular religion of art. Like early Christianity, it had its true believers and its heathens. The believers in this case were the artists themselves, the elect of the spirit, touched with the divine power of imagination, while the heathen were the commercial middle classes who had 40 propagated as a result of increased commodity production in the wake of the Industrial Revolution.

[1] To the artists, these were people of no imagination who were only concerned with material things. [2] As Philistines, they seemed inhabit a different country from that of the 41 Bohemians; Murger”s achievement was to define, quite persuasively, the boundaries of Bohemia in terms of a particular lifestyle. [3] In his Bohemia, the production of art was in fact less important than 42 whether one had the capacity for art . [4] Murger was also responsible for the term Bohemian becoming inseparably linked with the supposedly unconventional, outlandish behavior of artists, yet it is evident that he did not invent Bohemianism. [5]

39

  1. A) NO CHANGE
  2. B) What were they that were the basic elements
  3. C) What basic elements were there
  4. D) What were the basic elements

40

  1. A) NO CHANGE
  2. B) propitiated
  3. C) prospered
  4. D) preempted

41

  1. A) NO CHANGE
  2. B) Bohemians, Murger had the achievement of defining
  3. C) Bohemians, but Murger”s achievement was in defining
  4. D) Bohemians; but Murger achieved defining

42

  1. A) NO CHANGE
  2. B) the capacity for art
  3. C) whether one has the capacity for art
  4. D) one”s capacity of art

Most of its ingredients had existed in Paris for at least two decades before he started writing. 43

Bohemia had been a haven for the political rebel and, as the nineteenth century drew to a close, more than one French observer had seen it as the breeding-ground of cynicism, as the source of much potential danger. “It is quite clear,” Jules Claretie wrote indignantly in 1888, “that every country has its Bohemians. But they do not have the influence over the rest of the nation which they do in France—thanks to that poisonous element in the French character which is known as la blague —or cynicism.” 44

43

The author is considering including the following sentence in this paragraph.

Murger can thus be described as a Bohemian of the second generation .

Where should it be placed?

  1. A) after sentence 1
  2. B) after sentence 2
  3. C) after sentence 3
  4. D) after sentence 5

44

If the author were to delete the quotation from Jules Claretie at the conclusion of this paragraph, the passage would primarily lose

  1. A) a sanguine view of the late nineteenth-century French culture
  2. B) an scathing perspective on Murger”s literary work
  3. C) a dire assessment of France”s national temperament
  4. D) an urgent warning against a potential immigration problem

STOP

If you finish before time is called, you may check your work on this section only. Do not turn to any other section of the test.

Math Test – No Calculator

25 MINUTES, 20 QUESTIONS

Turn to Section 3 of your answer sheet to answer the questions in this section.

DIRECTIONS

For questions 1–15, solve each problem, choose the best answer from the choices provided, and fill in the corresponding circle on your answer sheet. For questions 16–20, solve the problem and enter your answer in the grid on the answer sheet. Please refer to the directions before question 16 on how to enter you answers in the grid. You may use any available space in your test booklet for scratch work.

NOTES

  1. The use of a calculator is NOT permitted.
  2. All variables and expressions used represent real numbers unless otherwise indicated.
  3. Figures provided in this test are drawn to scale unless otherwise indicated.
  4. All figures lie in a plane unless otherwise indicated.
  5. Unless otherwise indicated, the domain of a given functionfis the set of all real numbers for which f (x ) is a real number.

REFERENCE

The number of degrees of arc in a circle is 360.

The number of radians of arc in a circle is 2π.

The sum of the measures in degrees of the angles of a triangle is 180.

1

If 2b − 1 = 5, what is the value of 2b 2 − 1?

  1. A) 15
  2. B) 17
  3. C) 24
  4. D) 25

2

In the figure above, points P , Q , R , S , and T lie on the same line, and R is the center of the large circle. If the three smaller circles are congruent and the radius of the large circle is 6, what is the radius of one of the smaller circles?

  1. A) 1
  2. B) 2
  3. C) 3
  4. D) 4

3

Jeri has edited of her term paper. If she has edited 15 pages, how many pages does she have left to edit?

  1. A) 45
  2. B) 50
  3. C) 60
  4. D) 75

4

7, 12, 22, 42, 82

Which of the following gives a rule for finding each term in the sequence after the first?

  1. A) Add 5 to the preceding number.
  2. B) Add 5 to the sum of all of the preceding terms.
  3. C) Double the preceding term and then subtract 2 from the result.
  4. D) Add 14 to the preceding term and divide that result by 2.

5

The figure above shows a rectangular box. What is the longest length of a diagonal of one of the faces of this box?

  1. A)
  2. B)
  3. C)
  4. D)

6

Which of the following points is NOT on the graph of the line −2x − 3y = 36 in the xy -plane?

  1. A) (−9, 6)
  2. B) (−24, 4)
  3. C) (6, −16)
  4. D) (12, −20)

7

During a coyote repopulation study, researchers determine that the equation P = 250(1.32) t describes the population P of coyotes t years after their introduction into a new region. Which of the following gives the values of I , the initial population of coyotes, and r , the annual percent increase in this population?

  1. A) I= 250,r = 32%
  2. B) I= 250,r = 132%
  3. C) I= 330,r = 32%
  4. D) I= 330,r = 132%

8

Which of the following is equal to

  1. A)
  2. B)
  3. C)
  4. D)

9

Which of the following could be the x -intercept and y -intercept of a line that is perpendicular to the line 3x + 6y = 0?

  1. A) (−6, 0) and (0, 3)
  2. B) (3, 0) and (0, −6)
  3. C) (3, 0) and (0, 6)
  4. D) (6, 0) and (0, 3)

10

The function f is defined by the equation f (x ) = xx 2 . Which of the following represents a quadratic with no real zeros?

  1. A)
  2. B)
  3. C)
  4. D)

11

In the xy -plane, the graph of the line intersects the graph of the equation y = x 2 + x at two points. What is the distance between these two points?

  1. A)
  2. B)
  3. C)
  4. D) 4

12

If i 2 k = 1, and , which of the following must be true about k ?

  1. A) kis a multiple of 4.
  2. B) kis a positive integer.
  3. C) When 2kis divided by 4, the remainder is 1.
  4. D) is an integer.

13

For all numbers x and y , let z be defined by the equation z = |22x 2y 2 | + 22 . What is the smallest possible value of z ?

  1. A) 0
  2. B) 4
  3. C) 8
  4. D) 16

14

If the polynomial P (x ) has factors of 12, (x − 5), and (x + 4), which of the following must also be a factor of P (x )?

  1. A) 2x2+ 8
  2. B) 4x2− 20
  3. C) 6x2− 6x − 120
  4. D) x2− 10x + 25

15

If f (x ) = –x + 7 and g (f (x )) = 2x + 1, what is the value of g (2)?

  1. A) −11
  2. B) −5
  3. C) 5
  4. D) 11

DIRECTIONS

For questions 16–20, solve the problem and enter your answer in the grid, as described below, on the answer sheet.

  1. Although not required, it is suggested that you write your answer in the boxes at the top of the columns to help you fill in the circles accurately. You will receive credit only if the circles are filled in correctly.
  2. Mark no more than one circle in any column.
  3. No question has a negative answer.
  4. Some problems may have more than one correct answer. In such cases, grid only one answer.
  5. Mixed numberssuch as 3 must be gridded as 3.5 or .

(If 3 is entered into the grid as , it will be interpreted as , not .)

  1. Decimal answers:If you obtain a decimal answer with more digits than the grid can accommodate, it may be either rounded or truncated, but it must fill the entire grid.

16

In a writer”s workshop, there are half as many men as women. If there are 24 total men and women in the writer”s workshop, how many men are there?

17

If what is the value of b ?

18

The square of a positive number is 0.24 greater than the number itself. What is the number?

19

The function f is a quadratic function with zeros at x = 1 and x = 5. The graph of y = f (x ) in the xy -plane is a parabola with a vertex at (3, −2). What is the y -intercept of this graph?

20

When graphed in the xy -plane, the line y = mx − 4 intersects the x -axis at an angle of θ . If m > 0, 0° < θ < 90°, and , what is the value of m ?

STOP

If you finish before time is called, you may check your work on this section only. Do not turn to any other section of the test.

Math Test – Calculator

55 MINUTES, 38 QUESTIONS

Turn to Section 4 of your answer sheet to answer the questions in this section .

DIRECTIONS

For questions 1–30, solve each problem, choose the best answer from the choices provided, and fill in the corresponding circle on your answer sheet. For questions 31–38, solve the problem and enter your answer in the grid on the answer sheet. Please refer to the directions before question 31 on how to enter you answers in the grid. You may use any available space in your test booklet for scratch work.

NOTES

  1. The use of a calculator is permitted.
  2. All variables and expressions used represent real numbers unless otherwise indicated.
  3. Figures provided in this test are drawn to scale unless otherwise indicated.
  4. All figures lie in a plane unless otherwise indicated.
  5. Unless otherwise indicated, the domain of a given functionfis the set of all real numbers for which f (x ) is a real number.

REFERENCE

The number of degrees of arc in a circle is 360.

The number of radians of arc in a circle is 2π.

The sum of the measures in degrees of the angles of a triangle is 180.

1

If and 2a + 4b = 20, what is the value of b ?

  1. A) 2.5
  2. B) 4
  3. C) 5
  4. D) 15

2

The spinner for a board game has 10 sectors, numbered 1 through 10. It is spun 20 times and the results summarized in the table above. What is the median value of these 20 spins?

  1. A) 2
  2. B) 4
  3. C) 5
  4. D) 6

3

A 48-gram serving of breakfast cereal contains 8 grams of sugar. How many grams of sugar are there in a 57-gram serving of the same cereal?

  1. A) 9.5
  2. B) 10.5
  3. C) 11.5
  4. D) 12.5

4

The graph above shows the number of applicants and finalists for a statewide college scholarship program over four consecutive years. For which year was the ratio of finalists to applicants the greatest?

  1. A) 2010
  2. B) 2011
  3. C) 2012
  4. D) 2013

5

If y 3 = 20 and z 2 = 10, what is the value of (yz )6 ?

  1. A) 2 × 105
  2. B) 4 × 104
  3. C) 2 × 105
  4. D) 4 × 105

6

If the sum of a , b , and c is three times the sum of a and b , which of the following expresses the value of a in terms of b and c ?

  1. A)
  2. B)
  3. C)
  4. D)

7

Note: Figure not drawn to scale.

In the figure above, BCDE is a rectangle, AC = 14, BC = 12, and EC = 13. What is the value of tan x ?

  1. A) 0.4
  2. B) 0.6
  3. C) 1.3
  4. D) 2.5

8

Which of the following binomials is a factor of x 2 − 6x + 8?

  1. A) x− 4
  2. B) x+ 4
  3. C) x+ 2
  4. D) x− 8

Questions 9–11 are based on the graph below.

The pie graph above represents the monthly ad sales for four salespeople—Maria, Eli, Georgia, and Zoe—at a social media website. For the month, Maria”s sales accounted for 25% of the total, Eli had $3,000 in sales, Georgia had $5,000 in sales, and Zoe had $10,000 in sales.

9

Which sector represents Georgia”s sales for the month?

  1. A) SectorA
  2. B) SectorB
  3. C) SectorC
  4. D) SectorD

10

What is the sum of the monthly sales for all four salespeople?

  1. A) $22,500
  2. B) $24,000
  3. C) $25,000
  4. D) $27,000

11

If Eli and Georgia both earn 10% commission on their sales, and Maria and Zoe both earn 15% commission on their sales, how much more did Maria earn in monthly commissions than Georgia?

  1. A) $300
  2. B) $360
  3. C) $375
  4. D) $400

12

Let the function f be defined by f (x ) = 2 − |x − 4| for all real values of x . What is the greatest value of f ?

  1. A) −2
  2. B) 2
  3. C) 4
  4. D) 6

13

If , what is the value of b ?

  1. A)
  2. B)
  3. C)
  4. D) 5

14

For the function f , f (1) = 4 and f (2) = 13. Which of the following equations could describe f ?

  1. A) f(x) = x 2 + 3
  2. B) f(x) = x 2 + 9
  3. C) f(x) = 2x 2 + 2
  4. D) f(x) = 3x 2 + 1

15

Which of the following is NOT equivalent to 12b 2 ?

  1. A) (6b)(6b)
  2. B) 12b(b)
  3. C)
  4. D) 6b2+ 6b 2

16

If m is a number chosen randomly from the set {2, 3, 4, 6} and n is a number chosen randomly from the set {1, 2, 3, 4}, what is the probability that mn is a multiple of 12?

  1. A)
  2. B)
  3. C)
  4. D)

17

If y = 3x + 4 and x < 3, which of the following represents all the possible values of y ?

  1. A) y> 7
  2. B) y< 13
  3. C) 7 <y< 13
  4. D) y> 13

18

If g (x + 1) = x 2 + 2x + 4 for all values of x , which of the following is equal to g (x )?

  1. A) x2+ 4
  2. B) x2+ 3
  3. C) (x− 1)2+ 4
  4. D) (x− 1)2+ 3

19

A: 2, 7, 12, 17, 22, . . .

B: 5, 15, 25, 35, 45, . . .

Two sequences, A and B, follow the patterns shown above. If the n th term of sequence A is 72, what is the n th term of sequence B?

  1. A) 125
  2. B) 135
  3. C) 145
  4. D) 155

20

A website received 2,100 visitors in July from both subscribers and nonsubscribers. If the ratio of subscribers to nonsubscribers among this group was 2:5, how many more nonsubscribers visited the site in July than subscribers?

  1. A) 126
  2. B) 630
  3. C) 900
  4. D) 1,260

21

The figure above shows the locations of quadrants I–IV in the xy -plane. Which of the following represents a pair of linear equations that do NOT intersect in quadrant I?

  1. A) 3x+ 5y= 15

y = 4

  1. B) 5x+ 3y= 15

y = 4

  1. C) 5x− 3y= 15

y = 4

  1. D) 3x− 5y= 15

y = 4

22

During a 40-minute session at a 220 volt charging station, the charge on an electric car battery increases from an initial charge of 50 power units to a final charge of 106 power units. If this charge increases linearly with time, which of the following best describes the charge, q , in power units, on this same battery after charging for t hours from an initial charge of 20 power units? (1 hour = 60 minutes)

  1. A) q= 55t+ 50
  2. B) q= 84t+ 50
  3. C) q= 55t+ 20
  4. D) q= 84t+ 20

Questions 23 and 24 are based on the graph below.

23

The scatterplot above shows the length and weight of a group of 20 salmon and the line of best fit for the data. According to this line of best fit, which of the following best approximates the weight, in kilograms, of a salmon that is 95 centimeters long?

  1. A) 7.6
  2. B) 7.8
  3. C) 8.3
  4. D) 8.8

24

Which of the following equations best describes the relationship between w , the weight in kilograms of each salmon, and l , its length in centimeters?

  1. A)
  2. B)
  3. C)
  4. D)

25

The average size of a compressed image file is 750 kB. If Ronika”s data plan allows her to send 2 GB of data each month before she pays any overage charges, but she plans to use 85% of that data for texting, approximately how many compressed images can she send each month before she incurs any overage charges? (1 GB = 1,000 MB; 1 MB = 1,000 kB)

  1. A) 227
  2. B) 400
  3. C) 2,267
  4. D) 4,000

26

Perfectioner”s Chocolate Company makes two varieties of truffles: dark chocolate and milk chocolate. Each dark chocolate truffle requires 0.65 ounces of cocoa powder, and each milk chocolate truffle requires 0.45 ounces of cocoa powder. If cocoa powder costs c dollars per pound, and Perfectioner”s Chocolate Company has budgeted $200 per week for cocoa powder, which of the following inequalities indicates the restrictions on the number of dark chocolate truffles, d , and the number of milk chocolate truffles, m , the company can make in one week? (1 pound = 16 ounces)

  1. A)
  2. B)
  3. C)
  4. D)

27

If n is a positive integer and m = 2 n + 2 + 2 n , what is 2 n + 3 in terms of m ?

  1. A) m
  2. B)
  3. C)
  4. D) 3m2

28

For how many values of x between 0 and 2π does sin

  1. A) Two
  2. B) Three
  3. C) Four
  4. D) Six

29

The figure above shows the graphs of functions f and g in the xy -plane. Which of the following equations could express the relationship between f and g ?

  1. A) f(x) = g (x − 2)
  2. B) f(x) = g (x + 2)
  3. C) f(x) = g (x ) + 2
  4. D) f(x) = g (x ) − 2

30

A researcher is trying to estimate the daily amount of time undergraduate computer science majors spend on nonrecreational computer activities. She surveys 120 students from among the computer science majors at a large state university and asks them, “How much time do you spend in nonrecreational computer activities each day?” The mean of these responses is 210 minutes per day, with a standard deviation of 16.5 minutes. If another researcher wishes to present the same question to a new set of subjects at the same university, which of the following subject groups would most likely yield a data set with a smaller margin of error for the estimated daily amount of time undergraduate computer science majors spend on nonrecreational computer activities?

  1. A) 240 randomly selected computer science majors
  2. B) 240 randomly selected liberal arts majors
  3. C) 80 randomly selected computer science majors
  4. D) 80 randomly selected liberal art majors

Student-Produced Response Questions

DIRECTIONS

For questions 31–38, solve the problem and enter your answer in the grid, as described below, on the answer sheet.

  1. Although not required, it is suggested that you write your answer in the boxes at the top of the columns to help you fill in the circles accurately. You will receive credit only if the circles are filled in correctly.
  2. Mark no more than one circle in any column.
  3. No question has a negative answer.
  4. Some problems may have more than one correct answer. In such cases, grid only one answer.
  5. Mixed numberssuch as must be gridded as 3.5 or .

(If is entered into the grid as , it will be interpreted as , not .)

  1. Decimal answers:If you obtain a decimal answer with more digits than the grid can accommodate, it may be either rounded or truncated, but it must fill the entire grid.

31

What number is 40% greater than the sum of 40 and 80?

32

The table above shows a set of ordered pairs that correspond to the function . What is the value of k ?

33

hx + 4y = −3

The equation above is the equation of a line in the xy -plane, and h is a constant. If the slope of this line is −13, what is the value of h ?

34

The sum of two numbers is four times their difference. The smaller of these numbers is 15. What is the greater number?

35

If 0 < x < 2π and , what is the value of

36

Note: Figure not drawn to scale.

In the figure above, the circle with center O has a circumference of 50, and AB = BC . What is the length of arc AB ?

Questions 37 and 38 are based on the scenario described below.

An Internet service provider offers three different plans for residential users. Plan A charges users $500 for the first year of service, and $80 per month thereafter. Plan B charges users $68 per month. Plan C is a “high speed” plan that offers 200% higher speeds for $92 per month.

37

Isabelle has been using Plan A for over a year. She recently reviewed her plan and realized that if she had been using Plan B for same amount of time, she would have saved $104 for Internet service over the entire period. At the time of her review, how many months had Isabelle been on Plan A?

38

Isabelle is now considering switching to either Plan B or Plan C for her home business, but she calculates that having the “high speed” plan will save her only approximately 45 minutes of work each month. At what minimum hourly rate, in dollars per hour, would she have to value her work (that is, how much more would she have to value one hour of free time over one hour of work time) for Plan C to be worth the extra cost over Plan B?

STOP

If you finish before time is called, you may check your work on this section only. Do not turn to any other section of the test.

Essay

50 MINUTES, 1 QUESTION

DIRECTIONS

As you read the passage below, consider how James Schlesinger uses

  • evidence, such as facts or examples, to support his claims
  • reasoning to develop ideas and connect claims and evidence
  • stylistic or persuasive elements, such as word choice or appeals to emotion, to add power to the ideas expressed

Adapted from James Schlesinger, “Cold Facts on Global Warming.” ©2004 by The Los Angeles Times. Originally published January 22, 2004.

1 We live in an age in which facts and logic have a hard time competing with rhetoric—especially when the rhetoric is political alarmism over global warming.

2 We continue to hear that “the science is settled” in the global warming debate, that we know enough to take significant action to counter it. Those who hold this view believe emissions of carbon dioxide are the primary cause of any change in global temperature and inevitably will lead to serious environmental harm in the decades ahead.

3 In 1997, for instance, Vice President Al Gore played a leading role in the negotiation of the Kyoto Protocol, the international agreement to deal with the fears about global warming. He was willing to embrace severe reductions in U.S. emissions, even though the Clinton administration”s own Department of Energy estimated that Kyoto-like restrictions could cost $300 billion annually. Then, when it became clear that the Senate would not agree to a treaty that would harm the economy and exempt developing countries like China and India, the Clinton administration did not forward it for ratification. Since then, the treaty”s flaws have become more evident, and too few countries have ratified it to allow it to “enter into force.”

4 The Bush administration, as an alternative to such energy-suppressing measures, has focused on filling gaps in our state of knowledge, promoting the development of new technology, encouraging voluntary programs and working with other nations on controlling the growth of greenhouse gas emissions. Collectively, these actions involve spending more than $4 billion annually, and the U.S. is doing more than any other nation to address the climate-change issue.

5 Of these efforts, filling the gaps in our knowledge may be the most important. What we know for sure is quite limited. For example, we know that since the early 1900s, the Earth”s surface temperature has risen about 1 degree Fahrenheit. We also know that carbon dioxide, a greenhouse gas, has been increasing in the atmosphere. And we know that the theory that increasing concentrations of greenhouse gases like carbon dioxide will lead to further warming is at least an oversimplification. It is inconsistent with the fact that satellite measurements over 35 years show no significant warming in the lower atmosphere, which is an essential part of the global-warming theory.

6 Much of the warming in the 20th century happened from 1900 to 1940. That warming was followed by atmospheric cooling from 1940 to around 1975. During that period, frost damaged crops in the Midwest during summer months, and glaciers in Europe advanced. This happened despite the rise in greenhouse gases. These facts, too, are not in dispute.

7 And that”s just our recent past. Taking a longer view of climate history deepens our perspective. For example, during what”s known as the Climatic Optimum of the early Middle Ages, the Earth”s temperatures were 1 to 2 degrees warmer than they are today. That period was succeeded by the Little Ice Age, which lasted until the early 19th century. Neither of these climate periods had anything to do with man-made greenhouse gases.

8 The lessons of our recent history and of this longer history are clear: It is not possible to know now how much of the warming over the last 100 or so years was caused by human activities and how much was because of natural forces. Acknowledging that we know too little about a system as complicated as the planet”s climate is not a sign of neglect by policymakers or the scientific community. Indeed, admitting that there is much we do not know is the first step to greater understanding.

9 Meanwhile, it is important that we not be unduly influenced by political rhetoric and scare tactics. Wise policy involves a continued emphasis on science, technology, engagement of the business community on voluntary programs and balancing actions with knowledge and economic priorities. As a nation, by focusing on these priorities, we show leadership and concern about the well-being of this generation and the ones to follow.

Write an essay in which you explain how James Schlesinger builds an argument to persuade his audience that the debate on global warming is unduly influenced by political alarmism. In your essay, analyze how Schlesinger uses one or more of the features listed in the box above (or features of your own choice) to strengthen the logic and persuasiveness of his argument. Be sure that your analysis focuses on the most relevant features of the passage.

Your essay should NOT explain whether you agree with Schlesinger”s claims, but rather explain how Schlesinger builds an argument to persuade his audience.

SAT PRACTICE TEST 2 ANSWER KEY

Section 1: Reading

1 . A

2 . D

3 . C

4 . D

5 . B

6 . B

7 . A

8 . B

9 . C

10 . C

11 . A

12 . A

13 . C

14 . A

15 . D

16 . C

17 . B

18 . C

19 . D

20 . D

21 . A

22 . C

23 . B

24 . D

25 . A

26 . D

27 . D

28 . C

29 . A

30 . C

31 . B

32 . D

33 . C

34 . C

35 . B

36 . A

37 . D

38 . B

39 . C

40 . D

41 . B

42 . A

43 . B

44 . B

45 . A

46 . A

47 . B

48 . A

49 . D

50 . C

51 . D

52 . D

Total Reading Points (section 1)

Section 2: Writing and Language

1 . A

2 . C

3 . B

4 . C

5 . B

6 . A

7 . D

8 . C

9 . B

10 . B

11 . D

12 . D

13 . A

14 . B

15 . C

16 . B

17 . D

18 . D

19 . C

20 . B

21 . D

22 . A

23 . C

24 . D

25 . C

26 . A

27 . B

28 . D

29 . A

30 . D

31 . A

32 . A

33 . C

34 . C

35 . D

36 . B

37 . B

38 . A

39 . D

40 . C

41 . A

42 . B

43 . D

44 . C

Total Writing and Language Points (section 2)

Section 3: Math (No Calculator)

1 . B

2 . B

3 . C

4 . C

5 . B

6 . A

7 . A

8 . A

9 . B

10 . B

11 . D

12 . D

13 . B

14 . C

15 . D

-------

16 . 8

17 . 2/3 or.666 or. 667

18 . 1.2 or 6/5

19 . 2.5 or 5/2

20 . 7/3 or 2.33

Total Math Points (Section 3)

Section 4: Math (Calculator)

1 . B

2 . B

3 . A

4 . C

5 . D

6 . A

7 . A

8 . A

9 . A

10 . B

11 . D

12 . B

13 . C

14 . D

15 . A

16 . C

17 . B

18 . B

19 . C

20 . C

21 . A

22 . D

23 . C

24 . D

25 . B

26 . C

27 . C

28 . D

29 . B

30 . A

-----------------

31 168

32 . 3/2 or 1.5

33 . 52

34 . 25

35 . 4/45 or. 088 or. 089

36 . 75/4 or 18.75

37 . 47

38 . 32

Total Math Points (Section 4)

SCORE CONVERSION TABLE

Scoring Your Test

  1. Use the answer key to mark your responses on each section.
  2. Total the number of correct responses for each section:
  3. Add the raw scores for sections 3 and 4. This is yourMath Raw Score :_____________
  4. Use theTable 1 to calculate your Scaled Test and Section Scores (10–40) .
  5. Add theReading Test Scaled Score and the Writing and Language Test Scaled Score and multiply this sum by 10 to get your Reading and Writing Test Section Score (20–80) .

Table 1: Scaled Section and Test Scores (10–40)

SAT PRACTICE TEST 2 DETAILED ANSWER KEY

Section 1: Reading

1 . A

Tone

In the first paragraph, the author tells us that he has little prudence and no skill in inventing of means and methods … in adroit steering … nor in gentle repairing . He also has no skill to make money spend well . These are self-effacing descriptions. They are certainly not pontifical (speaking as a high priest), aspirational (expressing high hopes and goals), or sardonic (grimly cynical).

2 . D

Inference

The statement that whoever sees my garden discovers that I must have some other garden is the last statement in the author”s list of his personal inadequacies. Therefore, this statement must be taken to be self-effacing as the other statements are, and specifically to mean that he lacks gardening skill.

3 . C

Word in Context

Recall that the first paragraph begins with the question What right have I to write on prudence … ? The second provides a response to this question about his right : in saying I have the same title to write on prudence as I have to write on poetry or holiness , then, he is clearly saying that he has the standing or authority to write on prudence.

4 . D

Inference

In following his declaration that he has the right to write on prudence (lines 11–12), Emerson states that [w]e write from aspiration as well as from experience . In other words, we gain the standing to write on prudence not only from expertise in prudent behavior, but also from a focused yearning .

5 . B

Textual Evidence

As the explanation to the previous question indicates, the best support for this answer is in lines 13–14.

6 . B

Specific Purpose

The sixth paragraph (lines 39–57) discusses three classes of people according to their proficiency of knowledge of the world (lines 39–40). The first class values health and wealth [as] a final good (lines 42–43). The second class values the beauty of the symbol (line 46–47). The third class lives above the beauty of the symbol to the beauty of the thing signified (lines 46–47). This last group has spiritual perception (line 49). Therefore, its members are superior for their ability to discern sublime qualities .

7 . A

Interpretation

This phrase appears in a discussion of the individual who traverses the whole scale (line 50–51), that is, who has the skills of all three classes: practicality, taste, and spiritual perception. In saying that such a person does not offer to build houses and barns (lines 54) on the sacred volcanic isle of nature (lines 53–54), Emerson is saying that nature is merely a symbol that points to the splendor of God (55), and therefore not what a truly wise person chooses to fix his or her gaze upon. In other words, the building of houses and barns is an unwise allegiance to worldly things .

8 . B

Word in Context

In saying that the world is filled with the proverbs and acts of a base prudence (lines 58–59), Emerson means that most of our actions and words are devoted to practical things, like the question will it bake bread (lines 64)? As Emerson made clear in his previous paragraph, these considerations are those of the lowest and least noble class, so theirs is an ignoble prudence.

9 . C

Interpretation

As a whole, this paragraph discusses the problem that the world is filled with the proverbs and acts of a base prudence (lines 58–59), in other words, that our words and actions are too focused on a devotion to matter (lines 59–60) and its effect on our senses, as if we possessed no other faculties than the palate, the nose, the eye and ear (lines 60–61). Emerson describes this problem with a simile: this is a disease like a thickening of the skin until the vital organs are destroyed (lines 64–66). To Emerson, then, the disease is the problem of sensuousness (devotion to the senses rather than the intellect).

10 . C

Characterization

In line 20, Emerson defines prudence as the virtue of the senses , but he regards the world of the senses [as] a world of shows (lines 22–23), that is false when detached (line 35) from the thing signified (line 47) by the natural, sensory, intellectual world, that is, from the splendor of God (lines 55). Furthermore, he says that prudence is a devotion to matter, as if we possessed no other faculties than the palate, the nose, the touch, the eye and ear (lines 59–61). Therefore, as a whole, the passage characterizes prudence as a pursuit of practical skills and sensory experience .

11 . A

Tone

The opening paragraph describes this breathless pause at the threshold of a long passage (lines 8–9) in which the narrator and his crew seemed to be measuring our fitness for a long and arduous enterprise (lines 9–10). This describes the reflective anticipation of a journey. Notice that this description provides no evidence of anxiety or excitement . In fact, the scene is described in peaceful terms, with the ship very still in an immense stillness (line 2).

12 . A

Specific Purpose

The narrator states that some glare in the air (lines 14–15) prevented him from seeing sooner something that did away with the solemnity of perfect solitude (lines 18–19). That is, he saw something that led him to believe they were not alone. In the next paragraph, this something is revealed to be the mastheads of a ship anchored inside the islands (lines 35–36).

13 . C

Specific Detail

This sentence describes the scene as the narrator surveys the tide of darkness and a swarm of stars (lines 20–21) while resting his hand on the rail of the ship as if it were the shoulder of a trusted friend (line 24). In the next sentence, he describes this as a moment of quiet communion (line 26) with the ship, now interrupted by the sight of a strange ship beyond and the disturbing sounds (lines 27–28) being made by the crew. In other words, this sentence describes a moment of wistful (expressing vague longing) contemplation . Choice (A) is incorrect because, although the disturbing sounds and the omen of a distant ship may seem to be signs of impending danger , the sentence in lines 20–24 makes no mention of these things. Choice (B) is incorrect, because this moment is described as a moment of quiet communion , not deep inner turmoil . Choice (D) is incorrect, because there is no mention of any tragic experience .

14 . A

Characterization

Since this story is being told from the perspective of the captain, we can infer his character from the nature of his narration. In the opening paragraph, the captain states that we seemed to be measuring our fitness for a long arduous enterprise, the point of our existences to be carried out (lines 9–12), demonstrating that he is more reflective than reactive as a leader. Much later he says, what I felt most was my being a stranger to the ship; and if all the truth must be told, I was somewhat of a stranger to myself … I wondered how far I should turn out faithful to that ideal conception of one”s own personality every man sets up for himself secretly (lines 56–65). These descriptions of reflection and self-doubt reveal the captain as being self-conscious and diffident .

15 . D

Textual Evidence

As the explanation to question 14 shows, the best evidence for this answer can be found in lines 63–65.

16 . C

Word in Context

In saying I mention this because it has some bearing on what is to follow (lines 54–55), the narrator means that the fact that he was the only stranger on board (line 54) is relevant to what he is about to say.

17 . B

Word in Context

This sentence describes how the chief mate, described as earnest (line 70) and painstaking (71), is trying strenuously to figure out why there is another ship anchored nearby. In saying that he was trying to evolve a theory , the narrator means he is pondering (thinking) strenuously .

18 . C

Specific Detail

The truth that the narrator mentions in line 57 is the fact that I am a stranger to myself . He later goes on to explain what he means by this: I wondered how far I should turn out faithful to that ideal conception of one”s own personality every man sets up for himself secretly (lines 63–65). In other words, this truth is the fact that he lacks self-confidence .

19 . D

Word in Context

In saying that the why and the wherefore of that scorpion … had exercised him infinitely (lines 75–81), the narrator means that the chief mate was using his dominant trait … [of] earnest consideration (lines 69–71) to figure out how a scorpion had made its way into his cabin. That is, the questions about the scorpion had disquieted (unsettled) him infinitely .

20 . D

Textual Evidence

The collaboration on the part of [the chief mate”s] round eyes and frightful whiskers (lines 67–68) describes his facial contortions as he deliberates about the anchored ship. In other words, it is an act of strained contemplation .

21 . A

Interpretation

In the final line, the narrator says that the ship within the islands was much more easily accounted for . In other words, the scorpion was less easily accounted for, or less explicable .

22 . C

Inference

The second paragraph discusses the “outsourcing” [of] the creation of human life (lines 8–9), so the design and control mentioned in line 13 refer specifically to the design and control of the process of conception .

23 . B

Tone

The author of Passage 1 states that the “outsourcing” [of] the creation of human life … mocks the profundity of life (lines 8–12) and he provides no indication in the passage that he otherwise approves of it. Clearly, then, he regards it with blunt disdain .

24 . D

Cross-Textual Analysis

Both of these quotations represent viewpoints with which the authors of the respective passages disagree. In Passage 1, the quotation “Sorry, but I think I can do better” (line 15) is from those who turn [their] noses up at the most precious gift in the universe (lines 13–14) much to the chagrin of the author. In Passage 2, the quotation “But you are playing God” (line 78) is described as the cry of all whose power is threatened by the march of human progress , and with whom the author clearly disagrees.

25 . A

Cross-Textual Analysis

Jeremy Rifkin is described in Passage 2 as a cloning foe (line 63) who is quoted as saying “It”s a horrendous crime to make a Xerox of someone. You”re putting a human into a genetic straitjacket. ” Presumably, then, he would agree that our attitude toward the creation of life must be one of humility (line 7).

26 . D

Graphical Analysis

The illustration shows a schematic overview of some Therapeutic Cloning Strategies that involve removing a somatic cell from a patient and transferring its nucleus to stem cells that can then be cultured into genetically matched tissue that can then replace diseased cells and tissues in the patient. This is an example of one of the procedures to clone human cells for seemingly beneficial purposes (lines 38–40) described in Passage 1. Choice (A) is incorrect because the guided purpose refers to a principle of creationism, which is not indicated at all in the diagram. Choice (B) is incorrect because, although the process in the diagram might resemble an assembly line, it is not the assembly line that could be used to create a child that is no longer uniquely human (lines 23–24), but with carefully designed and tested features (line 25). Choice (C) is incorrect because the diagram does not describe the course of human evolution , which would need to show how humans evolved from more primitive species.

27 . D

Specific Meaning

The process of introducing degraded germs (lines 52–53) describes the basic process of vaccination, which, like cutting flesh (line 52) (that is, surgery), must have seemed dangerous at first, but in fact can be a life-saving technology. This process is the injection of vaccines.

28 . C

Inference

In this paragraph, the author of Passage 2 describes the position of cloning foes who believe that cloning is the end of evolution , or at least the beginning of its corporate management (lines 59–61). The author of Passage 1 is deeply concerned that the executive boards of these [cloning] companies will decide the course of human evolution, with more concern for quarterly profit reports than for the sake of humanity (lines 32–35). Clearly, then, the author of Passage 1 regards this management as a regrettable invasion of commercial interests into human reproduction .

29 . A

Specific Purpose

Jeremy Rifkin”s belief that cloning is a horrendous crime (line 64) directly contradicts the thesis of Passage 2, which is that cloning and similar technologies can provide ample food for a starving world, cure devastating illnesses, and replace diseased organs (lines 98–100). Therefore, to the author of Passage 2, Rifkin”s opinion exemplifies an untenable (indefensible) position . Choice (B) may seem plausible, since Rifkin is warning of the potential dangers of cloning , but notice that this cannot be the reason that the author of Passage 2 quotes Rifkin, because the passage clearly disagrees with his sentiments.

30 . C

Specific Purpose

The author of Passage 2 mentions the Twins Days Festival (line 68) in order to demonstrate the absurdity of Jeremy Rifkin”s statement that creating a genetic Xerox of a person is a horrendous crime (line 64). To the author of Passage 2, then, the Twins Days Festival represents the innocuousness (harmlessness) of genetic duplication , since twins are genetic duplicates, and nothing to be feared.

31 . B

Cross-Textual Inference

The author of Passage 2 does not object to the procedures to clone human cells for seemingly beneficial purposes (lines 38–40), and in fact believes they are necessary contributions to medical progress since they potentially provide technologies to provide ample food for a starving world, cure devastating illnesses, and replace diseased organs (lines 98–100).

32 . D

Textual Evidence

As the explanation to question 31 indicates, the best evidence for this answer is found in lines 95–100.

33 . C

General Purpose

The first paragraph establishes that this passage is focused on the specific processes involved in children”s acquisition of language (line 12). Therefore, the passage is primarily concerned with exploring academic questions about how we learn language . Choice (A) is incorrect because the passage does not begin to delineate the general principles of linguistics , which is a far greater subject than simply language acquisition. Choice (B) is incorrect, because although the passage does refer to children”s ability to acquire diverse languages like English … Bantu or Vietnamese (lines 63–64), it does not compare their structural qualities. Choice (D) is incorrect because, although the passage does discuss the ideas of the influential linguists Benjamin Whorf (in the second paragraph) and Noam Chomsky (in the last paragraph), these references only serve the larger purpose of exploring the questions of language acquisition, and do not serve as the overall focus of the passage.

34 . C

Inference

In the first paragraph, the author indicates that [e]very time we speak we are revealing something about language, so the facts of language structure are easy to come by (lines 3–6). Therefore, the data mentioned in line 6 are the facts of language structure , which would likely include the syntax (rules governing word order) of different languages . Choice (A) is incorrect because information about literacy levels is not information about language structure . Choice (B) is incorrect because methods of teaching are not facts of language structure . Choice (D) is incorrect because, although the passage does mention the innate structure (line 59) of the brain a few paragraphs later, this is clearly not what line 6 is referring to.

35 . B

Inference

The phrase the two (line 2) refers to two nouns in the previous clause: language and thoughts , in other words, thinking and expressing .

36 . A

Word in Context

The author uses the phrase sticking communicable labels on thoughts (lines 15–16) to describe one particularly simplistic theory about the language acquisition. The author is using the metaphor of applying name tags or labels to describe one way of describing how words are used. Choice (B) is incorrect because upholding refers to a process of confirming an official claim or pronouncement. Choice (C) is incorrect because, although sticking (as with a needle) can mean piercing , this reference clearly does not imply any act of puncturing. Choice (D) is incorrect because this phrase describes an act of acquisition , that is, learning something new, rather than maintaining something old.

37 . D

Tone/Attitude

After describing Benjamin Whorf”s theory, the author then states that virtually all modern cognitive scientists believe it is false (lines 25–26). The author”s ensuing discussion makes it clear that he agrees with these cognitive scientists. That is, he is antagonistic toward Whorf”s hypothesis. Choice (A) is wrong because the author does not dismiss Whorf”s hypothesis, but rather regards it as an intriguing hypothesis which just happens to be incorrect. (To dismiss an idea is to believe it is not even worthy of consideration, not merely to reject it after consideration.) Choice (B) is clearly wrong because the author does not support Whorf”s hypothesis. Choice (C) is wrong because the author does not have any conflicting feelings about the hypothesis.

38 . B

Interpretation

The author states that babies can think before they can talk (line 27) in order to refute Whorf”s hypothesis that we can”t think in terms of categories and relations (line 19) until our language gives us the words to do so. Whorf believes that language precedes thought. The author of this passage is saying the opposite: that skills associated with basic reasoning are not dependent on verbal communication .

39 . C

Thesis

The author”s view on human language acquisition can be found in lines 95–97: language acquisition depends on an innate, species-specific module that is distinct from general intelligence . This module must have an intricate innate structure (line 59) in order to acquire a language that is itself intricately complex (line 55). Choice (A) is incorrect because it represents the Whorf hypothesis, which the author explicitly rejects. Choice (B) is incorrect because the author does not state that the structures for learning language are simple. Choice (D) is incorrect because the author places more emphasis on the innate structure in the brain that enables language acquisition than he does on environmental input.

40 . D

Textual Evidence

As the explanation to question 39 indicates, the best evidence for this answer is found in lines 95–97.

41 . B

Interpretation

Lines 58–64 discuss the author”s belief that the innate structure in the brain dedicated to language acquisition cannot be either too simple or too complex. This kind of structure refers to the functional organization of the mind . Notice that the structure being discussed here is not the same as the structure mentioned in line 5, which refers to the structure of language itself.

42 . A

Inference

The author states that, in 1959, Anglo-American natural science, social science, and philosophy had come to a virtual consensus about the answers to the questions listed above (lines 78–81), that is, the questions listed in lines 14–17: Is language simply grafted on top of cognition as a way of sticking communicable labels on thoughts ? Or does learning a language somehow mean learning to think in that language ? The consensus on these topics was that language must be learned; it cannot be a module; and thinking must be a form of verbal behavior (lines 85–87) Therefore, the disciplines accepted the hypothesis that cognition depends on verbal skills .

43 . B

General Purpose

The passage as a whole describes the history and scientific underpinnings of the medical technique of vaccination. Choice (A) is incorrect, because the passage does not discuss the various means by which the human immune system works, but only the particular method in which vaccination “tricks” our immune system into fighting particularly virulent infections that it could not fight on its own. Choice (C) is incorrect because, although the passage does discuss some medical practices of cultures like the Chinese and the Turks (lines 7–8), as well as techniques develop by German, British, and French scientists, the passage as a whole is not concerned with cross-cultural comparisons of medical practices. Choice (D) is incorrect because the passage does not focus on the controversies surrounding vaccination, but rather the science behind and history of the technique.

44 . B

Interpretation

The Chinese and the Turks (lines 7–8) are mentioned as civilizations that produced a medicine against smallpox by grinding up the scabs of people with mild cases of the disease (lines 8–10). This process closely resembles the same process currently used in vaccinations, that is, infecting patients with mild forms of the disease agent.

45 . A

Detail

From lines 23–25: Could it have been a bacterium ? In Germany, in 1882, Robert Koch had shown that just such a germ caused tuberculosis . In other words, tuberculosis is cause by a bacterium, not a virus.

46 . A

Inference

In the first paragraph, the passage states that Pasteur produced a rabies vaccine without actually realizing that he was enhancing the body”s own immune system; he knew only that the vaccine worked (lines 18–21). Therefore, some effective remedies for infectious disease were used before their mechanisms were understood .

47 . B

Textual Evidence

As the explanation to the previous question indicates, the evidence for this answer is in lines 18–21.

48 . A

Inference

The third paragraph states that a virus must attach itself to a cell, impregnate the cell with the viral genes, and then, parasite that it is, turn that cell into a reproductive machine for the virus”s benefit (lines 38–42). That is, it invades another organism and exploits its reproductive process. The cowbird-phoebe relationship as described in (A) is the closest to this relationship.

49 . D

Inference

The passage indicates that both viruses and bacteria can be parasitic (that is, harmful to their hosts without providing any compensatory benefit). Viruses can cause measles, chicken pox, polio, herpes, and whooping cough, and bacteria can cause tuberculosis (line 25) as well as much human misery (line 28). Choice (A) is incorrect because the passage does not discuss any potential benefit of bacteria. Choice (B) is incorrect because the passage does not discuss the harshness of the conditions under which bacteria can live. Choice (C) is incorrect because bacteria were not to play the starring role in the vaccine story (lines 28–29).

50 . C

Word in Context

This sentence says that the body, for the most part, is able to recognize these viruses as foreign invaders by the signature proteins on their surface (lines 42–46), in other words, the viruses have unique proteins that enable the body”s immune system to identify and attack them. That is, these are distinctive proteins that enable the immune systems to distinguish them from other proteins.

51 . D

Structure and Purpose

The final paragraph begins by saying that although advances against viruses continue (line 68) … herpes, another viral affliction, still flourishes, and the most ubiquitous of all the viral maladies—the common cold … may never be thwarted (lines 71–75). This contrasts sharply with the triumphant tone of the previous paragraph, which said that thanks to advances in modern vaccines, measles are nearly gone, and chicken pox, whooping cough, typhoid, and cholera are under control (lines 56–58). Therefore, the final paragraph qualifies (makes less absolute) the tone of the previous paragraph .

52 . D

Graphical Inference

The two graphs in Figure 1 and Figure 2 show the incidence of measles infections in the United States as well as in England and Wales from 1950 to 2000. The measles vaccine was introduced five years later in England and Wales than it was in the United States, but in both cases the decline in incidence occurred almost immediately. Therefore, the two graphs are presented together in order to illustrate the effects of the measles vaccine administered to comparable populations but at different times .

Section 2: Writing and Language

1 . A

Idiom

The original phrasing is best. Choice (B) is incorrect because choice with getting is not idiomatic. Choice (C) is incorrect because choice of the way is not idiomatic. Choice (D) is incorrect choice of getting , although idiomatic conveys an illogical idea in this context.

2 . C

Diction

Here we are asked to choose the best word to convey the appropriate idea in this sentence. The sentence indicates that we might be surprised to learn something about the world of journalism, and hence that most of us are not as informed about the world of journalism as we could be. In other words, we are not particularly savvy (knowledgeable) about the world of journalism. Apt = suitable to the circumstances; acute = sharp; comprehensive = complete.

3 . B

Diction/Logic

In the original phrasing, the pronoun which is illogical, since it refers to the media : that is, saying the media is 90% does not make sense. Choice (C) is incorrect because the phrase all 90% of it is illogical: all of it means 100% of it. Choice (D) is incorrect because it is both unidiomatic and illogical.

4 . C

Possessive Form/Pronoun Agreement

In the original phrasing, the pronoun their disagrees with its antecedent corporation , which is singular. Recall that the possessive form of the pronoun it is its (it”s = it is) . The only choice that avoids both the agreement error and the diction error is (C).

5 . B

Parallelism

This phrase should be parallel to the subject-verb pair in the previous sentence, Some argue . The only choice with a parallel verb form is (B).

6 . A

Data Analysis

The original phrasing is the only option that represents the data in Figure 1 accurately. Since the second circle graph represents all broadcast television media and its ownership, it indicates that 3.2% (0.6% + 1.3% + 0.9% + 0.4%) of American broadcast television outlets were controlled by minorities.

7 . D

Idiom

The idiomatic form of this phrase is antidote to .

8 . C

Clear Expression/Pronoun Antecedents

In choices (A), (B), and (D), the pronoun it lacks any clear referent. The only choice that avoids this problem is (C).

9 . B

Cohesiveness/Purpose

Examples that contrast civilized debate would have to be examples of uncivilized debate. Gossip and fear-mongering certainly qualify as relatively uncivilized and unsophisticated forms of discourse.

10 . B

Idiom, Pronoun-Antecedent Agreement

Here, we are looking for the most appropriate logical transition from the previous paragraph to the new one. The last sentence of the previous paragraph gave examples of in-depth, nonprofit, public-supported journalism that is less influenced by any corporate or political agenda . The new paragraph, however, begins with a discussion of sensationalism and how it sells , which provides a stark contrast to the previous paragraph. This requires a contrasting coordinator, such as nevertheless or still .

11 . D

Diction/Clear Expression of Ideas

We want a word to represent the websites like ProPublica and NPR, as mentioned in the previous paragraph, that engage in relatively noncorporate and apolitical journalism. The phrase instances of journalism indicates specific articles or broadcasts, rather than the organizations themselves. The phrase patterns of journalism indicates trends in those articles or broadcasts, rather than the organizations themselves. The phrase receptacles of journalismindicates containers that receive journalism rather than organizations that produce it. Only (D) repositories of journalism provides a phrase that refers to the organizations themselves.

12 . D

Coordination/Verb Tense

The original phrasing creates a sentence fragment rather than an independent clause. Choice (B) is incorrect because it commits the same error. Choices (C) and (D) both form independent and idiomatic clauses, but choice (C) is incorrect because the sentence is clearly making a claim about the current state of being of these rituals, rather than the current status-as-consequence of these rituals, so the present perfect (or “present consequential”) form is not appropriate. (For more on using the “perfect” or “consequential” aspect, see Chapter 4 , Lesson 23.)

13 . A

Coordination/Conjunctions

The original phrasing is best. Choice (B) is incorrect because the second clause does not explain the first. Choice (C) is incorrect because the second clause does not follow as a consequence of the first. Choice (D) is incorrect because there is no tonal or semantic contrast between the clauses.

14 . B

Diction/Clear Expression of Ideas

The phrase collide against is not idiomatic: collide with is the correct idiom, although this phrase would imply more of a physical relationship than the sentence intends. Since the sentence indicates a conflict between an event and a belief so the verb should express a relationship between ideas , rather than objects . Of the choices, only (B) contradict serves this purpose effectively.

15 . C

Coordination

In this sentence, the conjunction and establishes the relationship between the coordinate independent clauses, so any subordinating conjunction like since, so , or because is inappropriate.

16 . B

Diction, Agreement

Colons must always follow independent clauses, so choices (A) and (D) are incorrect. This phrase must provide a definition of the term “agency,” which is precisely what choice (B) that is, intentional action does. Choice (C) is incorrect because it categorizes rather than defines.

17 . D

Parallelism

This sentence presents a list of present tense verbs: design … compose … and formulate . The original phrasing is incorrect because it reinserts the auxiliary can , which breaks the parallel structure of the list. Only choice (D) maintains this parallel form.

18 . D

Cohesiveness

This sentence belongs before sentence 5, because it provides a parallel idea to the one presented in sentence 4. Sentence 4 states that Many of these [superstitions] are harmless if not quaint , so the next sentence should provide a transition to some of the less pleasant aspects of superstitious thinking.

19 . C

Pronoun-Antecedent Agreement/Verb Mood

In the original phrasing, the pronoun it does not agree with its plural antecedent explanations ; therefore, choices (A) and (B) are incorrect. Choice (D) is incorrect because the auxiliary would implies necessity, rather than ability, which is illogical in this context.

20 . B

Clear Expression/Pronoun Antecedents

The original phrasing is incorrect because the two instances of the pronoun their have conflicting antecedents, and the second clause is needlessly in the passive voice. Choices (C) and (D) have similar pronoun referent problems. Only choice (B) is phrased without ambiguous pronouns.

21 . D

Diction

The previous sentence, as well as the passage as a whole, indicates that superstitious rituals are used to satisfy a craving in our brains for control .

22 . A

Cohesiveness

Only choice (A) maintains the skeptical and analytical tone toward superstitious rituals that is established in the rest of the passage.

23 . C

Clear Expression

The original phrasing is incorrect because the phrase debating a warming planet is illogical: only theories, claims , or ideas can be topics of debate. Choices (B) and (D) are incorrect because the clause if we should lacks a logical object.

24 . D

Comma Splices/Coordination

The original phrasing is incorrect because it creates a comma splice. Two independent clauses may not be joined by only a comma. Choice (B) is incorrect because the conjunction while is illogical. Choice (C) is incorrect because it forms a noun phrase, which does not coordinate with any part of the main clause. Choice (D) creates a participial phrase that appropriately modifies the subject of the main clause.

25 . C

Clear Expression/Idiom

In the original phrasing, the verb provide is used illogically and the phrase cost of the American taxpayer is unidiomatic. Choice (B) is incorrect because needless in costing is unidiomatic. Choice (D) is incorrect because the verb isdisagrees in number with its subject attempts .

26 . A

Diction

The original phrasing is best. The verb is serves most effectively in the role of defining science .

27 . B

Parallelism

The underlined phrase is the third item in a parallel list: identifying … gathering … and finding . Choice (B) best maintains this parallel structure without introducing any other error. Choice (D) provides a parallel form, but the phrase way for explaining is unidiomatic.

28 . D

Parallelism

The underlined phrase is part of a parallel list: that the earth … that the effects … that there are things … and that many of these things … Only choice (D) maintains this parallel structure.

29 . A

Logical Cohesiveness

The original phrasing best, since the passage is about eliminating politics and ideology from discussions about climate change. The other choices insert points of advocacy that conflict with the tone and purpose of the passage as a whole.

30 . D

Clear Expression of Ideas/Verb Mood/Verb

Tense

The original phrasing includes an illogical core: the fact … isn”t the same as it being true . Choice (B) is incorrect because a statement of general fact should not be in the future tense. Choice (C) is incorrect because a statement of general fact should not be in the subjunctive mood. Choice (D) uses the idiom make it so logically and grammatically.

31 . A

Parallelism

The original phrasing is the only option that completes the parallel construction caused not by germs … but by demons .

32 . A

Verb Form/Clear Expression

The original phrasing is the most logical and concise.

33 . C

Logical Coherence

The adverb furthermore indicates that this sentence is extending a line of reasoning. Since it clearly follows the parallel clauses of sentence 2, When we … become … we become … and therefore it most logically follows sentence 2 but precedes sentence 3.

34 . C

Pronoun Agreement/Verb Aspect

The original phrasing is incorrect because the pronoun they disagrees in number with the antecedent Bohemia . Choice (B) is incorrect because it produces a comma splice. Choice (D) is illogical because the use of the present perfect participle having been improperly implies a consequence.

35 . D

Diction

The original phrasing is illogical because a country cannot regard (consider in a particular way; concern) anything. This verb must show a relationship between a particular country and a particular geographical region. Only choice (D) comprises (makes up) expresses this relationship in a logical way.

36 . B

Coordination

The original phrasing is incorrect because it creates a dangling participle: the participle designating does not share its subject with the main clause. Choice (C) is incorrect because it creates a comma splice. Choice (D) is incorrect because it also creates a dangling participle. Choice (B) is best because it avoids both the comma splice and dangling participle.

37 . B

Verb Tense/Verb Aspect

The phrase by 1843 indicates that the status of the term Bohemian had become established prior to that point in time. Therefore, the verb requires the past perfect or past consequential form: had come to mean . Although choice (D) is a verb in the past consequential form, it incorrectly implies that the term no longer had that particular meaning in 1843.

38 . A

Coordinating Modifiers

This sentence is trying to convey the fact that [La Vie de Bohème] proved [to be so] extraordinarily successful that the stories themselves were published . This requires the active voice, so choices (B) and (C) are incorrect. Choice (D) is incorrect because it is unidiomatic.

39 . D

Number Shift

The original phrasing is incorrect because of the number shift between it and elements . Choices (B) and (C) are needlessly wordy. Choice (D) is clear and concise.

40 . C

Diction/Logical Coherence

This sentence indicates the effect that increased commodity production had on the commercial middle class . It is illogical to say that the middle class propagated (was transmitted), propitiated (won the favor of someone), or preempted (took action to prevent something) as a result of this increased production. It is, however, logical to say that the middle class prospered (flourished) as a result of it.

41 . A

Coordination of Clauses

The original phrasing best coordinates the two related, but independent, clauses. Choice (B) produces a run-on sentence with a comma splice. Choice (C) is illogical and unidiomatic. Choice (D) is illogical and misuses the semicolon.

42 . B

Clarity of Expression/Parallelism

Choice (B) provides the most parallel comparison: the production of art was in fact less important than the capacity for art .

43 . D

Coordination of Ideas

The use of the adverb thus indicates that this sentence represents a logical consequence of some particular state of affairs. That state of affairs is best indicated by sentence 5: Most of its ingredients had existed in Paris for at least two decades before he started writing . This explains why Murger can be described as a Bohemian of the second generation .

44 . C

Coherence/Meaning

This question is essentially asking us to describe the function of Claretie”s quotation. Since it refers to a poisonous element in the French character , it is clearly indicating a dire assessment of France”s national temperament .

Section 3: Math (No Calculator)

1 . B

Algebra (solving equations) EASY

Original equation:

2b – 1 = 5

Add 1:

2b = 6

Divide by 2:

b = 3

Substitute b = 3 into 2b 2 – 1:

2b 2 – 1 = 2(3)2 – 1 = 18 – 1 = 17

2 . B

Special Topics (circles) EASY

Marking up the diagram with the given information, as shown, shows that three of the smaller radii make up one larger radius. Therefore, the radius of each small circle is 6/3 = 2.

3 . C

Algebra (word problems/fractions) EASY

If 1/5 of her term paper is 15 pages, then the entire paper must be 15 × 5 = 75 pages long. This means she has 75 – 15 = 60 more pages to edit.

4 . C

Advanced Mathematics (functions and sequences) EASY

Notice that the rule in choice (C) generates the entire sequence: 7 (times 2 minus 2 equals) 12 (times 2 minus 2 equals) 22 (times 2 minus 2 equals) 42 (times 2 minus 2 equals) 82.

5 . B

Special Topics (three dimensional geometry) MEDIUM

Notice that the question asks us for the longest length of a diagonal on one of the faces of the box, and that there are three different rectangles as faces: a 3 × 4 rectangle, a 3 × 5 rectangle, and a 4 × 5 rectangle. Clearly the one with the two greatest dimensions will have the longest diagonal, which we can find using the Pythagorean Theorem.

42 + 52 = d 2

Simplify:

16 + 25 = d 2

Simplify:

41 = d 2

Take the square root:

6 . A

Algebra (linear equations) MEDIUM

We can test each point to find the one that does NOT satisfy the equation.

(A) –2(–9) – 3(6) = 18 – 18 = 0 ≠ 36

(B) –2(–24) – 3(4) = 48 – 12 = 36

(C) –2(6) – 3(–16) = –12 + 48 = 36

(D) –2(12) – 3(–20) = –24 + 60 = 36

Therefore, the correct answer is (A).

7 . A

Advanced Mathematics (parabolas) MEDIUM

The initial population, I , is the population when the time is 0. Therefore, I = 250(1.32)0 = 250(1) = 250. The annual percent increase in the population, r , can be calculated by finding the population at t = 1 and then calculating the percent change from the initial population. If t = 1, P = 250(1.32)1 . As we discussed in Chapter 8 , Lesson 7, multiplying a quantity by 1.32 is equivalent to increasing a number by 32% (that is, 1.32 = 100% + 32%), so r = 32%.

8 . A

Advanced Mathematics (rational expressions) MEDIUM

3 + 1

Multiply the numerator and denominator by the conjugate ( ):

Simplify:

Simplify:

Distribute:

9 . B

Algebra (linear relationships) MEDIUM

As we discussed in Chapter 7 , Lesson 5, a line in the form ax + by = c has a slope of –a/b . Therefore, the line 3x + 6y = 0 has a slope of –3/6 = –1/2. Recall, also, from Chapter 7 , Lesson 7, that perpendicular lines have slopes that are opposite reciprocals. Therefore, the line we are looking for must have a slope of 2. You might draw a quick sketch of the xy -plane and plot the points given in each choice to find the line that has a slope of 2, or you could use the slope formula from Chapter 7 , Lesson 5: slope = (y 2y 1 )/(x 2x 1 ).

(A) slope = (3 – 0)/(0 – (–6)) = 3/6 = 1/2

(B) slope = (–6 – 0)/(0 – 3) = –6/–3 = 2

(C) slope = (6 – 0)/(0 – 3) = 6/–3 = –2

(D) slope = (3 – 0)/(0 – 6) = 3/–6 = –1/2

The only choice that gives a slope of 2 is (B).

10 . B

Advanced Mathematics (quadratics) HARD

Perhaps the simplest way to begin this problem is to draw a quick sketch of the function in the xy -plane, and then compare this graph to the transformations of the original function given in the choices. Notice that the original function f (x ) = xx 2 is easily factored as f (x ) = x (1 – x ). The Zero Product Property (Chapter 9 , Lesson 5) tells us that this function must have zeros at x = 0 and x = 1. Notice, also, that since the coefficient of the x 2 term in the original function is negative (–1), the graph of this quadratic is an “open-down” parabola. Also, the axis of symmetry is halfway between the zeros, at x = ½. Plugging x = ½ back into the function gives us , and therefore, the vertex of the parabola is .

The question asks us to find the function that has no real zeros. This means that the graph of this function must not intersect the x -axis at all. Each answer choice indicates a different transformation of the function f . Recall from Chapter 9 , Lesson 3, that choice (A) f (x ) + ½ is the graph of f shifted up ½ unit, choice (B) f (x ) – ½ is the graph of f shifted down ½ unit, choice (C) f (x /2) is the graph of f stretched by a factor of 2 in the horizontal direction, and choice (D) f (x – ½) is the graph of f shifted right ½ unit. As the sketch above shows, only (B) yields a graph that does not intersect the x -axis.

11 . D

Advanced Mathematics (polynomials) MEDIUM-HARD

Given equation:

y = x 2 + x

Substitute :

Multiply by 4:

15 = 4x 2 + 4x

Subtract 15:

0 = 4x 2 + 4x – 15

Factor using the Product-Sum Method (Chapter 9 , Lesson 4):

0 = (2x + 5)(2x – 3)

Use Zero Product Property (Chapter 9 , Lesson 5):

2x + 5 = 0; 2x – 3 = 0

Solve each equation for x :

x = –5/2; x = 3/2

Therefore, the two points of intersection are and , and the distance between these points is

12 . D

Special Topics (complex numbers) MEDIUM-HARD

Recall from Chapter 10 , Lesson 10, that i n = 1 if and only if n is a multiple of 4. (If you need refreshing, just confirm that i 4 = 1, i 8 = 1, i 12 = 1, etc.) Therefore, if i 2k = 1, then 2k must be a multiple of 4, and therefore, k must be a multiple of 2. If k is a multiple of 2, then k /2 must be an integer. Choice (A) is incorrect, because k = 2 is a solution, but 2 is not a multiple of 4. Choice (B) is incorrect because k = –2 is a solution, and –2 is not a positive integer. Choice (C) is incorrect because k = 2 is a solution, but when 2(2) = 4 is divided by 4, the remainder is 0, not 1.

13 . B

Algebra (absolute values) MEDIUM-HARD

In order to minimize the value of |22x 2y 2 | + 22 , we must minimize the absolute value. But the least possible value of any absolute value expression is 0, so we must ask: is it possible for the expression inside the absolute value operator to equal 0? A little trial and error should reveal that it can if, for instance, x = 2 and y = 0. Notice that this gives us |2 – 22 – 02 | + 2 = |0| + 22 = 4. Since the absolute value cannot be less than 0, this must be the minimum possible value.

14 . C

Advanced Mathematics (analyzing polynomial functions) HARD

The simplest polynomial with factors of 12, (x – 5), and (x + 4) is P (x ) = 12(x – 5)(x + 4). The completely factored form (including the prime factorization of the coefficient) of this polynomial is P (x ) = (2)2 (3)(x – 5)(x + 4).

Now, using the methods we discussed in Chapter 9 , Lesson 4, we can look at the factored form of each choice:

(A) 2x 2 + 8 = 2(x 2 + 8) (x 2 + 8 is not factorable over the
reals, but it does equal

(B)

(C) 6x 26x – 120 = 6(x 2x – 20) = (2)(3)(x – 5)(x + 4)

(D) x 2 – 10x + 25 = (x – 5)(x – 5)

Notice that every polynomial in (A), (B), and (D) contains at least one factor that is NOT in the factored form of P (x ). (In (D), the factor (x – 5) appears twice, but it appears only once in P (x ).) Only choice (C) contains ONLY factors that appear in P (x ), so it is the only choice that must be a factor of P (x ).

15 . D

Advanced Mathematics (functions) HARD

Given function:

g (f (x )) = 2x + 1

Substitute f (x ) = –x + 7:

g (–x + 7) = 2x + 1

To evaluate g (2), we must let –x + 7 = 2:

x + 7 = 2

Subtract 7:

x = –5

Multiply by –1:

x = 5

Substitute x = 5:

g (–5 + 7) = 2(5) + 1

Simplify:

g (2) = 11

16 . 8

Algebra (ratios/word problems) EASY

Let x equal the number of men in the workshop. If there are half as many men as women, there must be 2x women in the workshop, or a total of x + 2x = 3x men and women in the workshop. Since this total equals 24:

3x = 24

Divide by 3:

x = 8

As with all algebra problems, make sure you confirm that the value you”ve solved for is the value the question is asking for. Since x is in fact the number of men, it is the final answer.

17 . 2/3 or .666 or .667

Advanced Mathematics (rational equations) EASY

Multiply by the common denominator, 2b :

6b – 2 = 3b

Add 2:

6b = 3b + 2

Subtract 3b :

3b = 2

Divide by 3:

18 . 1.2 or 6/5

Algebra (word problems) HARD

First, translate the given fact into an equation.

The square of a positive number is 0.24 greater than the number itself:

x 2 = x + 0.24

Subtract x and 0.24:

x 2x – 0.24 = 0

Multiply by 100 to eliminate the decimal:

100x 2 – 100x – 24 = 0

Now we factor using Product-Sum Method. Remember that the product number is ac = (100)(–24) = –2,400, and the sum number is b = –100. The two numbers with a sum of –100 and a product of –2,400 are 20 and –120.

Expand middle term using –100 = 20 – 120:

100x 2 + 20x – 120x – 24 = 0

Factor by grouping in pairs:

20x (5x + 1) – 24(5x + 1) = 0

Take out common factor:

(5x + 1)(20x – 24) = 0

Using the Zero Product Property, we see that x = –1/5 or x = 24/20 = 6/5. Since we are told that x is a positive number, x = 6/5 or 1.2.

19 . 5/2 or 2.5

Advanced Mathematics (quadratics) MEDIUM-HARD

Using the Factor Theorem from Chapter 9 , Lesson 7, we know that if a quadratic has zeroes at x = 1 and x = 5, it must have factors of (x – 1) and (x – 5). Since a quadratic can only have two linear factors, f must be of the form f (x ) = k (x – 1)(x – 5).

Substitute x = 3 and y = –2 for the coordinates of vertex:

–2 = k (3 – 1)(3 – 5)

Simplify:

–2 = k (2)(–2)

Simplify:

–2 = –4k

Divide by –4:

Therefore the equation of the function is , and we can find its y -intercept by

substituting x = 0:

Simplify:

20 . 7/3 or 2.33

Special Topics (trigonometry) HARD

The graph of the line y = mx – 4 has a slope of m and a y -intercept of –4. Since m > 0, this slope is positive. We are told that this line intersects the x -axis at an angle of θ , where cos θ = . This gives us enough information to sketch a fairly detailed graph:

Notice that this information lets us construct a right triangle that includes θ , in which the adjacent side has length 3 and the hypotenuse has length (remember cos θ = adjacent/hypotenuse). This triangle is particularly handy because it depicts the rise and the run for a portion of the line, which will enable us to find the slope. We simply have to find the rise with the

Pythagorean Theorem:

Simplify:

9 + rise 2 = 58

Subtract 9:

rise 2 = 49

Take square root:

rise = 7

Therefore, the slope of the line is m = rise/run = 7/3.

Section 4: Math (Calculator)

1 . B

Algebra (systems) EASY

Since the question asks for the value of b , it makes sense to substitute for a so that we get a single equation in terms of b .

Second equation:

2a + 4b = 20

Substitute

Simplify and combine:

b + 4b = 5b = 20

Divide by 5:

b = 4

2 . B

Data Analysis (central tendency) EASY

The table summarizes the following list of 20 numbers: 1, 1, 1, 2, 2, 2, 2, 3, 3, 3, 5, 6, 7, 8, 8, 8, 9, 10, 10, 10. If a set of numbers is listed in increasing order, the median is the middle number (if the set contains an odd number of elements) or the average of the two middle terms (if the set contains an even number of elements). The median of a set of 20 numbers, therefore, is the average of the 10th and 11th terms. Since the 10th number is 3 and the 11th number is 5, the median is (3 + 5)/2 = 4.

3 . A

Problem Solving/Data Analysis (proportions) EASY

Set up a proportion:

Cross multiply:

456 = 48x

Divide by 48:

9.5 = x

4 . C

Data Analysis (tables) EASY

The ratio of applicants to finalists is simply the quotient of those two values, which we can calculate for each year.

(A) 8/25 = 0.32,

(B) 8/26 ≈ 0.31,

(C) 9/23 ≈ 0.39,

(D) 10/26 ≈ 0.38.

5 . D

Algebra (exponentials) EASY

Although solving for y and z isn”t hard, it is even simpler to just express (yz )6 in terms of y 3 and z 2 , using the Laws of Exponentials from Chapter 9 , Lesson 9.

Original expression:

(yz )6

Law of Exponentials #5:

y 6 z 6

Law of Exponentials #8:

(y 3 )2 (z 2 )3

Substitute y 3 = 20 and z 2 = 10:

(20)2 (10)3

Substitute 20 = (2)(10):

(2)2 (10)2 (10)3

Combine terms with like bases with Law of Exponents #4:

22 (10)5

Simplify:

4 × 105

6 . A

Algebra (word problems) EASY

The sum of a , b , and c is three times the sum of a and b :

a + b + c = 3(a + b )

Distribute:

a + b + c = 3a + 3b

Subtract a :

b + c = 2a + 3b

Subtract 3b :

–2b + c = 2a

Divide by 2:

7 . A

Advanced Mathematics (triangle trigonometry) EASY

First, let”s mark up the diagram with the given lengths, as above. Remember from SOH CAH TOA that the tangent of an angle is equal to the opposite side over the adjacent side, so tan x = AB/EB. AB = ACBC = 14 – 12 = 2, and we can find EB with the Pythagorean Theorem:

EB 2 + 122 = 132

Simplify:

EB 2 + 144 = 169

Subtract 144:

EB 2 = 25

Take the square root:

EB = 5

Or, even better, just notice that triangle EBC is a 5–12–13 triangle.

So, tan x = AB/EB = 2/5 = 0.4

8 . A

Advanced Math (quadratics) EASY

We can factor this quadratic easily with the Product-Sum Method from Chapter 9 , Lesson 4.

x 2 – 6x + 8 = (x – 4)(x – 2)

9 . A

Data Analysis (pie graph) MEDIUM

Since Maria”s sales accounted for 25% of the total, her sector must be 0.25(360°) = 90°, which is sector D. This means that Eli ($3,000), Georgia ($5,000), and Zoe ($10,000) account for sectors A , B , and C . Since Georgia”s total is between Eli”s and Zoe”s, her sector is the neither the largest nor the smallest of the remaining sectors. Therefore, it must be sector A , which is in the middle.

10 . B

Data Analysis (pie graph) MEDIUM

Perhaps the simplest way to approach this is to notice that, since Maria”s sales account for 25% of the total, the other salespeople must account for 100% – 25% = 75% of the total. Since this total is $3,000 + $5,000 + $10,000 = $18,000, we can find the total with a proportion.

Cross multiply:

$1,800,000 = 75x

Divide by 75:

$24,000 = x

11 . D

Data Analysis (pie graph) MEDIUM

Since Maria accounted for 25% of the total sales, she accounted for (0.25)($24,000) = $6,000 in sales. If she earned 15% commission for all sales, she earned (0.15)($6,000) = $900 in commissions. If Georgia earns 10% in commissions, she earned (0.10)($5,000) = $500. Therefore, Maria earned $900 – $500 = $400 more in commissions that Georgia did.

12 . B

Algebra (absolute value) MEDIUM

The function f (x ) = 2 – |x – 4| reaches its greatest value when the absolute value is minimized. Since absolute values cannot be negative, the least value |x – 4| can have is 0, which it has when x = 4:

f (4 ) = 2 – |4 – 4| = 2 – 0 = 2

13 . C

Advanced Math (rational equations) MEDIUM

Original equation:

Multiply both sides by common denominator 5b :

15 – 2b = 5b

Add 2b :

15 = 7b

Divide by 7:

14 . D

Advanced Mathematics (functions) MEDIUM

(A) f (1) = 12 + 3 = 4; f (2) = 22 + 3 = 7

(B) f (1) = 12 + 9 = 10; f (2) = 22 + 9 = 13

(C) f (1) = 2(1)2 + 2 = 4; f (2) = 2(2)2 + 2 = 10

(D) f (1) = 3(1)2 + 1 = 4; f (2) = 3(2)2 + 1 = 13

The only function that satisfies the two given equations is (D).

15 . A

Advanced Mathematics (exponentials) MEDIUM

(A) (6b ) (6b ) = 36b 2

(B) 12b (b ) = 12b 2

(C)

(D) 6b 2 + 6b 2 = b 2 (6 + 6) = 12b 2

16 . C

Data Analysis (probability) MEDIUM

One way to represent this problem clearly is to construct a table that shows all of the possible products mn . A representation of all the equally likely possible outcomes of an event is called the sample space for that event. We can label the columns with the possible values of m and the rows with the possible values of n . As we write in the products, let”s shade in those that are multiples of 12.

This shows that 4 out of the possible 16 products are multiples of 12, and therefore, the probability is 4/16 or .

17 . B

Algebra (inequalities) MEDIUM

Original inequality:

x < 3

Multiply by 3:

3x < 9

Add 4:

3x + 4 < 13

Substitute y = 3x + 4:

y < 13

18 . B

Advanced Mathematics (functions) MEDIUM-HARD

Since the function takes “all values of x ,” one way to solve this problem is to choose a value of x to work with, like x = 1.

Original function:

g (x + 1) = x 2 + 2x + 4

Substitute x = 1:

g (2) = (1)2 + 2(1) + 4 = 1 + 2 + 4 = 7

Therefore, the function g (x ) will give an output of 7 for an input of 2. We can now test our choices for an input of x = 2. (Notice g (x ) and g (x + 1) have different inputs.)

(A) (2)2 + 4 = 8

(B) (2)2 + 3 = 7

(C) (2 – 1)2 + 4 = 5

(D) (2 – 1)2 + 3 = 4

Notice that only the expression in (B) gives the correct output.

19 . C

Advanced Mathematics (sequences) MEDIUM-HARD

The “brute force” method is to write out sequence A until you reach 72, and see which element in sequence B “matches up” to it. But first we must determine the rule for each sequence. A little guessing and checking should confirm that sequence A follows the “add 5” rule, and sequence B follows the “add 10” rule.

A more elegant method, however, is to find the formulas for the n th term of A and the n th term of B. This would be a much more efficient method, also, if it takes a while for 72 to appear in set A. If you recall the general formula for the n th term of an arithmetic sequence (a n = a 1 + (n – 1)d ), then it”s straightforward to see that the formula for A is a n = 2 + (n – 1)5 = 5n – 3 and the formula for B is b n = 5 + (n – 1)10 = 10n – 5. Since we”re looking for where the number 72 appears in set A, we can solve 5n – 3 = 72 to find n = 15, then insert this value for n into the formula for B: b 15 = 10(15) – 5 = 145.

20 . C

Problem Solving and Data Analysis (ratios) MEDIUM

If the ratio of subscribers to nonsubscribers is 2:5, then we can say there are 2n subscribers and 5n non-subscribers, where n is some integer. This means there were a total of 2n + 5n = 7n July visitors to the website. Since we know that there were 2,100 visitors in July, we can solve for n :

2,100 = 7n

Divide by 7:

300 = n

Therefore, there were 2(300) = 600 subscriber visits and 5(300) = 1,500 nonsubscriber visits, and so there were 1,500 – 600 = 900 more nonsubscribing visitors than subscribing visitors.

21 . A

Algebra (graphing lines) MEDIUM-HARD

In quadrant I, both the x - and y -coordinates are positive. Since y = 4 in all four systems, we simply need to find the system for which the x -coordinate of the solution is not positive. We can find the corresponding x -coordinate for each system by just substituting y = 4 and solving for x .

Substitute y = 4 into first equation in (A):

3x + 5(4) = 15

Simplify:

3x + 20 = 15

Subtract 20:

3x = –5

Divide by 3:

x = –5/3

In this case, we don”t need to go any further, because the solution to the system in (A) is (–5/3, 4), which is in quadrant II, not quadrant I.

22 . D

Advanced Mathematics (quadratics) MEDIUM-HARD

Read the question carefully, and note particularly what it is asking for and what information can help you find it. We are asked to find an equation to relate two variables, q , the number of power units, and t , the number of hours the battery has been charging. We are told that the initial charge is 20 power units, so q = 20 when t = 0. We are also told that the charge increases from 50 power units to 106 power units in 40 minutes. But since our time unit t is in hours , we should convert 40 minutes to 40/60 = 2/3 hours. Therefore, the charging station charges at a rate of (106 – 50)/(2/3) = (56)/(2/3) = 84 charging units per hour. This unit rate is the slope of the line, as we discussed in Chapter 8 , Lesson 5. Therefore, the equation should represent a line with slope of 84 that contains the point t = 0 and q = 20, which is the equation in (D) q = 84t + 20.

23 . C

Data Analysis (scatterplots) EASY

This question simply asks us to find the point on the line of best fit that corresponds to a length of 95 centimeters. As the dotted lines show below, this corresponds to a weight less than halfway between 8 and 9 kilograms, so (C) 8.3 is the best approximation among the choices.

24 . D

Data Analysis (scatterplots) HARD

To find the equation of the line of best fit, we can take two points on the line and then use the point-slope formula (Chapter 7 , Lesson 5) to find the equation of the line. To get the most accurate representation of the line, we should choose two points that are fairly far apart, but whose coordinates are easy to determine. The graph shows that this line appears to pass through the points (60, 3) and (100, 9), and so, by the slope formula (Chapter 7 , Lesson 5) we can calculate that the slope is (9 – 3)/(100 – 60) = 6/40 = 3/20. Using the first point in the

point-slope formula gives

Distribute:

Add 3:

25 . B

Problem Solving (rates) MEDIUM

If Ronika plans to use 85% of her 2 GB data plan for texting, she will have only (15%)(2 GB) = (0.15)(2,000 MB) = 300 MB = 300,000 kB available for image files. Since the average image file is 750 kB, she will be able to send 300,000 kB/750 kB = 400 images per month.

26 . C

Problem Solving (rates) HARD

This question asks us to write a mathematical statement that “indicates the restrictions” in this situation. So, what keeps us from making as many truffles as we want? Simple: we are only allowed to spend $200 per week on cocoa powder. Therefore, we can state the restriction on truffles as “the total cost of cocoa powder for our weekly production of truffles must be less than or equal to $200.”

Now we must figure out a way to express “the total cost of cocoa powder for our weekly production of truffles.” Clearly, this is the total cost for the dark chocolate truffles plus the total cost for the milk chocolate truffles:

Cost of cocoa powder for d dark chocolate truffles:

Simplify:

Cost of cocoa powder for m milk chocolate truffles:

Simplify:

The total cost for cocoa powder must not be greater than $200:

Multiply by 16:

3,200 ≥ 0.65cd + 0.45cm

Divide by c :

27 . C

Advanced Mathematics (exponentials) HARD

Notice that this question asks us to find the value of 2 n +3 , so we should try to solve the given equation for 2 n +3 .

Original equation:

m = 2 n + 2 + 2 n

Factor 2n from the terms on the right side:

m = 2 n (22 + 1)

Simplify:

m = 2 n (5)

Divide by 5:

Multiply by 23 :

Simplify:

28 . D

Special Topics (trigonometry) HARD

Although this question can be solved by graphing, it is simpler and more efficient to imagine the unit circle, as we discussed in Chapter 10 , Lesson 9.

We are asked to consider those angles that have a sine of ½. As you recall from Chapter 10 , Lesson 9, the angles whose sine is ½ correspond to those angles that intersect the unit circle at any point where y = ½, as shown in the diagram above. Notice that the line y = ½ intersects the unit circle in two points. We are asked to consider sin 3x , where x takes values from 0 to 2π. This means that 3x takes values from 0 to 6 π. In other words, we are taking three complete trips around the unit circle (since each trip around is 2π radians). How many times will we visit those two points if we take three trips around the circle? Clearly (3)(2) = 6 times.

29 . B

Advanced Mathematics (function transformations) HARD

The figure clearly shows that the function y = f (x ) is similar in shape to the function y = g (x ), but is shifted to the left by some positive distance. Recall from Chapter 9 , Lesson 3, that when the graph of y = g (x ) is shifted to the left by k units, the equation of the new function is y = g (x + k ). The only equation that has this form is (B ) f (x ) = g (x + 2).

30 . A

Data Analysis (data spread) HARD

One important rule in data gathering is, the more data we can gather on a population, the more reliable our statistics about that population will be . More specifically, the greater fraction of the population we can sample, the smaller our margin of error from the true value of the population statistic. Another important rule in data gathering is the more similar our sample is to the population of interest, the smaller our statistical error will be . Since the sample size is highest and the group is most like the population (of undergraduate computer science majors) in choice (A), that group should produce the smallest margin of error in the data.

31 . 168

Algebra (percents) EASY

The sum of 40 and 80 is 120, and 40% of 120 is (0.40) (120) = 48, so the number that is 40% greater than 120 is 120 + 48 = 168. Also remember that increasing a number by 40% is equivalent to multiplying it by 1.4.

32 . 3/2 or 1.5

Advance Mathematics (quadratics) MEDIUM

Original function:

Substitute h (3 ) = 6 (from table):

Simplify:

6 = 4.5 + k

Subtract 4.5:

1.5 = k

To check your answer, you can plug in the second row of the table to verify that

33 . 52

Algebra (linear equations) MEDIUM

In Chapter 7 , Lesson 5 we discussed the fact that the slope of a linear equation in “standard form,” ax + by = c is equal to –a/b . Therefore, the linear equation hx + 4y = –3 has a slope of –h /4. If this slope equals –3, then

Multiply by –4:

h = 52

34 . 25

Algebra (word problems) EASY

Let”s let x be the larger number. 15 is the smaller number.

The sum of the numbers is four times their difference:

x + 15 = 4(x – 15)

Distribute:

x + 15 = 4x – 60

Add 60:

x + 75 = 4x

Subtract x :

75 = 3x

Divide by 3:

25 = x

35 . 4/45 or .088 or .089

Special Topics (trigonometry) MEDIUM-HARD

Given equation:

Divide by 5:

This gives us the value of cos x , but we are asked to evaluate , which of course is in terms of sin x .

This should remind you of the Pythagorean Identity we discussed in Chapter 10 , Lesson 9: for all real numbers x , sin2 x + cos2 x = 1.

Pythagorean Identity:

sin2 x + cos2 x = 1

Subtract cos2 x :

sin2 x = 1 – cos2 x

Expression to be evaluated:

Simplify:

Substitute sin2 x = 1 – cos2 x :

Substitute cos :

Simplify:

Simplify:

Simplify by multiplying :

36 . 75/4 or 18.75

Special Topics (arcs and triangles) MEDIUM-HARD

Let”s start by drawing the three radii OA, OB , and OC . Since these radii are all congruent, and because AB = BC , the triangles AOB and COB are congruent (by the SSS Theorem). This implies that OB bisects angle ABC , so the base angles of both isosceles triangles must have measure 45°/2 = 22.5°. Therefore, angle AOB , which is the central angle for arc AB , must have measure 180° – 22.5° − 22.5° = 135°. Now we can use the fact that the circumference of the circle is 50 to find the length or arc AB .

Let x = and cross multiply:

360x = 6,750

Divide by 360:

x = 75/4 = 18.75

37 . 47

Problem Solving (extended thinking) HARD

Let n equal the number of months that Isabelle has been on Plan A. If she has been on Plan A for over a year, then n > 12. This means that she has been on Plan A for n − 12 months beyond the first year. Since Plan A costs $500 for the first year and $80 per month thereafter, the total cost for her n months of service is $500 + $80(n − 12). If she had been on Plan B, the cost would have been $68 per month, or a total of $68n . If Plan B would have saved her $104 over this period,

500 + 80(n − 12) − 104 = 68n

Distribute and simplify:

396 + 80n − 960 = 68n

Simplify:

80n − 564 = 68n

Add 564:

80n = 68n + 564

Subtract 68n :

12n = 564

Divide by 12:

n = 47

38 . 32

Problem Solving (extended thinking) HARD

Since Plan C costs $92 per month and Plan B costs $68 per month, Plan C costs $92 − $68 = $24 more dollars per month than plan B. Since shifting plans would save her only 45 minutes of work, or 3/4 hour, each month, she would have to value one hour of free time over one hour of work time at $24/(¾ hour) = $32.

Section 5: Essay

Sample Response

Reading Score: 8 out of 8

Analysis Score: 8 out of 8

Writing Score: 8 out of 8

James Schlesinger”s essay, “Cold Facts on Global Warming,” is a counterargument to the “political alarmism” (to use Schlesinger”s words) over global warming. His tone is critical but sober, and he makes frequent use of carefully selected scientific and historical data, juxtaposed with hints at the dangers of political posturing, to make the case for caution in addressing the issue of climate change. He appeals frequently to the ethics of economic prudence and global stewardship, as well as the value of scientific judiciousness. Unfortunately, because Schlesinger”s essay was written over a decade ago, it lacks the evidence from the current golden age of climate science. More substantially, however, Schlesinger undermines his own purpose by making political criticisms while calling for nonpartisan objectivity, by mongering fearsome scenarios while arguing against “scare tactics,” and by ignoring the scientific evidence against his claims while advocating an “emphasis on science.”

Schlesinger begins his discussion with a call for “facts and logic” over “rhetoric.” This is classic polemical posturing: we all believe that our positions are “factual and logical” and that our opponents” are merely “rhetoric.” In Schlesinger”s view, the “rhetoric” includes the claims that “emissions of carbon dioxide are the primary cause of any change in global temperature and inevitably will lead to serious environmental harm in the decades ahead.” By inserting the modifiers “any” and “inevitably,” he creates a straw man. Most who argue about the seriousness of climate change generally avoid such absolute assertions and instead present evidence from satellites, ice cores, atmospheric analysis, and comprehensive long-term climatic studies to build a case for action. Schlesinger does not address this evidence.

In his argument, Schlesinger appears to value small government and the protection of American industry over the stewardship of the planet. His concern about the Kyoto Protocol of 1997 is not that it eschews the “facts and logic” of climate science, but rather that it “could cost $300 billion annually.” He presents no scientific critique of the Kyoto Protocol of 1997 beyond the assertion that Democrat Al Gore was “willing to embrace” a “treaty that would harm the economy,” and the vague claim that “the treaty”s flaws have become more evident.” His method of argumentation here appears to contradict his call for “facts and logic” over “rhetoric.”

In contrast to the irresponsibility of Al Gore and the Clinton administration, Schlesinger offers the soberly scientific Bush administration, which “focused on filling in gaps in our state of knowledge, promoting the development of new technology, encouraging volunteer programs, and working with other nations on controlling the growth of greenhouse gas emissions.” Schlesinger does not offer a specific benefit our planet has gained from these efforts, which even Schlesinger himself admits involved “spending more than $4 billion annually.” Someone pleading for fiscal responsibility might try to account for such a huge expenditure.

Schlesinger believes that our inaction on climate change is a virtue: that scientific prudence requires “filling the gaps in our state of knowledge” above everything else, including industrial restraint. He states that “what we know for sure is quite limited,” yet is confident enough in his limited knowledge to assert that “the theory that increasing concentrations of greenhouse gases like carbon dioxide will lead to further warming is at least an oversimplification,” directly contradicting the simple middle school experiment showing that a soda bottle filled with carbon dioxide warms far more quickly than one filled only with air.

Schlesinger then selects data trends that seem to support his call for caution, rather than action: he asserts that “satellite measurements over 35 years show no significant warming in the lower atmosphere” and that there was “atmospheric cooling from 1940 to around 1975.” Schlesinger does not explain why climate scientists, who are certainly aware of these data, nevertheless believe in anthropogenic global warming.

Not to be accused of cherry-picking data, Schlesinger next offers “a longer view of climate history.” He asserts that temperatures “were 1 to 2 degrees warmer than they are today” during the Climatic Optimum of the early Middle Ages, and this warming did not have “anything to do with man-made greenhouse gases.” Evidently, we should think that because it was warmer a very long time ago, burning coal today must not be changing the climate.

In the last two paragraphs, Schlesinger essentially retracts his concern about “filling the gaps in our state of knowledge” after all, because he believes it is impossible to fill the most important gaps: “It is not possible to know now how much of the warming over the last 100 years or so was caused by human activities and how much was because of natural forces.” So if it is impossible to know, we might ask, why should we expend “more than $4 billion annually” to study it? He does not say. We get Schlesinger”s most sonorous call to action in the last paragraph, where he suggests “engagement of the business community on voluntary programs.” That is, get big government off the backs of corporations and let them do as they please.

Scoring

Reading—8 (both readers gave it a score of 4)

This response demonstrates a very strong and thorough comprehension of Schlesinger”s essay through skillful use of summary, paraphrase, and direct quotations. The author summarizes Schlesinger”s central tone, thesis, and modes of persuasion (His tone is critical but sober, and he makes frequent use of carefully selected scientific and historical data, juxtaposed with hints at the dangers of political posturing, to make the case for caution in addressing the issue of climate change .) and shows a clear understanding of how Schlesinger”s supporting ideas string together and serve his overall thesis (Schlesinger begins his discussion with a call … He appears to value small government … Schlesinger offers the soberly scientific Bush administration … Schlesinger believes that our inaction on global warming is a virtue … Schlesinger next offers … In the last two paragraph, Schlesinger essentially retracts his concern). Importantly, this response also offers abundant supporting quotations to illustrate each paraphrase. Taken together, these elements demonstrate outstanding comprehension of Schlesinger”s essay.

Analysis—8 (both readers gave it a score of 4)

Although this response occasionally veers toward advocacy, it never turns away from careful analysis. Indeed, its thoughtful and thorough critique of Schlesinger”s essay demonstrates a sophisticated understanding of the analytical task. The author has identified Schlesinger”s primary modes of argument (He appeals frequently to the ethics of economic prudence and global stewardship, as well as the value of scientific judiciousness ) and even uses those standards to analyze Schlesinger”s essay itself, and indicates points at which Schlesinger”s argument seems self-defeating (Schlesinger undermines his own purpose by making political criticisms while calling for nonpartisan objectivity, by mongering fearsome scenarios while arguing against “scare tactics,” and by ignoring the scientific evidence against his claims while advocating an “emphasis on science” ). Overall, this analysis of Schlesinger”s essays demonstrates a thorough understanding not only of the rhetorical task that Schlesinger has set for himself, but also of the degree to which it upholds its own standards.

Writing—8 (both readers gave it a score of 4)

This response demonstrates an articulate and effective use of language and sentence structure to establish and develop a clear and insightful central claim that Schlesinger”s essay is a counterargument to the “political alarmism” … over global warming … but that it undermines [its] own purpose . The response maintains a consistent focus on this central claim, and supports it with a well-developed and cohesive analysis of Schlesinger”s essay. The author demonstrates effective choice of words and phrasing (undermines his own purpose … mongering fearsome scenarios … Schlesinger believes that our inaction on climate change is a virtue ), strong grasp of relevant analytical and rhetorical terms, like economic prudence, nonpartisan objectivity, and polemical posturing . The response is well-developed, progressing from general claim to specific analysis to considered evaluation. Largely free from grammatical error, this response demonstrates strong command of language and proficiency in writing.

PRACTICE TEST 3

  1. Reading Test

65 MINUTES 52 QUESTIONS

  1. Writing and Language Test

35 MINUTES 44 QUESTIONS

  1. Math Test – No Calculator

25 MINUTES 20 QUESTIONS

  1. Math Test – Calculator

55 MINUTES 38 QUESTIONS

  1. Essay (optional)

50 MINUTES 1 QUESTION

ANSWER SHEET

start with number 1 for each new section. If a section has fewer questions than answer spaces, leave the extra answer spaces blank. Be sure to erase any errors or stray marks completely.

Start with number 1 for each new section. If a section has fewer questions than answer spaces, leave the extra answer spaces blank. Be sure to erase any errors or stray marks completely.

CAUTION Use the answer spaces in the grids below for Section 3 only if you are told to do so in your test book.

Student-Produced Responses ONLY ANSWERS ENTERED IN THE CIRCLES IN EACH GRID WILL BE SCORED. YOU WILL NOT RECEIVE CREDIT FOR ANYTHING WRITTEN IN THE BOXES ABOVE THE CIRCLES.

Start with number 1 for each new section. If a section has fewer questions than answer spaces, leave the extra answer spaces blank. Be sure to erase any errors or stray marks completely.

CAUTION Use the answer spaces in the grids below for Section 3 only if you are told to do so in your test book.

Student-Produced Responses ONLY ANSWERS ENTERED IN THE CIRCLES IN EACH GRID WILL BE SCORED. YOU WILL NOT RECEIVE CREDIT FOR ANYTHING WRITTEN IN THE BOXES ABOVE THE CIRCLES.

SECTION 5: ESSAY

You may wish to remove these sample answer document pages to respond to the practice SAT Essay Test.

SECTION 5: ESSAY

You may wish to remove these sample answer document pages to respond to the practice SAT Essay Test.

SECTION 5: ESSAY

You may wish to remove these sample answer document pages to respond to the practice SAT Essay Test.

SECTION 5: ESSAY

You may wish to remove these sample answer document pages to respond to the practice SAT Essay Test.

Reading Test

65 MINUTES, 52 QUESTIONS

Turn to Section 1 of your answer sheet to answer the questions in this section.

DIRECTIONS

Each passage or pair of passages below is followed by a number of questions. After reading each passage or pair, choose the best answer to each question based on what is stated or implied in the passage or passages and in any accompanying graphics (such as a table or graph).

Questions 1–11 are based on the following passages and supplementary material.

Passage 1 is from F. J. Medina, “How to Talk about Sustainability.” ©2015 C. F. Black and College Hill Coaching. Passage 2 is adapted from an essay published in 2005 about the economic analysis of environmental decisions.

Passage 1

Passage 2

1

The first two sentences of Passage 1 serve primarily to

  1. A) provide historical background to a debate.
  2. B) establish the author”s central thesis.
  3. C) define terms for a technical discussion.
  4. D) characterize opposing viewpoints.

2

The repetition of the phrase “not all” in lines 15 and 16 emphasizes the author”s point that the “debaters” (line 13) tend to

  1. A) mischaracterize their opponents.
  2. B) discount evidence that does not support their positions.
  3. C) employ self-contradicting arguments.
  4. D) overlook relevant personal anecdotes.

3

The phrase “life cycles” (line 22) refers most directly to the

  1. A) reproductive and feeding habits of local plants and animals.
  2. B) variability in public perceptions about recycling.
  3. C) global economic trends that affect industrial production.
  4. D) processes that affect the substances used in manufacturing.

4

In line 49, “inserting” most nearly means

  1. A) installing.
  2. B) imposing.
  3. C) introducing.
  4. D) interjecting.

5

Which reference in Passage 1 would the author of Passage 2 most likely regard as a direct effect of “natural ecosystems” (line 57)?

  1. A) The “real solutions” in line 17
  2. B) The “provenance of the materials” in lines 27–28
  3. C) The “water filtration” in line 29
  4. D) The “maintenance” in line 33

6

The author of Passage 1 would most likely endorse which of the following industrial policies?

  1. A) Tax incentives for companies that recycle their waste products
  2. B) Sanctions against nations that permit slave or child labor
  3. C) Limits on factory emissions that contribute to acid rain
  4. D) Public investment in sustainable domestic energy sources

9

Which of the following references in Passage 1 best exemplifies the “clear exchange” mentioned in line 73 of Passage 2?

  1. A) The “debate” in line 14
  2. B) The “natural benefits” in line 28
  3. C) The “maintenance cost” in lines 41–42
  4. D) The “important insights” in line 44

10

Unlike Passage 2, Passage 1 specifically discusses

  1. A) the effect of a particular rhetorical strategy.
  2. B) the quantification of particular externalities.
  3. C) the popularity of a particular environmental policy.
  4. D) the cost of preserving particular environmental benefits.

11

Passage 2 compares the viewpoints of “environ-mentalists” and “industrialists” primarily to point out that

  1. A) disputes about environmental policies are deep and intractable.
  2. B) careful examination of externalities is controversial but necessary.
  3. C) many debates about environmental issues are needlessly politicized.
  4. D) both parties must learn to focus more on economic issues than environmental ones.

Questions 12–21 are based on the following passage.

This passage is from Cait Featherstone, Earth, Song and Sky Spirit: Shadows and Sleepwalkers . ©1992 by Random House, Inc.

12

The purpose of the passage as a whole is to

  1. A) describe a relationship between friends.
  2. B) portray the character of a small town.
  3. C) recount an episode in the narrator”s self-discovery.
  4. D) chronicle a preoccupation with an enigmatic figure.

13

The many stories that circulated about the stranger are best described as

  1. A) uniformly macabre.
  2. B) strangely entertaining.
  3. C) playfully deprecating.
  4. D) decreasingly speculative.

14

Which choice provides the best evidence for the answer to the previous question?

  1. A) Lines 8–9 (“Maybe … venture”)
  2. B) Lines 14–16 (“Eventually … fact”)
  3. C) Lines 17–18 (“Soundlessly … things”)
  4. D) Lines 25–27 (“Was … ended?”)

15

In lines 24–25, “moonscape surface” refers to

  1. A) a desolate aftermath.
  2. B) an unexplored vista.
  3. C) an idyllic location.
  4. D) a primordial stage.

16

In lines 59–60, “character” most nearly means

  1. A) visible symbol.
  2. B) abstract quality.
  3. C) strange person.
  4. D) moral integrity.

17

In lines 30–43, the narrator”s encounters with the stranger are notable for their

  1. A) incongruousness.
  2. B) ominousness.
  3. C) ubiquity.
  4. D) sentimentality.

18

Throughout the passage, the narrator describes the stranger”s physical characteristics chiefly through the use of

  1. A) literary allusions.
  2. B) military comparisons.
  3. C) avian metaphors.
  4. D) exaggerated juxtapositions.

19

The phrase “what we would all come to know” (lines 28–29) most likely refers to

  1. A) the identity of the stranger.
  2. B) the destiny of humankind.
  3. C) the inhumanity of war.
  4. D) a source of tranquility.

20

In line 74, the narrator indicates that the stranger “seemed to journey” from

  1. A) foreign terrain to a familiar homeland.
  2. B) grim memory to current experience.
  3. C) vague obscurity to public recognition.
  4. D) self-consciousness to self-confidence.

21

The first words the narrator heard from the stranger most likely made her feel a sense of

  1. A) solemn respect.
  2. B) surprised relief.
  3. C) sudden dread.
  4. D) deep tranquility.

Questions 22–31 are based on the following passage.

This passage is adapted from Mary Gay Humphries, “Women Bachelors,” an essay originally published in 1896. During the latter part of the American Industrial Era (c. 1840–1900), many unmarried women began migrating to urban areas throughout the country.

22

The first paragraph portrays the “exodus of women” (line 1) as

  1. A) tentative.
  2. B) regrettable.
  3. C) inevitable.
  4. D) transformative.

23

The author suggests that, compared to women living in urban areas, those living in rural areas are less

  1. A) diffident.
  2. B) humorous.
  3. C) innovative.
  4. D) traditional.

24

Which choice provides the best evidence for the answer to the previous question?

  1. A) Lines 7–13 (“New … lines”)
  2. B) Lines 13–16 (“The … air”)
  3. C) Lines 17–18 (“When … knowledge”)
  4. D) Lines 18–20 (“Instead … interest”)

25

The author suggests that, to the new urban woman, poverty is

  1. A) a challenge to be embraced.
  2. B) a career burden to be avoided.
  3. C) the consequence of male dominance.
  4. D) a surprising source of freedom.

26

Which choice provides the best evidence for the answer to the previous question?

  1. A) Lines 24–28 (“She … before”)
  2. B) Lines 31–32 (“In … homes”)
  3. C) Lines 33–37 (“The … places”)
  4. D) Lines 43–48 (“It … achievement”)

27

In line 16, “dash” most nearly means

  1. A) propriety.
  2. B) flair.
  3. C) diligence.
  4. D) haste.

28

As it is used in line 26, “old set” most likely refers to a group of

  1. A) traditional gender roles.
  2. B) established acquaintances.
  3. C) historical ideals.
  4. D) abandoned opportunities.

29

The passage indicates that city women want to maintain their own homes primarily because

  1. A) they are naturally predisposed to performing domestic duties rather than having careers.
  2. B) they should maintain a social status comparable with that of men.
  3. C) they require living conditions conducive to their social independence.
  4. D) they need ample space to do the work that is required of them in an industrial economy.

30

Which choice best summarizes the main point of the passage?

  1. A) Women who are moving to the cities are subject to many unfair expectations and social burdens.
  2. B) Traditional female duties, such as housekeeping, should be re-evaluated in the context of modern urbanization.
  3. C) In modern times, the social independence of women corresponds to their desire to own and maintain a home.
  4. D) Women who choose to live in cities are more creative and industrious than those who choose to live in rural areas.

31

The “exemption” mentioned in line 36 is

  1. A) a reprieve from a social obligation.
  2. B) an exception to a legal rule.
  3. C) an anomaly among personal characteristics.
  4. D) an irregularity within an established hierarchy.

Questions 32–42 are based on the following passage and supplementary material.

This passage is adapted from Christopher F. Black, “The Promise of Immunotherapy and Oncolytic Virotherapy.” ©2015 by Christopher F. Black and College Hill Coaching.

32

The second paragraph (lines 8–22) serves mainly to

  1. A) relate a point of view toward viruses that contrasts with that adopted by modern microbiologists.
  2. B) provide some technical information pertaining to virotherapy in order to clarify the discussion that follows.
  3. C) describe several insidious diseases the treatments for which serve as a model for virotherapy.
  4. D) illustrate the early failures of virotherapy that modern physicians have since overcome.

33

The passage indicates that HIV and leukemia are similar in that both

  1. A) can be effectively treated through virotherapy.
  2. B) are caused by viral infection of healthy cells.
  3. C) can be treated by reinforcing the immune system.
  4. D) can be used as treatments for other diseases.

34

In line 19, “front” most nearly means

  1. A) façade.
  2. B) campaign.
  3. C) beginning.
  4. D) bearing.

35

According to the information in the passage, the “poor viral receptivity” illustrated in the diagram is most likely due to

  1. A) a failure of the “lock-and-key” mechanism.
  2. B) a “cloaking” of the cell against T-cell attack.
  3. C) the triggering of apoptosis.
  4. D) the toxic effects of chemotherapy.

36

The diagram illustrates all of the following processes EXCEPT

  1. A) T-cell attack.
  2. B) lysis.
  3. C) selective infection.
  4. D) apoptosis.

37

What potential drawback does this diagram suggest could compromise the effectiveness of virotherapy as a cancer treatment?

  1. A) The virus may not be able to infect the cancer cell or replicate within it.
  2. B) The immune system may eliminate the viruses before they have a chance to destroy the cancer cell.
  3. C) The virus may cause lysis of healthy cells.
  4. D) The virus may not spread after killing the cancer cell by lysis.

38

The passage indicates that, until recently, the human immune system has been unable to attack cancers cells effectively because the human immune system

  1. A) selectively attacks and destroys viruses rather than cells.
  2. B) cannot penetrate the blood vessels to initiate vascular collapse.
  3. C) is compromised by therapies such as radiation and chemotherapy.
  4. D) is thwarted by chemical defenses that cancer cells have developed.

39

Which choice provides the best evidence for the answer to the previous question?

  1. A) Lines 43–45 (“Evidently … cancer”)
  2. B) Lines 55–58 (“As … unscathed”)
  3. C) Lines 80–84 (“Lastly … tumor”)
  4. D) Lines 85–89 (“One … effectively”)

40

In lines 46–47, “attempts to replicate” refers to acts of

  1. A) viral reproduction.
  2. B) therapeutic repetition.
  3. C) scientific refutation.
  4. D) pharmaceutical marketing.

41

A student claims that viruses can destroy cancer cells only by directly infecting them. Which of the following statements from the passage most directly contradicts this claim?

  1. A) Lines 58–62 (“Such … tissue”)
  2. B) Lines 68–73 (“First … cells”)
  3. C) Lines 73–76 (“Second … suicide”)
  4. D) Lines 76–80 (“Third … starvation”)

42

The passage suggests that the “precision” mentioned in line 59 is most likely to be achieved through

  1. A) vaccination against viruses like rabies.
  2. B) genetic manipulation of viruses.
  3. C) the combination of radiation and chemotherapy.
  4. D) the triggering of apoptosis.

Questions 43–52 are based on the following passage.

This passage is adapted from Stephen T. Asma, “Animal Spirits.” ©2013 Stephen T. Asma. Originally published in Aeon (aeon.co), February 6, 2013. In this article, Asma describes a visit to the Serengeti plains of Africa and reflects on human and animal emotions.

43

Which of the following best summarizes the main thesis of the passage?

  1. A) We should appreciate the emotional lives of animals if we want to live sustainably with them.
  2. B) Modern research is revealing that animals are far more intelligent than we previously believed.
  3. C) The African savanna must be preserved so that we may continue to explore animal behavior.
  4. D) We cannot understand human intelligence without first understanding animal emotions.

44

In lines 16–21 (“Scientists … bonds”), the sequence of words in italics represents

  1. A) incrementally more powerful motivators.
  2. B) progressively more speculative theories.
  3. C) increasingly human sentiments.
  4. D) decreasingly rational behaviors.

45

In line 26, “tilt” most nearly means

  1. A) upset.
  2. B) sway.
  3. C) slope.
  4. D) contend.

46

The passage mentions the work of Antonio Damasio in lines 27–34 primarily to

  1. A) highlight a surprising discovery about the function of emotions in human thinking.
  2. B) identify a setback to early research into human cognitive function.
  3. C) illustrate a problem with drawing inferences about an entire species on the basis of a few individuals.
  4. D) illustrate the necessity of establishing emotional relationships with research subjects.

47

In line 39, “those early days” refers to

  1. A) the first stages of a child”s neural development.
  2. B) the period before neuroscientists had access to modern diagnostic tools.
  3. C) the epoch in which our ancestors were developing the ability to think.
  4. D) the time when researchers first began exploring the emotional brain.

48

In line 64, “sculpt” refers to an act of

  1. A) deliberate fabrication.
  2. B) aesthetic creation.
  3. C) theoretical refinement.
  4. D) gradual development.

49

In line 76, “corresponds” most nearly means

  1. A) coincides.
  2. B) communicates.
  3. C) agrees.
  4. D) meets.

50

The main purpose of the sixth paragraph (lines 61–78) is to

  1. A) explain a technical term.
  2. B) describe an ancient habitat.
  3. C) refute a scientific theory.
  4. D) illustrate a psychological phenomenon.

51

The author objects to the theory that our brains evolved problem-solving modules, because such modules would be inconsistent with

  1. A) the variable conditions of the Pleistocene epoch.
  2. B) the complex emotional behaviors we share with other animals.
  3. C) our ability to use feelings to make decisions.
  4. D) our long childhoods and elaborate child-rearing practices.

52

Which choice provides the best evidence for the answer to the previous question?

  1. A) Lines 16–21 (“Scientists … bonds”)
  2. B) Lines 32–34 (“Without … interest”)
  3. C) Lines 67–69 (“In … minds”)
  4. D) Lines 79–83 (“Hominids … mothers”)

STOP

If you finish before time is called, you may check your work on this section only. Do not turn to any other section of the test.

Writing and Language Test

35 MINUTES, 44 QUESTIONS

Turn to Section 2 of your answer sheet to answer the questions in this section.

DIRECTIONS

Each passage below is accompanied by a number of questions. For some questions, you will consider how the passage might be revised to improve the expression of ideas. For other questions, you will consider how the passage might be edited to correct errors in sentence structure, usage, or punctuation. A passage or a question may be accompanied by one or more graphics (such as a table or graph) that you will consider as you make revising and editing decisions.

Some questions will direct you to an underlined portion of a passage. Other questions will direct you to a location in a passage or ask you to think about the passage as a whole.

After reading each passage, choose the answer to each question that most effectively improves the quality of writing in the passage or that makes the passage conform to the conventions of Standard Written English. Many questions include a “NO CHANGE” option. Choose that option if you think the best choice is to leave the relevant portion of the passage as it is.

Questions 1–11 are based on the following passage and supplementary material.

Careers in Engineering

1 Wherever engineers are often unfairly portrayed in the media as mere number-crunchers, we all depend on their work every day. The safety of our drinking water, the reliability of our roads and bridges, 2 how usable our smartphones are , and even the sustainability of the earth”s ecosystem all depend on the work of engineers. As we become more dependent on technologies of all sorts, engineering fields are growing quickly. 3

1

  1. A) NO CHANGE
  2. B) When
  3. C) Although
  4. D) Being that

2

  1. A) NO CHANGE
  2. B) the usability our smartphones have
  3. C) the usability of our smartphones
  4. D) our smartphones” usability

3

The writer wants to add a sentence here that provides a specific and relevant detail from the graph. Which choice best accomplishes this?

  1. A) All engineering fields are expected to grow by at least 5% per year for the foreseeable future.
  2. B) Some engineering fields will more than double in size over the next 10 years.
  3. C) Some engineering fields will remain stagnant over the next 10 years, while many will grow dramatically.
  4. D) Some engineering fields are expected to grow by over 25% in the next 10 years.

Many careers in engineering go far beyond 4 merely the application of formulas , throwing switches, and analyzing data. Although 5 a strong foundation in mathematics and science is required for all engineering careers , many also require strong skills in art, design, and the human sciences. Increasingly, technological devices don”t just need to work, they also need to work with people.

Engineering can be regarded as the science of systems. Computer software and hardware engineers analyze the systems that guide computer tasks. Industrial engineers examine the systems by which factories 6 transform raw materials and make products out of them . Civil engineers look at the systems involved in the flow of traffic, water, electricity, and communication. Environmental engineers analyze ecosystems and 7 the ways human activities impact them.

If you like to solve mathematical and physical problems and 8 seeing the tangible fruits of your labor, you should consider a career in engineering. Many entry-level engineering jobs require no higher than a bachelor”s degree in science, but higher paying jobs will likely require a professional engineer (PE) certification or master”s degree.

4

  1. A) NO CHANGE
  2. B) mere applying formulas
  3. C) mere application of formulas
  4. D) merely applying formulas

5

  1. A) NO CHANGE
  2. B) all engineering careers require a strong foundation in mathematics and science
  3. C) a strong mathematics and science foundation would be required for all engineering careers
  4. D) all engineering careers would require a strong foundation in mathematics and science

6

  1. A) NO CHANGE
  2. B) change and transform raw materials into products
  3. C) make products out of raw materials
  4. D) transform raw materials out of which to make new products

7

  1. A) NO CHANGE
  2. B) the ways human activity impact
  3. C) human activity has an impact on
  4. D) the ways by which human activities impact

8

  1. A) NO CHANGE
  2. B) like also seeing
  3. C) like also to see
  4. D) to see

[1] Early in life, many of us have a natural love for 9 engineering: they take apart toy cars to see how they work, or build bridges and castles out of boxes and blocks. [2] Sadly, this enthusiasm is often destroyed by schooling. [3] One solution to this problem is to expose children to fun building activities without pitting them against each other, turning the task into a performance. 10

11 We need to make engineering fun for children again, because so much depends on it . Cultures may be built by philosophers and poets, but societies are built by engineers.

9

  1. A) NO CHANGE
  2. B) engineering, they
  3. C) engineering, we
  4. D) engineering: we

10

The author is considering adding the following sentence to this paragraph.

By forcing students to complete dull worksheets and take competitive, passion-destroying tests, schools can transform the joy of learning into misery.

The best place for this sentence is immediately

  1. A) before sentence 1.
  2. B) after sentence 1.
  3. C) after sentence 2.
  4. D) after sentence 3.

11

Which choice provides the most logical and effective transition to the final paragraph?

  1. A) NO CHANGE
  2. B) Engineering can be not only a joyful experience but also among the most lucrative careers one can have.
  3. C) Students in Finland and Singapore score consistently at the top of international tests in math and science.
  4. D) Like medicine, engineering is a many-faceted discipline that requires years of specialized practice.

Questions 12–22 are based on the following passage.

An American Duty

Too many American voters haven”t changed the way they 12 thought about elections since they first voted for their middle school student council. When it comes to choosing a leader, we are too influenced by his or her looks, personality, and 13 what people are saying, good and bad . Even worse, we seem to be terrible at understanding our biases and predicting the consequences of our votes, and so many of us even end up voting against our own interests.

Choosing our political leaders should be a serious task. This means that, like serious students, we should come to class 14 having done our homework, rather than merely complaining about the teacher not being entertaining enough. Our homework should be to identify the most important problems our society 15 faces, studying the mechanisms at the heart of those problems, and to determine what roles, if any, our leaders can and should play in addressing them.

Instead, we are far too lazy and far too easily manipulated. We let other 16 people—attractive news presenters, blustery radio hosts, or celebrities—tell us how we should vote, usually by giving us scary stories about the opposing party or slickly produced profiles of their favored candidates. Negative campaign ads, with ominous music playing over blurry black-and-white pictures accompanied by a threatening voice, are far too 17 valid . Responsible adults shouldn”t fall for such transparently dishonest tactics.

12

  1. A) NO CHANGE
  2. B) are thinking
  3. C) have thought
  4. D) think

13

  1. A) NO CHANGE
  2. B) what others are saying
  3. C) their reputations
  4. D) reputation

14

  1. A) NO CHANGE
  2. B) doing
  3. C) having to do
  4. D) when we did

15

  1. A) NO CHANGE
  2. B) faces, to study
  3. C) would face, to study
  4. D) face, studying

16

  1. A) NO CHANGE
  2. B) people who are attractive
  3. C) people, especially attractive
  4. D) people: attractive

Instead of 18 us listening to the chattering class and blatantly biased ads, we must learn for ourselves what is going on in the world, with as few corporate or political filters as possible. 19 To that end , we should seek out reliable international newspapers and news sites with high journalistic standards, that is, those that seek more to inform than to entertain. This will give us a better perspective on both domestic and international issues.

[1] Next, we should make sure that the candidates at least understand the issues deeply, and aren”t merely reciting a political platform. [2] For instance, good representatives should be able to objectively explain, in depth, the issues plaguing the American health care system, rather than using distortions and incendiary language to smear their opponents. [3] They articulate the grievances and dynamics that drive wars and international conflicts, rather than merely posture as a “hawk” or a “dove.” 20

17

  1. A) NO CHANGE
  2. B) beneficial
  3. C) effectual
  4. D) credible

18

  1. A) NO CHANGE
  2. B) when we listen
  3. C) listening
  4. D) the listening

19

  1. A) NO CHANGE
  2. B) Alternately,
  3. C) Notwithstanding that,
  4. D) Nevertheless,

20

The author is considering adding the following sentence to this paragraph.

Strong leaders should convey that understanding to voters, rather than merely manipulate or pander to them.

The best place for this sentence is immediately

  1. A) before sentence 1.
  2. B) after sentence 1.
  3. C) after sentence 2.
  4. D) after sentence 3.

We must also find out 21 who”s interests do the candidates really represent. Are the candidates likely to fight for large industries or common consumers? Are they strict party loyalists, or do they act and think independently? Have they pledged allegiance to any group that holds dangerous or misguided views? 22 Do they spend a lot of time in the spotlight, or do they tend to shy away from publicity?

We must answer these questions for ourselves, and not merely swallow the perspectives of radio or television personalities, no matter how blustery or attractive they might be. The strength of our democracy depends on it.

21

  1. A) NO CHANGE
  2. B) whose interests
  3. C) who”s interests
  4. D) about who”s interests

22

The author is considering deleting this sentence to make the paragraph more concise. Should the author do this?

  1. A) No, because it makes a new and important point about political interests.
  2. B) No, because without it the preceding rhetorical questions do not make sense.
  3. C) Yes, because the question is incongruous with the central idea of the paragraph.
  4. D) Yes, because the question has already been answered in the preceding discussion.

Questions 23–33 are based on the following passage.

Idol Worship in Sports

As a source of both inspiration and relaxation, 23 human cultures have always had sports playing a central role . Correspondingly, our greatest athletes are always among our greatest icons. In ancient Greece, the wrestler Milo of Croton earned such 24 renown for his strength and skill that he was known as “The Son of Zeus.” Centuries later, 25 gladiators like Spartacus of Thrace earned a powerful following that enabled him to liberate thousands of slaves. Today, children and adults alike wear jerseys emblazoned with the names of their favorite athletes, and spend hours every week not only watching their games, but also 26 dissecting those performances later with friends .

We don”t 27 regard our most popular athletes as immortal any more, yet sports idolatry certainly doesn”t seem to have diminished much since ancient times. Most sports fans see it as a harmless, and perhaps even beneficial, pastime. What little boy doesn”t look back fondly on trying to juggle a soccer ball like Clint Dempsey or throw a split-fingered fastball like Mariano Rivera? And little girls have long aspired to be the next Serena Williams or Abby Wambach.

23

  1. A) NO CHANGE
  2. B) human cultures have always used sports in a central role.
  3. C) sports are always playing a central role in human cultures.
  4. D) sports have always played a central role in human cultures.

24

  1. A) NO CHANGE
  2. B) mastery
  3. C) diligence
  4. D) revelation

25

  1. A) NO CHANGE
  2. B) the gladiator Spartacus of Thrace
  3. C) gladiators such as Spartacus of Thrace
  4. D) a gladiator such as Spartacus of Thrace

26

Which choice best extends and completes the idea of the sentence?

  1. A) NO CHANGE
  2. B) learning about other sports that might be of interest
  3. C) practicing their own athletic skills
  4. D) going to see their performances in person

27

  1. A) NO CHANGE
  2. B) regard our most popular athletes to be
  3. C) consider our most popular athletes as
  4. D) consider that our most popular athletes are

Is there a cost to all of this idol worship? In fact, there are many. For one, it uses a lot of physical and mental energy, and 28 nevertheless distracts us from our tackling important issues that need our attention. An American is far more likely to be able to name all of the NFL quarterbacks than to name all of the countries in which we have an active military presence. Second, it 29 decelerates the moral fabric of our society. Well-known professional and college athletes are regularly given special treatment, even when they commit heinous crimes, like spousal abuse and felony assault, and often receive 30 just slaps on the wrist when less privileged defendants might receive lengthy prison terms.

Perhaps most damning, all of this worship doesn”t even seem to help the vast majority of the athletes themselves. According to a 2009 Sports Illustrated article, 78% of NFL players 31 have either declared bankruptcy , or suffered serious financial distress within two years of retirement. Even the most successful stars often leave the sport with long-term physical and mental deficits due to the constant physical pounding they must endure throughout their careers. This says nothing of the countless college and semi-pro players who never made it to the NFL but who nevertheless squandered their educations or suffered debilitating injuries 32 .

We don”t live in hunting tribes anymore, so perhaps it”s time to stop acting as if we do. In our complex world, we should be valuing those who can think and care for others, not just those who can run or throw. If we need idols, why not biochemists who toil incessantly to find cures for cancer? Why not soldiers, teachers, nurses, and police who serve honorably and selflessly, usually for very little pay, 33 but they often have as much skill as the finest athletes?

28

  1. A) NO CHANGE
  2. B) instead
  3. C) conversely
  4. D) thereby

29

  1. A) NO CHANGE
  2. B) demoralizes
  3. C) demonizes
  4. D) degrades

30

Which choice best maintains the tone and style of the paragraph and adds a relevant detail?

  1. A) NO CHANGE
  2. B) essentially nothing to speak of
  3. C) barely the punishment that they deserve
  4. D) a light sentence or mere probation

31

  1. A) NO CHANGE
  2. B) have either declared bankruptcy or
  3. C) either have declared bankruptcy, or
  4. D) have, either declared bankruptcy, or

32

At this point, the author is considering adding the following information.

that rendered them unproductive in what should have been their prime earning years

Should the author make this addition here?

  1. A) Yes, because it provides an example of a particular physical injury related to playing sports.
  2. B) Yes, because it describes the impact that sports injuries can have on the athletes” lives.
  3. C) No, because it mentions the years after the athletes” sports careers, blurring the focus of the paragraph.
  4. D) No, because it contradicts the point in the previous paragraph that athletes receive special benefits.

33

  1. A) NO CHANGE
  2. B) and with often
  3. C) but often with
  4. D) but often

Questions 34–44 are based on the following passage.

The Secret Life of Photons

Anyone who has gazed into the deep darkness of space on a clear, moonless night can understand why stars have always fascinated us. Ancient cultures believed that stars 34 sufficed as omens of earthly events, and even influenced individual lives through their “interactions” with the planets. To the more science-minded, they 35 have beckoned for us long with the physical mysteries behind their colossal beauty.

36 Although we now know that the stars beyond our sun are far too distant to exert any significant influence on planetary motion, let alone human affairs, it seems that modern astronomers are nevertheless exercising mystical powers of divination. Today”s astronomers claim to know the distance, age, speed, and even chemical composition of objects that are so distant we 37 might hope never to send probes there. 38 How can they claim to know so much about stars that are often millions of light years away?

It”s really an incredible feat. Consider the remarkable fact that all the information we have about stars is indirect. Unlike other objects of fascination—like microbes, the ocean floor, or even the moon— 39 we can”t easily sample stars to observe them under a microscope. Instead, we must depend on the photons they emit, tiny packets of energy that must often travel thousands of billions of billions of miles or 40 more. At that point, the photons are finally absorbed by a digital camera or the retina of an eyeball that is peering through a telescope. Additionally, we can infer information about a star by looking in its neighborhood. Changes in the behavior of nearby bodies such as other stars or clouds of gas often indicate the presence of a star, 41 even when that star is too dim to see.

The light from the most distant observable 42 stars, began their journey more than 50 million years ago. In that time, the steady expansion of space itself stretched the wavelength of each photon, in a process called “redshifting.” The 43 experience of this redshifting tells astronomers how far the photon had to travel, and hence how far away the star is (or was).

The photons from a single star are not all the same wavelength, however, and that”s a good thing. The wavelengths fall into a wide 44 spectrum, which characteristics reveal the size of the star as well as the presence of elements such as hydrogen, helium, calcium, sodium, and even titanium.

34

  1. A) NO CHANGE
  2. B) performed
  3. C) attended
  4. D) served

35

  1. A) NO CHANGE
  2. B) have long beckoned us
  3. C) have beckoned long for us
  4. D) long have beckoned for us

36

  1. A) NO CHANGE
  2. B) Nevertheless, we know now
  3. C) We now know
  4. D) Now we have known

37

  1. A) NO CHANGE
  2. B) never hoped
  3. C) would never hope
  4. D) could never hope

38

The author is considering deleting this sentence in order to make the paragraph more concise. Should the author do this?

  1. A) Yes, because it asks a question about astronomers that has already been answered implicitly.
  2. B) Yes, because the fact that some stars are millions of light years away has already been established.
  3. C) No, because the question provides a logical transition to the discussion of inferential methods that follows.
  4. D) No, because it provides an important rhetorical question about astronomy that emphasizes the author”s main thesis.

39

  1. A) NO CHANGE
  2. B) we can”t sample to make observations easily of stars
  3. C) stars are not easily sampled for observation
  4. D) stars are not easy to sample in observation

40

  1. A) more, the photons being then
  2. B) more when the photons are
  3. C) more where the photons are
  4. D) more until the photons are

41

Which choice best matches the style of the sentence and adds a relevant new piece of information?

  1. A) NO CHANGE
  2. B) which is an incredible discovery for the astronomers.
  3. C) which is often very different in size from the nearby stars.
  4. D) even when that gas is spinning very quickly.

42

  1. A) NO CHANGE
  2. B) stars began their
  3. C) stars began it”s
  4. D) stars began its

43

  1. A) NO CHANGE
  2. B) exposure
  3. C) extent
  4. D) expedience

44

  1. A) NO CHANGE
  2. B) spectrum that reveals by characteristics
  3. C) spectrum, the characteristics of which reveal
  4. D) spectrum revealing, by its characteristics

STOP

If you finish before time is called, you may check your work on this section only. Do not turn to any other section of the test.

Math Test – No Calculator

25 MINUTES, 20 QUESTIONS

Turn to Section 3 of your answer sheet to answer the questions in this section .

DIRECTIONS

For questions 1–15, solve each problem, choose the best answer from the choices provided, and fill in the corresponding circle on your answer sheet. For questions 16–20, solve the problem and enter your answer in the grid on the answer sheet. Please refer to the directions before question 16 on how to enter you answers in the grid. You may use any available space in your test booklet for scratch work.

NOTES

  1. The use of a calculator is NOT permitted.
  2. All variables and expressions used represent real numbers unless otherwise indicated.
  3. Figures provided in this test are drawn to scale unless otherwise indicated.
  4. All figures lie in a plane unless otherwise indicated.
  5. Unless otherwise indicated, the domain of a given functionfis the set of all real numbers for which f (x ) is a real number.

REFERENCE

The number of degrees of arc in a circle is 360.

The number of radians of arc in a circle is 2π.

The sum of the measures in degrees of the angles of a triangle is 180.

1

If 2x − 3y = 9 and y = 3, then what is the value of x ?

  1. A) 0
  2. B) 3
  3. C) 6
  4. D) 9

2

xy = −4

x − 2y = −6

Which of the following ordered pairs (x , y ) satisfies the system of equations above?

  1. A) (22, 2)
  2. B) (22, 4)
  3. C) (4, 8)
  4. D) (4, 28)

3

An information technology company estimates the cost of a project, in dollars, using the expression 240 1 3nt , where n is the number of computer servers working on the project and t is the total time, in hours, the project will take using n servers. Which of the following is the best interpretation of the number 3 in the expression?

  1. A) Each server costs the company $3 per hour to run.
  2. B) A minimum of 3 servers will work on the project.
  3. C) The price of the project increases by $3 every hour.
  4. D) Each server can work 3 hours per day.

4

If , what is the value of x ?

  1. A)
  2. C)
  3. D)
  4. E)

5

a 4 − 6a 2 + 10

Which of the following is equivalent to the expression shown above?

  1. A) (a2− 2)(a 2 − 5)
  2. B) (a2− 1)(a 2 − 10)
  3. C) (a2+ 3)2 + 1
  4. D) (a2− 3)2 + 1

6

In triangle ABC , angle C has a measure of 90°. If sin A 5 0.6, what is the value of cos B ?

  1. A) 0.3
  2. B) 0.4
  3. C) 0.6
  4. D) 0.8

7

If m , 0 and x 5 6 in the equation above, what is the value of m ?

  1. A) −13
  2. B) −10
  3. C) −7
  4. D) −3

8

If and , what is the value of ba ?

  1. A) −2
  2. B) 2
  3. C) 4
  4. D) 8

9

The table above shows ordered pairs that satisfy the function f . Which of the following could define f ?

  1. A) f(b) = 3b 2 − 2
  2. B) f(b) = 3b 2 − 4
  3. C) f(b) = 2b 2 − 7
  4. D) f(b) = 2b 2

10

The equation y = kx − 1, where k is a constant, describes a line in the xy -plane. If the graph of this line contains the point (a , b ), where a and b are nonzero, what is the value of k in terms of a and b ?

  1. A)
  2. B)
  3. C)
  4. D)

11

The equation is true for all values of , where b is a constant. What is the value of b ?

  1. A) −12
  2. B) −6
  3. C) 4
  4. D) 6

12

If h and k are functions such that h(x ) = x + 3 and h(g (2)) = 9, which of the following could describe g(x )?

  1. A) x2+ 2
  2. B) x2+ 3
  3. C) x2+ 4
  4. D) x2+ 5

13

y = a (x + 3)(x − 1)

In the quadratic function above, a is a nonzero constant. The graph of the equation in the xy –plane is a parabola with vertex (m , n ). Which of the following is equal to n ?

  1. A) 0
  2. B) −a
  3. C) −2a
  4. D) −4a

14

x 2 − 2ax + b = 0

In the equation above, a and b are constants. If this equation is solved for x , there are two solutions. What is the sum of these two solutions?

  1. A) 2a
  2. B) −2a
  3. C) b
  4. D) −b

15

Which of the following can represent the graph in the xy -plane of y = a (xb )(x + c )2 , where a , b , and c are all positive constants?

  1. A)
  2. B)
  3. C)
  4. D)

DIRECTIONS

For questions 16–20, solve the problem and enter your answer in the grid, as described below, on the answer sheet.

  1. Although not required, it is suggested that you write your answer in the boxes at the top of the columns to help you fill in the circles accurately. You will receive credit only if the circles are filled in correctly.
  2. Mark no more than one circle in any column.
  3. No question has a negative answer.
  4. Some problems may have more than one correct answer. In such cases, grid only one answer.
  5. Mixed numberssuch as must be gridded as 3.5 or .

(If is entered into the grid as , it will be interpreted as , not .)

  1. Decimal answers:If you obtain a decimal answer with more digits than the grid can accommodate, it may be either rounded or truncated, but it must fill the entire grid.

16

At a restaurant, each large order of fries has 350 more calories than one large soda. If 2 large orders of fries and 3 large sodas have a total of 1,500 calories, how many calories does one large order of fries have?

17

If a and , what is the value of b ?

18

If x > 0, what is the solution to the equation above?

19

Note: Figure not drawn to scale.

x ≥ 0

y ≥ 0

3x + yk

In the figure above, the shaded region represents the solution set for the system of inequalities shown. If the area of this shaded region is 24 square units, what is the value of k ?

20

In the figure above, a circle is inscribed in a square that is inscribed in a larger circle. If the area of the larger circle is 16.5 square units, what is the area of the smaller circle?

STOP

If you finish before time is called, you may check your work on this section only. Do not turn to any other section of the test.

Math Test – Calculator

55 MINUTES, 38 QUESTIONS

Turn to Section 4 of your answer sheet to answer the questions in this section.

DIRECTIONS

For questions 1–30, solve each problem, choose the best answer from the choices provided, and fill in the corresponding circle on your answer sheet. For questions 31–38, solve the problem and enter your answer in the grid on the answer sheet. Please refer to the directions before question 31 on how to enter you answers in the grid. You may use any available space in your test booklet for scratch work.

NOTES

  1. The use of a calculator is permitted.
  2. All variables and expressions used represent real numbers unless otherwise indicated.
  3. Figures provided in this test are drawn to scale unless otherwise indicated.
  4. All figures lie in a plane unless otherwise indicated.
  5. Unless otherwise indicated, the domain of a given functionfis the set of all real numbers for which f (x ) is a real number.

REFERENCE

The number of degrees of arc in a circle is 360.

The number of radians of arc in a circle is 2π.

The sum of the measures in degrees of the angles of a triangle is 180.

1

A multiple-choice math test consists of 50 questions. Every student earns 2 points for each correct answer, –0.25 points for each incorrect answer, and 0 points for each question left unanswered. If a student answers 40 questions and gets 32 of them correct, how many points does the student earn?

  1. A) 61.5
  2. B) 62.0
  3. C) 62.5
  4. D) 64.0

2

If the average of 3, 5, and m is 10, what is the value of m ?

  1. A) 2
  2. B) 6
  3. C) 12
  4. D) 22

3

If 3b + 4 = −1, what is the value of 9b + 12?

  1. A) −8
  2. B) −4
  3. C) −3
  4. D) −2

4

The graph above shows Rebecca”s distance from her base camp as she hiked to a mountaintop, took a 1-hour break for lunch, and returned back to base camp. According to the graph, approximately how much longer was her hike to the mountaintop than her hike from the mountaintop back to base camp?

  1. A) 20 minutes
  2. B) 30 minutes
  3. C) 45 minutes
  4. D) 60 minutes

5

In the 2014 season, the Bombers baseball team had a win-to-loss ratio of 5:3, with no game ending in a tie. If the Bombers played 120 total games in 2014, how many games did they lose?

  1. A) 24
  2. B) 36
  3. C) 45
  4. D) 72

6

3x 3 − 2x 2 + 5

5x 2 + x − 10

Which of the following is the sum of the two polynomials shown above?

  1. A) 8x3− 2x − 5
  2. B) 3x3x 2 − 5
  3. C) 3x3+ 3x 2 + x − 5
  4. D) 8x5x 3 − 5

7

In the figure above, what is the value of a + b ?

  1. A) 212
  2. B) 238
  3. C) 296
  4. D) 328

8

If , where , what is the value of K ?

  1. A) 22
  2. B) −2
  3. C) 2 +i
  4. D) −2 +i

9

Based on the ordered pairs in the table above, which of the following could express the relationship between the variables x and y ?

  1. A) yvaries linearly with, but not directly as,x .
  2. B) yvaries directly asx .
  3. C) yvaries inversely asx .
  4. D) yvaries exponentially asx .

Questions 10–12 refer to the following information.

The scatterplot above charts the temperature (in degrees Kelvin) and luminosity (in Suns) for 50 stars, including our own sun, that fall under the category of “Main Sequence” stars.

10

The vertical axis indicates the luminosity of the stars in units called “Suns.” (A Sun unit equals the luminosity of our own sun.) According to the scatterplot, which of the following is the best estimate for the temperature of our sun?

  1. A) 2,600°K
  2. B) 5,800°K
  3. C) 10,100°K
  4. D) 12,400°K

11

According to the line of best fit shown on the scatterplot, a Main Sequence star with a temperature of 10,000°K is approximately how many times as luminous as a Main Sequence star with a temperature of 5,000°K?

  1. A) 9 times as luminous
  2. B) 90 times as luminous
  3. C) 900 times as luminous
  4. D) 900,000 times as luminous

12

What percent of the stars represented in the scatter-plot have a luminosity less than 0.0001 Sun?

  1. A) 0.2%
  2. B) 0.4%
  3. C) 2%
  4. D) 4%

13

Given the formula above, which of the following expresses k in terms of a and b ?

  1. A)
  2. B)
  3. C)
  4. D)

14

Which of the following functions, when graphed in the xy -plane, will intersect the x -axis exactly 3 times?

  1. A) f(x) = (x 2 + 1)(x 2 + 1)
  2. B) f(x) = (x 2 − 1)(x 2 + 1)
  3. C) f(x) = x 2 (x 2 − 1)
  4. D) f(x) = x 2 (x 2 + 1)

15

For how many distinct integer values of n is (n 1 2)(n + 8) negative?

  1. A) Four
  2. B) Five
  3. C) Six
  4. D) Seven

16

Lauren”s car can travel d miles per gallon of gasoline. If she travels at a constant speed of s miles per hour, which of the following represents the number of hours she can travel on 6 gallons of gasoline?

  1. A)
  2. B)
  3. C)
  4. D)

17

If a and b are non-zero constants in the linear equation above, what is the slope of this line when it is graphed in the xy -plane?

  1. A)
  2. B)
  3. C)
  4. D)

Questions 18 and 19 refer to the following information.

P (t ) = 250(2.4) t

The formula above shows the relationship between the population, P , of a certain mushroom species on a one-acre plot of land as a function of t , the number of weeks that have passed since the mushrooms were first introduced on the plot.

18

What is the meaning of the number 250 in the formula above?

  1. A) The plot initially contained 250 mushrooms.
  2. B) The population of mushrooms increases by 250 mushrooms per week.
  3. C) The population of mushrooms increases by 250% each week.
  4. D) It will take 250 weeks for the population of mushrooms to double.

19

By what percent should we expect the mushroom population to increase between the start of week 6 and the start of week 7?

  1. A) 40%
  2. B) 140%
  3. C) 240%
  4. D) 480%

20

If , which of the following expressions gives both possible values of x , in terms of p ?

  1. A)
  2. B)
  3. C)
  4. D)

21

If the variable a varies inversely as b , which of the following statements must be true?

  1. A) a+b is a constant.
  2. B) ab is a constant.
  3. C) is a constant.
  4. D) abis a constant.

22

A researcher is studying the effectiveness of a method for testing the presence of an antibody in a patient”s bloodstream. The table above shows the results of 1,000 patient trials. According to these data, what is the probability that a patient who has the antibody will nevertheless have a negative test result?

  1. A) 0.01
  2. B) 0.04
  3. C) 0.05
  4. D) 0.20

23

Class A: 68, 79, 88, 91, 97, 98, 99

Class B: 85, 85, 85, 88, 88, 90, 90

The lists above indicate the tests scores, in increasing order, for two of Mr. Pearlman”s classes, each of which has 6 students. Which of the following correctly compares the standard deviation of the scores for each class?

  1. A) The standard deviation of the scores in Class A is smaller.
  2. B) The standard deviation of the scores in Class B is smaller.
  3. C) The standard deviations of the scores in Class A and Class B are equal.
  4. D) The relationship cannot be determined from the information given.

24

Mrs. Black has a bag of candy bars to hand out to the students in her class before they take their AP calculus BC exam. If she gives each student 3 candy bars, she will have 6 left over. In order to give each student 5 candy bars, she will need 50 more candy bars. How many students are in Mrs. Black”s class?

  1. A) 18
  2. B) 27
  3. C) 28
  4. D) 44

25

The sum of three numbers is 240. If the greatest of these numbers is 50% more than the sum of the other two, what is the value of the greatest of these numbers?

  1. A) 96
  2. B) 120
  3. C) 140
  4. D) 144

26

In the xy -plane, points A (2, 5) and B (–12, k ) lie on a line that has a slope of What is the value of k ?

  1. A) 13
  2. B) 16.5
  3. C) 18
  4. D) 29.5

27

The original price of an outboard motor was marked down by 30% for a week-long sale. Since the motor was not sold in the first week, it was marked down an additional p percent. If the total markdown from the original price was then 58%, what is the value of p ?

  1. A) 12
  2. B) 28
  3. C) 40
  4. D) 42

28

In the figure above, if cos A = 0.8, what is the slope of line l ?

  1. A) 0.60
  2. B) 0.75
  3. C) 0.90
  4. D) 1.10

Questions 29 and 30 refer to the following information.

The table above shows the results of a survey of 520 adults who were asked whether they approved of a recent state budget proposal.

29

If n of the females had voted yes instead of no, the ratio of yes votes to no votes would have been exactly the same for the females as for the males. What is the value of n ?

  1. A) 62
  2. B) 66
  3. C) 68
  4. D) 70

30

If this survey is representative of the entire voting population of a state in which 32,760 people are expected to vote on this budget referendum, how many males are expected to vote yes?

  1. A) 9,450
  2. B) 15,120
  3. C) 19,270
  4. D) 20,475

Student-Produced Response Questions

DIRECTIONS

For questions 31–38, solve the problem and enter your answer in the grid, as described below, on the answer sheet.

  1. Although not required, it is suggested that you write your answer in the boxes at the top of the columns to help you fill in the circles accurately. You will receive credit only if the circles are filled in correctly.
  2. Mark no more than one circle in any column.
  3. No question has a negative answer.
  4. Some problems may have more than one correct answer. In such cases, grid only one answer.
  5. Mixed numberssuch as must be gridded as 3.5 or .

(If is entered into the grid as , it will be interpreted as , not .)

31

If a shipment of fruit contains 6 tons of bananas, 4 tons of grapes, 2 tons of apples, and 3 tons of oranges, what fraction of the shipment, by weight, is oranges?

32

A state environmental study determines that the coastal regions of the state lose 24.5 acres of wet-lands per month. At this rate, how many months will it take these coastal regions to lose a total of 343 acres?

33

dn = 13n + 200

The formula above represents the number of donuts, dn , that a bakery sold on the n th day of a festival. If the festival lasted 3 days, what was the total number of donuts that the bakery sold during the festival?

34

Connor and Joachim collaborated to write a computer program that consisted of 3,500 lines of code. If Joachim wrote 600 more lines of code than Connor did, how many lines of code did Connor write?

35

V (t ) = 1000(1 + k ) m

An analyst wants to use the formula above to estimate the value, in dollars, of a $1,000 initial investment in a mutual fund after m quarters have passed. If a $1,000 initial investment in this fund is worth $1,102.50 after 2 quarters, what number should the analyst choose for k ?

36

The figure above shows a hemispherical bowl made of glass. The bowl is 9 centimeters high and the glass is 3 centimeter thick. A second bowl is to be constructed to scale with the original bowl, but with one-half the height and diameter. The smaller bowl can hold a maximum of k π cubic centimeters of water. What is the value of k ? (The volume of a sphere with radius r is given by the formula .)

4

Questions 37 and 38 refer to the following information.

The formula above indicates the initial investment, I , that must be made in an account with an annual interest rate of r to ensure a future value of FV after a period of n years.

37

To the nearest dollar, what initial investment should be made in an account that earns 20% annually (r = 0.20) in order to ensure a future value of $432 in two years? (Ignore the $ sign when gridding your answer. That is, enter $125 as 125.)

38

What value of r , to the nearest thousandth, would ensure that the value of an investment would increase by 69% in 2 years?

STOP

If you finish before time is called, you may check your work on this section only. Do not turn to any other section of the test.

Essay

50 MINUTES, 1 QUESTION

DIRECTIONS

The essay gives you the opportunity to show how effectively you can read and comprehend a passage and write an essay analyzing the passage. In your essay, you should demonstrate that you have read the passage carefully, present a clear and logical analysis, and use language precisely.

Your essay must be written on the lines provided in your answer booklet; except for the Planning Page of the answer booklet, you will receive no other paper on which to write. You will have enough space if you write on every line, avoid wide margins, and keep your handwriting to a reasonable size. Remember that people who are not familiar with your handwriting will read what you write. Try to write or print so that what you are writing is legible to those readers.

You have 50 minutes to read the passage and write an essay in response to the prompt provided inside this booklet.

As you read the passage below, consider how Eric Schwitzgebel uses

  • evidence, such as facts or examples, to support claims
  • reasoning to develop ideas and connect claims and evidence
  • stylistic or persuasive elements, such as word choice or appeals to emotion, to add power to the ideas expressed

Adapted from Eric Schwitzgebel “We Have Greater Moral Obligations to Robots Than to Humans.” ©2016 Aeon Media (Aeon.co). Originally published in Aeon , November 12, 2015.

1 Down goes HotBot 4b into the volcano. The year is 2050 or 2150, and artificial intelligence has advanced sufficiently that such robots can be built with human-grade intelligence, creativity and desires. HotBot will now perish on this scientific mission. Does it have rights? In commanding it to go down, have we done something morally wrong?

2 The moral status of robots is a frequent theme in science fiction, back at least to Isaac Asimov”s robot stories, and the consensus is clear: if someday we manage to create robots that have mental lives similar to ours, with human-like plans, desires and a sense of self, including the capacity for joy and suffering, then those robots deserve moral consideration similar to that accorded to natural human beings. Philosophers and researchers on artificial intelligence who have written about this issue generally agree.

3 I want to challenge this consensus, but not in the way you might predict. I think that, if we someday create robots with human-like cognitive and emotional capacities, we owe them more moral consideration than we would normally owe to otherwise similar human beings.

4 Here”s why: we will have been their creators and designers. We are thus directly responsible both for their existence and for their happy or unhappy state. If a robot needlessly suffers or fails to reach its developmental potential, it will be in substantial part because of our failure—a failure in our creation, design or nurturance of it. Our moral relation to robots will more closely resemble the relation that parents have to their children, or that gods have to the beings they create, than the relationship between human strangers.

5 In a way, this is no more than equality. If I create a situation that puts other people at risk—for example, if I destroy their crops to build an airfield—then I have a moral obligation to compensate them, greater than my obligation to people with whom I have no causal connection. If we create genuinely conscious robots, we are deeply causally connected to them, and so substantially responsible for their welfare. That is the root of our special obligation.

6 Frankenstein”s monster says to his creator, Victor Frankenstein:

7 I am thy creature, and I will be even mild and docile to my natural lord and king, if thou wilt also perform thy part, the which thou owest me. Oh, Frankenstein, be not equitable to every other, and trample upon me alone, to whom thy justice, and even thy clemency and affection, is most due. Remember that I am thy creature: I ought to be thy Adam … .

8 We must either only create robots sufficiently simple that we know them not to merit moral consideration—as with all existing robots today—or we ought to bring them into existence only carefully and solicitously.

9 Alongside this duty to be solicitous comes another, of knowledge—a duty to know which of our creations are genuinely conscious. Which of them have real streams of subjective experience, and are capable of joy and suffering, or of cognitive achievements such as creativity and a sense of self? Without such knowledge, we won”t know what obligations we have to our creations.

10 Yet how can we acquire the relevant knowledge? How does one distinguish, for instance, between a genuine stream of emotional experience and simulated emotions in an artificial mind? Merely programming a superficial simulation of emotion isn”t enough. If I put a standard computer processor manufactured in 2015 into a toy dinosaur and program it to say “Ow!” when I press its off switch, I haven”t created a robot capable of suffering. But exactly what kind of processing and complexity is necessary to give rise to genuine human-like consciousness? On some views—John Searle”s, for example—consciousness might not be possible in any programmed entity; it might require a structure biologically similar to the human brain. Other views are much more liberal about the conditions sufficient for robot consciousness. The scientific study of consciousness is still in its infancy. The issue remains wide open.

11 If we continue to develop sophisticated forms of artificial intelligence, we have a moral obligation to improve our understanding of the conditions under which artificial consciousness might genuinely emerge. Otherwise we risk moral catastrophe—either the catastrophe of sacrificing our interests for beings that don”t deserve moral consideration because they experience happiness and suffering only falsely, or the catastrophe of failing to recognize robot suffering, and so unintentionally committing atrocities tantamount to slavery and murder against beings to whom we have an almost parental obligation of care.

12 We have, then, a direct moral obligation to treat our creations with an acknowledgement of our special responsibility for their joy, suffering, thoughtfulness and creative potential. But we also have an epistemic obligation to learn enough about the material and functional bases of joy, suffering, thoughtfulness and creativity to know when and whether our potential future creations deserve our moral concern.

Write an essay in which you explain how Eric Schwitzgebel builds an argument to persuade his audience that we have strong moral obligations toward the intelligent machines we create. In your essay, analyze how he uses one or more of the features listed in the box above (or features of your own choice) to strengthen the logic and persuasiveness of his argument. Be sure that your analysis focuses on the most relevant features of the passage.

Your essay should NOT explain whether you agree with Schwitzgebel”s claims, but rather explain how he builds an argument to persuade his audience.

SAT PRACTICE TEST 2 ANSWER KEY

Section 1: Reading

  1. D
  2. A
  3. D
  4. C
  5. C
  6. B
  7. C
  8. A
  9. C
  10. A
  11. B
  12. D
  13. D
  14. B
  15. A
  16. A
  17. C
  18. C
  19. C
  20. B
  21. B
  22. D
  23. C
  24. A
  25. A
  26. A
  27. B
  28. B
  29. C
  30. C
  31. A
  32. A
  33. C
  34. B
  35. A
  36. D
  37. B
  38. D
  39. D
  40. B
  41. D
  42. B
  43. D
  44. C
  45. B
  46. A
  47. C
  48. D
  49. A
  50. C
  51. A
  52. C

Total Reading Points (section 1)

Section 2: Writing and Language

  1. C
  2. C
  3. D
  4. D
  5. B
  6. C
  7. A
  8. D
  9. D
  10. C
  11. A
  12. D
  13. D
  14. A
  15. B
  16. A
  17. C
  18. C
  19. A
  20. B
  21. B
  22. C
  23. D
  24. A
  25. B
  26. A
  27. A
  28. D
  29. D
  30. D
  31. B
  32. B
  33. C
  34. D
  35. B
  36. A
  37. D
  38. C
  39. C
  40. D
  41. A
  42. D
  43. C
  44. C

Total Writing and Language Points (section 2)

Section 3: Math (No Calculator)

  1. D
  2. A
  3. A
  4. B
  5. D
  6. C
  7. C
  8. B
  9. C
  10. A
  11. B
  12. A
  13. D
  14. A
  15. D

-------

  1. 510
  2. 64
  3. 6
  4. 12
  5. 8.25

Total math Points (section 3)

Section 4: Math (Calculator)

  1. B
  2. D
  3. C
  4. B
  5. C
  6. C
  7. A
  8. A
  9. C
  10. B
  11. C
  12. D
  13. A
  14. C
  15. B
  16. A
  17. A
  18. A
  19. B
  20. C
  21. D
  22. B
  23. B
  24. C
  25. D
  26. A
  27. C
  28. B
  29. C
  30. A

-----------------

31 .2 or 1/5

  1. 14
  2. 678
  3. 1450
  4. .05 or 1/20
  5. 18
  6. 300
  7. .300

Total math Points (section 4)

SCORE CONVERSION TABLE

Scoring Your Test

  1. Use the answer key to mark your responses on each section.
  2. Total the number of correct responses for each section:
  3. Add the raw scores for sections 3 and 4. This is yourMath Raw Score :
  4. Use theTable 1 to calculate your Scaled Test and Section Scores (10–40) .
  5. Add theReading Test Scaled Score and the Writing and Language Test Scaled Score and multiply this sum by 10 to get your Reading and Writing Test Section Score (20–80) .

Table 1: Scaled Section and Test Scores (10–40)

SAT PRACTICE TEST 3 DETAILED ANSWER KEY

Section 1: Reading

  1. D

Purpose

The first sentence indicates what proponents of recycling assume (line 1), and the second sentence indicates what opponents [of recycling] scrutinize (lines 4–5). These sentences clearly characterize opposing viewpoints .

  1. A

Interpretation

In saying Environmentalists are not all ignorant anarchists, and opponents of recycling are not all rapacious blowhards (lines 15–17), the author is establishing a counterpoint to the demonization (line 13) that plagues debates about recycling. Therefore, the author is indicating that the debaters tend to mischaracterize their opponents .

  1. D

Meaning in Context

The full life cycles of various materials (lines 22–23) refers to how these materials are acquired, how they are used, and what happens to them after they have been used for industrial purposes. That is, the processes that affect the substances used in manufacturing .

  1. C

Word in Context

The phrase inserting some natural resources into a responsible “industrial cycle” (lines 49–51) refers to the process of using materials in industry rather than conserving them. This is a process of introducing those materials into an industrial process.

  1. C

Cross-Textual Inference

Passage 2 discusses the problem of quantifying the benefits of natural ecosystems . It mentions several such benefits, such as biodiversity, the filtration of groundwater, the maintenance of the oxygen and nitrogen cycles, and climate stability (lines 68–70). Therefore, water filtration (line 29) is clearly among these benefits.

  1. B

Inference

The author of Passage 1 asks several questions that he regards to be essential to a thorough analysis of environmental policy, such as Are any materials being imported from countries with irresponsible labor or environmental practices (lines 36–38)? This indicates that the author disapproves of irresponsible labor practices such as slave or child labor. Although it may seem reasonable to think that the author of Passage 1 would support tax incentives for recycling, limits on acid rain emissions, and public investment in sustainable domestic energy sources, there is no textual evidence in Passage 1 to support any contention that he, in fact, endorses such policies. In fact, the focus on Passage 1 is on the character of the debate about environmental policy, rather than on advocating any particular position.

  1. C

Textual Evidence

As the explanation to the previous question indicates, the best evidence is found in lines 36–38.

  1. A

Graphical Analysis

This pie graph shows the destination of U.S. solid municipal waste in 2012. It shows that 34.5% of this waste was recycled or composted , and therefore, it shows the scale of the recycling programs discussed in Passage 1.

  1. C

Cross-Textual Inference

The clear exchange mentioned in line 73 refers to the exchange with bought-and-sold commodities (lines 70–71), that is, financial expenditures. The only choice from Passage 1 that indicates a financial expenditure is the maintenance cost mentioned in lines 41–42.

  1. A

Content Analysis

Passage 1 focuses on the character of the debate between proponents of recycling (line 1) and their opponents (line 4). It refers to particular aspects of that debate, for example, the negative effects of the demonization (line 13) that each side uses to characterize its opponents. This demonization is a rhetorical strategy , that is, a persuasive technique.

  1. B

Purpose

Passage 2 compares the viewpoints of environmental-ists and industrialists in lines 102–106: Environmentalists argue that we cannot possibly put a price on the smell of heather and a cool breeze, while industrialists argue that the task is speculative, unreliable, and an impediment to economic progress . Previously in the passage, the author indicated that the act of put[ting] a price on the smell of heather and a cool breeze is known as quantifying “externalities” (line 75). The point in this sentence, therefore, is that careful examination of externalities is controversial , and the balance of the passage makes it clear that the author believes that this examination is necessary as well.

  1. D

Purpose

In this story, the narrator describes a person who had been in the area a long time (line 1), and whom she had seen in many places, but had never spoken to. He was a mystery to the town—a monastic stranger (line 10). She describes the many hypotheses that she and the other citizens of the town had about the man and his history. Finally, in the last sentences, she meets the man and hears him speak.

  1. D

Inference

This paragraph (lines 17–29) describes the author”s speculation that the stranger is from a war-torn land. She indicates what she “knew” about his situation: that he had run barefoot … through the rice paddies of Vietnam , that the earth [had] become a molten sea , and so on. Therefore, the phrase what we would all come to know most likely refers to the inhumanity of war .

  1. B

Textual Evidence

As the explanation to the previous question indicates, the best evidence is in lines 14–16.

  1. A

Meaning in Context

The question Had the earth become a molten sea, a hardened moonscape surface? (lines 23–25) is part of a series of questions about the mysterious man”s past, all of which imagine that he had seen unspeakable things (line 18) and had received the scar of war (line 23). This context makes clear that this reference is to a desolate aftermath of war.

  1. A

Word in Context

The phrase the character of a word in Japanese (lines 59– 60) refers to the written kanji symbols in Japanese, each of which represents a word. The point here is that the mysterious stranger resembled a kanji character.

  1. C

Inference

In this paragraph, the narrator says of the stranger that our paths crossed and converged daily (lines 30–31) and goes on to give several examples. Her point is that the stranger seemed to be everywhere, so these encounters are notable for their ubiquity (quality of appearing everywhere).

  1. C

Stylistic Analysis

The narrator describes the stranger as a crane (lines 19 and 52) and a black crow (line 62). These are avian (bird-related) metaphors . Although the narrator speculates that the stranger is from a war-torn land, she does not use any military comparisons to describe his physical characteristics. Likewise, the passage does not use any literary allusions or exaggerated juxtapositions to describe his physical appearance.

  1. C

Meaning in Context

This paragraph (lines 17–29) describes the author”s spec-ulation that the stranger is from a war-torn land. She indicates what she “knew” about his situation: that he had run barefoot … through the rice paddies of Vietnam , that the earth [had] become a molten sea , and so on. Therefore, the phrase what we would all come to know most likely refers to the inhumanity of war .

  1. B

Meaning in Context

When the narrator says that the stranger seemed to jour-ney momentarily out of that dark place (lines 74–75), she is explaining how his look was clear, not shrouded with darkness not veiled with otherness as I had come to expect(lines 72–74). The darkness here is the presumed trauma that she imagines he must have experienced. Therefore, the journey the narrator believes he has taken is from grim memory to current experience .

  1. B

Inference

The narrator makes it clear that she had come to expect (lines 73–74) the stranger to exude darkness (line 73) and otherness (line 73). Therefore, when the stranger greets her in a casual, upbeat way, she must have expressed surprised relief .

  1. D

Characterization

The author states that the exodus of women to the cities (line 1) has ameliorated (improved) the customs and diversified the streets (lines 5–6). These are transformative effects.

  1. C

Inference

The author states that New York women, and perhaps city women in general … are much more independent … and more original in their methods than women in smaller places (lines 7–11). In other words, rural women are less innovative .

  1. A

Textual Evidence

As the explanation to the previous question indicates, the best evidence is in lines 7–13.

  1. A

Inference

In lines 24–26, the author states that She accepts the situation [of her poverty] with the greatest good-humor and makes herself more acceptable to the old set by relating her discouragements . In other words, she considers her poverty a challenge to be embraced .

  1. A

Textual Evidence

As the explanation to the previous question indicates, the best evidence is in lines 24–28.

  1. B

Word in Context

The phrase done with a certain dash, élan, and sweeping air means done with a certain flair .

  1. B

Meaning in Context

The sentence she makes herself more acceptable to the old set by relating her discouragements, trials, and mistakes so comically that she is better company than before (lines 25–28) indicates that the old set is a group of people that the woman knew before she moved to the city; therefore this group is a set of established acquaintances .

  1. C

Inference

The passage states that the city woman who is occupied with daily work needs greater freedom of movement, more isolation, more personal comforts, and the exemption, moreover, from being agreeable at all times and places (lines 34–37). She also wants to extend hospitalities (lines 49–50) in her own home. Therefore, the author is saying that city women want to maintain their own homes primarily because they require living conditions conducive to their social independence . Choice (A) is incorrect because this passage is specifically about city women who have careers. Choice (B) is incorrect because, although the author indicates that one aspect of constructing homes is the male realization that the home is the proper stimulus to achievement (lines 47–48), she does not make any claims about homes helping women to maintain a social status comparable to that of men . Choice (D) is incorrect because the passage doesn”t indicate anything about the city woman doing work in the home, but rather retreating to her home for relaxation after work.

  1. C

Thesis

The thesis of the passage is that the social independence of women (specifically, their ability to move to cities and have careers) corresponds to their desire to own and maintain a home . Choice (A) is incorrect because the passage does not discuss unfair expectations of women. Choice (B) is incorrect because the passage does not discuss housekeeping as a traditional female duty , but rather a modern sign of female independence. Choice (D) is incorrect because, although the passage does mention that city women are creative and hardworking; this is not the main thesis of the passage as a whole.

  1. A

Meaning in Context

When the author states that a woman needs the exemption … from being agreeable at all times and places (lines 36–37), she means that city women with careers should not feel obliged to always pretend to be happy around others when they are not. Therefore, this exemption is a reprieve from a social obligation .

  1. A

Purpose

The second paragraph (lines 8–22) discusses how viruses have long been characterized as dangerous invaders (line 9). However, the rest of the passage discusses the great promise that viruses hold, through immuno-therapy and virotherapy, in curing diseases rather than causing them. Therefore, this paragraph serves to relate a point of view toward viruses that contrasts with that adopted by modern microbiologists .

  1. C

Inference

The second paragraph mentions HIV as an example of a viral disease that could possibly be treated by vaccination, which trains our immune system to produce antibodies that shield us from future infections . In lines 96–100, the passage discusses a study showing that lymphoblastic leukemia can be particularly responsive to immuno-therapy. Therefore, HIV and leukemia are both illnesses that can be treated by reinforcing the immune system . Although choice (A) may seem tempting, the passage makes it clear that vaccinations and immunotherapy work by different mechanisms than does virotherapy.

  1. B

Meaning in Context

The sentence Vaccinations are the major successes on this front means that vaccinations are the major successes in this campaign [against viraldiseases ].

  1. A

Integrated Inference

The diagram illustrates how engineered viruses used in virotherapy affect normal cells and cancer cells differently. It shows that relatively few of these viruses infect normal cells because of the poor viral receptivity of those cells. The passage indicates that the process by which viruses enter cells requires a “lock-and-key” mechanism (line 34) by which molecules on this virus shell match up with molecules on the surface of the cell.

  1. D

Inference/Graphical Analysis

The diagram illustrates T-cell attack on the bottom right, where the immune response (directed by T-cells as discussed in lines 80–84) attacks cells or viruses. It also illustrates lysis (lines 69–77) at the bottom center of the diagram, where it shows a cell breaking apart from overwhelming viral infection. It also illustrates selective infection by showing that more viruses infect the cancer cells than infect the normal cells. It does not, however, show apoptosis (lines 73–76), the process by which cells commit suicide.

  1. B

Graphical Analysis

The bottom right of the diagram illustrates how viral activity sometimes triggers anti-viral immune response , thereby destroying the viruses before they can kill the tumor cells. This illustrates one potential drawback that could compromise the effectiveness of virotherapy as a cancer treatment: the immune system may eliminate the viruses before they have a chance to destroy the cancer cell .

  1. D

Inference

In lines 85–90, the passage states that one pernicious aspect of cancer is its ability to “cloak” itself from the immune system and to evolve proteins that kill T-cells so that the immune system can”t attack the cancer effectively. In other words, the human immune system is thwarted by chemical defenses that cancer cells have developed .

  1. D

Textual Evidence

As the explanation to the previous question indicates, the best evidence is in lines 85–89.

  1. B

Meaning in Context

The attempts to replicate this success (lines 46–47) are attempts to use the rabies vaccine to kill a cancerous tumor. That is, it refers to a therapeutic repetition .

  1. D

Textual Evidence

Lines 77–80 mention a way that viruses can destroy cancer cells without directly infecting them: they can be programmed to selectively attack the blood vessels that supply nutrients to a tumor … and kill the cancer by starvation .

  1. B

Inference

The precision mentioned in line 59, as explained in the previous sentence, is the ability of viruses to selectively infect cancer cells and to leave healthy cells alone. The paragraph as a whole is discussing the powerful tools … for manipulating the very genetic code that produces some of those molecules [that control the interactions between cells and viruses] . Therefore, the paragraph is suggesting that genetic manipulation is the key to this kind of precision.

  1. D

Thesis

The author begins the passage by ruminating about the inner life of animals (line 2). He then goes on to show how animals do in fact have complex emotional lives (lines 7–8). From the third paragraph onward, the passage focuses on the fact that we hominids [who share common ancestors with these other animals] had to feel before we could think (lines 24–25), and how our emotions affect our thinking and enable us to build and manage large social groups. Therefore, the thesis of the passage as a whole is that we cannot understand human intelligence without first understanding animal intelligence .

  1. C

Purpose

The series of statements— fear keeps animals away from predators, lust draws them toward each other, panic motivates their social solidarity, and care glues their parent-offspring bonds (lines 18–21)—is intended to indicate more precisely how animals have complex emotional lives (lines 7–8) because emotion helps them navigate their outer life (lines 22–23) just as it does for humans. Therefore, the series of italicized words represents increasingly human sentiments .

  1. B

Meaning in Context

The statement that our cognitive brains work only when our emotions tilt our deliberations means that our emotions sway (influence) our rational thought processes.

  1. A

Purpose

Antonio Damasio”s work described in lines 32–34 shows how damage to the emotional systems of the brain can compromise decision-making. Since this discovery contradicts long-held beliefs about the distinction between reason and emotion, it highlights a surprising discovery about the function of emotions in human thinking .

  1. C

Inference

The phrase those early days (line 39) refers to the Pleistocene epoch (line 24) mentioned in the previous paragraph, when our hominid ancestors lived and were evolving modern human brains. Therefore, this phrase refers to the epoch in which our ancestors were developing the ability to think .

  1. D

Meaning in Context

The sentence in lines 56–60 indicates that the process of natural selection needs a stable environment over a long period of time in order to sculpt each module to fit our perennial environmental challenges . This evolutionary process by natural selection is a gradual process .

  1. A

Meaning in Context

The statement that the expansion of the brain corresponds with an increasingly adaptable mind means that as the human brain evolved to become larger, it also became more adaptable: the two processes coincided .

  1. C

Purpose

The sixth paragraph (lines 61–77) begins by stating that a vital premise for this modular theory is that the environment in which our ancestors evolved had to be extremely stable (lines 61–62). The paragraph then goes on to mention that recent discoveries (line 65) show that this environment was, in fact, anything but stable (lines 66–67). Therefore, this paragraph is refuting a scientific theory .

  1. A

Inference

In lines 67–69, the author states that In fact, it was precisely this climate chaos , and not the climate stability assumed by the modular theory, that created our multi-purpose, problem-solving minds . That is, the variable conditions of the Pleistocene epoch contradict an essential premise of the modular theory.

  1. C

Textual Evidence

As the explanation to the previous question indicates, the best evidence is in lines 67–69.

Section 2: Writing and Language

  1. C

Coordination

This sentence coordinates two contrasting claims, so the contrasting conjunction Although is most appropriate.

  1. C

Parallelism

This sentence contains a list, so the items in that list should have the same grammatical form. Since the first two items, the safety of our drinking water and the reliability of our roads , each has the form [definite article] [quality noun][prepositional phrase] , only choice (C), the usability of our smartphones , has the same form.

  1. D

Graphical Analysis

Choice (A) is incorrect because the graph does not show all engineering fields , and only indicates predictions for the next 10 years, not for the foreseeable future . Choice (B) is incorrect because the largest growth shown in the graph is 25%, which is not even close to doubling. Choice (C) is incorrect because the graph shows only growth, and no stagnation . Choice (D) is correct because some engineering fields , namely biomedical and petroleum, are expected to grow by more than 25% over the next 10 years.

  1. D

Parallelism

This sentence includes a list and, therefore, must follow the Law of Parallelism. Since the other two phrases in the list are gerund phrases, the underlined phrase must also be a gerund phrase. This eliminates choices (A) and (C). Choice (B) is incorrect because standard idiom requires the adverb merely rather than the adjective mere .

  1. B

Coordination/Voice

The underlined clause must coordinate grammatically and logically with the clause that follows. The subject of the clause that follows, many [engineering careers] , does not work unless the subject of the underlined clause also includes engineering careers . This eliminates choices (A) and (C). Choice (D) is incorrect because parallelism between the clauses involves the indicative mood, require , rather than the subjunctive mood, would require .

  1. C

Redundancy

The phrase make products out of [raw materials] indicates a transformative process, so the original phrasing is redundant. Choices (B) and (D) are also redundant. Choice (C) is concise and effective.

  1. A

Effective Expression

The original phrasing is best. Choice (B) is incorrect because the verb impact disagrees with the subject activity . Choice (C) is incorrect because it is not a noun phrase. Choice (D) is incorrect because it is not idiomatic.

  1. D

Parallelism

Parallelism requires the phrasing to solve… and to see .

  1. D

Coordination/Pronoun Consistency

The pronoun they is inconsistent with the phrase many of us . The colon is appropriate because the second clause explains the first.

  1. C

Logic/Cohesiveness

This sentence belongs immediately after sentence 2 because it indicates a specific way in which enthusiasm is often destroyed by schooling . It belongs before sentence 3 because it specifies the problem that needs to be solved.

  1. A

Transitions/Cohesiveness

The original phrasing provides the most logical and effective transition because it mentions the need to make engineering fun discussed in the previous paragraph, and connects it to the reason provided in the sentence that follows.

  1. D

Verb Tense

This sentence is referring to a general quality about American voters , so the verb should be in the simple present tense in order to indicate the habitual aspect.

  1. D

Clear Expression of Ideas/Parallelism

Parallelism requires that this be a list of quality nouns: looks, personality , and reputation .

  1. A

Verb Tense

The original phrasing is best. The participle takes the consequential aspect because it indicates a status affected by a previous action.

  1. B

Parallelism

This is a list of infinitive phrases: to identify … to study. and to determine .

  1. A

Coordination

The interrupting modifier must have the same punctuation before and after—in this case, dashes.

  1. C

Effective Expression/Diction

The sentence that follows says that we shouldn”t fall for such transparently dishonest tactics , thereby implying that it is a problem if these techniques are too effectual .

  1. C

Modifier Errors/Idiom

The original phrasing is incorrect because the clause we must learn is not logically modified by the prepositional phrase instead of us . The proper comparison requires a participial phrase to modify the main clause: Instead of listening … we must learn .

  1. A

Coordination

The original phrasing provides the most logical transition because this sentence describes a proposed action to achieve the goal described in the previous sentence.

  1. B

Development/Cohesiveness

This sentence belongs immediately after sentence 1 because it refers directly to the understanding of the issues mentioned in sentence 1.

  1. B

Diction

The context requires the possessive pronoun whose rather than the contraction who”s [who is] .

  1. C

Development/Cohesiveness

This sentence detracts from the paragraph”s focus on the interests that the candidates represent rather than their need for publicity.

  1. D

Clear Expression/Dangling Modifiers

The underlined clause must coordinate with the modifying phrase that starts the sentence. Since the source of both inspiration and relaxation is sports, not human cultures , choices (A) and (B) are incorrect. Since the sentence indicates a status due to a previous state of being, the present tense, consequential aspect is required, as in choice (D).

  1. A

Clear Expression/Diction

Since the topic sentence refers to athletes as our greatest icons , the topic of this paragraph is the renown [popularity] of athletes.

  1. B

Pronoun-Antecedent Agreement/Logic

This phrase must coordinate with the rest of the sentence, which uses the pronoun him to refer specifically to Spartacus. Only choice (B) provides the correct subject.

  1. A

Cohesiveness

The original phrasing is the only option that extends the idea of the sentence, which is that people spend a lot of time and effort worshipping athletes.

  1. A

Idiom

The proper idioms are regard as and consider to be . The only choice that is idiomatically correct is choice (A).

  1. D

Logical Coordination

The predicate that follows this word indicates a consequence of squander[ing] precious brain power on trivialities . Therefore, this adverb should indicate a consequence, as with choice (D).

  1. D

Diction/Clear Expression

The point the author is making here is that idol worship causes harm to our moral fabric . The best choice to indicate this effect is (D), degrades .

  1. D

Tone/Style

The passage has a moralizing tone and a formal style. Choice (D) best matches this tone and style.

  1. B

Commas/Parallelism

This sentence uses the parallel idiom either A or B . Choice (B) is the only one that uses parallel form as well as proper idiom.

  1. B

Coherence/Development

This is an appropriate addition here because it describes the impact that sports injuries can have on the athletes” lives and supports the main idea of the paragraph, which is that idol worship in sport doesn”t even seem to help the vast majority of the athletes themselves .

  1. C

Parallelism

Parallelism requires the phrasing usually for little pay, but often with as much skill . Notice that both phrases are prepositional phrases.

  1. D

Diction/Clear Expression

This word sufficed is illogical here because this sentence does not indicate that stars served any specific need for these cultures. Stars cannot perform as omens, because they are not people or mechanisms. Rather, they served as omens to ancient cultures.

  1. B

Idiom/Misplaced Modifiers

In the original phrasing the idiom beckon for is used illogically, and the modifier long is misplaced. Choice (B) is the only choice that avoids both of these problems.

  1. A

Coordination

The original phrasing best establishes the contrast between the two clauses in this sentence. Choice (B) indicates a contrast, but the wrong one: it implies a contrast between the previous sentence and this one, rather than between the two ideas within this sentence.

  1. D

Logic/Verb Mood

The point in this sentence is that these objects of astronomical study are very far away. They are so far away that we could never hope to send probes there. The other phrasings create illogical statements.

  1. C

Development/Logical Cohesiveness

This sentence should not be deleted, because this question provides a logical transition to the discussion of inferential methods that follows . The paragraph that follows answers this question very nicely.

  1. C

Dangling Modifiers/Voice/Logical Comparisons

The sentence clearly intends to compare stars to other objects of fascination . This requires that the subject of the underlined clause be stars . This eliminates choices (A) and (B). Choice (D) is incorrect because the phrase in observation is not idiomatic.

  1. D

Coordination

Choice (D) most effectively joins the ideas in a logical temporal sequence.

  1. A

Cohesiveness/Development

The original phrasing matches the expository and measured tone of the passage, and adds a specific detail supporting the idea that much astronomical information is gathered indirectly.

  1. D

Pronoun-Antecedent Agreement/Commas

The original phrasing is incorrect because the pronoun their disagrees in number with its antecedent light . Choice (B) is incorrect for the same reason. Choice (C) is incorrect because the context requires the possessive form itsrather than the contraction it”s [it is] .

  1. C

Diction/Clear Expression

The point of this sentence is that the amount of the red-shift in the light indicates how far the light has traveled. In other words, the extent of this redshifting tells astronomers how far away the star is (or was) .

  1. C

Idiom

This sentence discusses how the characteristics of the spectrum indicate the size and composition of the star. Choice (C) is the only one that uses the proper idiom for this prepositional phrase.

Section 3: Math (No Calculator)

  1. D

Algebra (solving equations) EASY

Original equation:

2x − 3y 5 = 9

Substitute y = 3:

2x − 3(3) = 9

Simplify:

2x − 9 = 9

Add 9:

2x = 18

Divide by 2:

x = 9

  1. A

Algebra (linear systems) EASY

Original system:

xy = − 4

Simplify:

x − 2y = −6

Subtract corresponding sides of equation:

(xy ) − (x − 2y ) = −4 − (−6)

Substitute y = 2 into first equation to find x: x − 2 = −4

Add 2:

x = −2

Therefore the ordered pair is (–2, 2).

  1. A

Algebra (interpreting formulas) EASY-MEDIUM

One way to analyze a formula is by “dimensional analysis.” This is simply a way to check that the units in the formula represent what they are supposed to represent. For instance, we are told that the expression 240 + 3nt represents the cost, in dollars , of the project. This means that both terms in this expression must be in dollars . Notice, also, that n is the number of servers and t is the number of hours . Let”s use a ? to represent the units of the number 3 in the expression 3nt . Translating the term into units, this gives us

(?)(servers)(hours) = $

Divide by (servers)(hours): _______________? =

In other words, it costs the company 3 dollars per hour per server.

  1. B

Algebra (linear equations) EASY

Multiply by 16 (the common denominator): 10x = –1

Divide by 10: x = –1/10

  1. D

Advanced Mathematics (multiplying polynomials) EASY-MEDIUM

The simplest way to approach this problem is to “expand” the choices to see which one yields an expression that is equivalent to the original expression. You must remember how to FOIL (Chapter 9 , Lesson 4) when multiplying two binomials:

(A) (a 2 − 2)(a 2 − 5) = a 4 − 5a 2 − 2a 2 + 10 = a 4 − 7a 2 + 10

(Close, but no.)

(B) (a 2 − 1)(a 2 − 10) = a 4 − 10a 2 − 1a 2 + 10 = a 4 − 11a 2 + 10

(Close, but no.)

(C) (a 2 + 3)2 + 1 = a 4 + 3a 2 + 3a 2 + 9 + 1 = a 4 + 6a 2 + 10

(Close, but no.)

(D) (a 2 – 3)2 + 1 = a 4 − 3a 2 − 3a 2 + 9 + 1 = a 4 − 6a 2 + 10

(Yes.)

  1. C

Advanced Mathematics (rational equations)

It always helps to draw a diagram for geometry and trigonometry problems. For this problem, we also have to remember the basic definitions of the trigonometric functions discussed in Chapter 10 , Lesson 9: SOH CAH TOA.

Since sin A = 0.6, this means a/c = 0.6. Notice that cos B is also a/c , so it must also equal 0.6. This is an example of the cofunction identity we discussed in Chapter 10 , Lesson 10.

  1. C

Advanced Mathematics (rational equations) EASY-MEDIUM

Original equation:

Substitute x = 6:

Add 6:

Square both sides:

m 2 − 13 = 36

Add 13:

m 2 = 49

Take the square root:

m = ±7

But since m < 0, m = −7.

  1. B

Advanced Mathematics (exponents) MEDIUM

Substitute

(Law #3 from Chapter 9 , Lesson 9):

Cross-multiply:

x 2 = 16

Take the square root:

x = ±4

Recall the other equation:

Law #6 from Chapter 9 , Lesson 9:

x b-a = 16

Substitute x = ±4:

(±4) b-a = 16

Notice that both 42 = 16 and

(−4)2 = 16, therefore:

ba = 2

  1. C

Advanced Mathematics (functions) MEDIUM

This question asks us to find the function that is satisfied by all four ordered pairs. Here, it is probably easiest to work by process of elimination, and cross off those choices that aren”t satisfied by at least one of the ordered pairs. Let”s consider the first ordered pair, (2, 1), and see which functions give an output of 1 for an input of 2:

(A) f (2) = 3(2)2 − 2 = 3(4) − 2 = 12 − 2 = 10

(No—eliminate.)

(B) f (2) = 3(2)2 − 4 = 3(4) − 4 = 12 − 4 = 8

(No—eliminate.)

(C) f (2) = 2(2)2 − 7 = 2(4) − 7 = 8 − 7 = 1

(Yes—don”t eliminate.)

(D) f (2) = 2(2)2 = 2(4) = 8

(No—eliminate.)

So, it turns out we only have to test one ordered pair to get the right answer.

  1. A

Algebra (linear equations) MEDIUM

Original equation:

y = kx − 1

Substitute x 5 a and y 5 b:

y = ka − 1

Add 1:

b + 1 = ka

Divide by a:

  1. B

Advanced Mathematics (rational equations) MEDIUM-HARD

The fact that the denominators are equal in both fractions suggests that multiplying both sides will simplify

Multiply by

bx − 3: 12x 2 + 6x − 23 = −2x (bx − 3) − 23

Distribute:

12x 2 + 6x − 23 = −2bx 2 + 6x − 23

Subtract 6x − 23:

12x 2 = −2bx 2

Divide by x 2 :

12 = −2b

  1. A

Advanced Mathematics (functions) MEDIUM-HARD

Given:

h(g (2)) = 9

Apply definition h(x) = x + 3:

g (2) + 3 = 9

Subtract 3:

g (2) = 6

Therefore, g must be a function that takes an input of 2 and gives an output of 6. The only choice that satisfies this condition is (A) g(x) = x 2 + 2, because g (2) = (2)2 + 2 = 4 + 2 = 6.

  1. D

Advanced Mathematics (analyzing quadratics) HARD

Original equation:

y = a(x + 3)(x − 1)

This equation represents a quadratic in x . Since it is in factored form, it shows us that the function has x -intercepts (also known as zeroes or roots) when x + 3 = 0 or x − 1 = 0, so its x -intercepts are at x = −3 and x = 1. Recall (from Chapter 9 , Lesson 6) that a parabola representing a quadratic function has an axis of symmetry at x = k , where k is midway between the x -intercepts. Therefore, k = (−3 + 1)/2 = −1. This axis of symmetry, x = −1, must pass through the vertex. We can get the y -coordinate of this vertex by just plugging x = −1 back into the function:

y = a (−1 + 3)(−1− 1)

Simplify:

y = a (2)(−2) = −4a

  1. A

Advanced Mathematics (solving quadratics) MEDIUM-HARD

Recall from Chapter 9 , Lesson 5, that the solutions to quadratic of the form x 2 + bx + c = 0, the sum of those solutions is −b (the opposite of whatever the x coefficient is), and the product of those solutions is c (whatever the constant term is). In the quadratic x 2 − 2ax + b = 0, the x coefficient is −2a . Since this must be the opposite of the sum of the solutions, the sum of the solutions is 2a .

Although using this theorem gives us a quick and easy solution, the theorem may seem a little abstract and mysterious to you. (You might want to review Lesson 5 in Chapter 9 to refresh yourself on the proof.) So, there is another way to attack this question: just choose values of a and b so that the quadratic is easy to factor. For instance, if we choose a = 1 and b = −3, we get:

x 2 − 2(1)x − 3 = 0

Simplify:

x 2 − 2x − 3 = 0

Factor:

(x − 3)(x + 1) = 0

Solve with the Zero Product Property:

x = 3 or −1

The sum of these two solutions is 3 + −1 = 2.

Now we plug a = 1 and b = −3 into the answer choices and we get (A) 2, (B) −2, (C) −3, (D) 3. Clearly, the only choice that gives the correct sum is (A).

  1. D

Advanced Mathematics (analyzing polynomial graphs) HARD

By the Zero Product Property (Chapter 9 , Lesson 5), the graph of y = a(xb)(x + c )2 has zeroes at x = b and a “double root” at x = −c (because this expression has two factors of (x + c )). Since b and c are both positive, this means that the graph must have one single positive root and a “double” negative root. That is, the graph passes through the x -axis at a positive value of x and “bounces” off of the x -axis at a negative value of x . Notice that this eliminates choices (B) and (C). We also know that a , the “leading coefficient” of the polynomial, is positive. If the leading coefficient of the polynomial is positive, the polynomial must eventually “shoot up” toward positive infinity; that is, it must go up as we move to the right. This rules out choice (A) and leaves only choice (D) as correct.

  1. 510

Algebra (rates) EASY

Let x represent the number of calories in a large order of fries, because that is what the question is asking us to find. Since we are told that this is 350 calories more than the calories in a large soda, the number of calories in a large soda is x − 350. If 2 large fries and 3 large sodas have a total of 1,500 calories,

x + 3(x − 350) = 1,500

Distribute:

2x + 3x − 1,050 = 1,500

Simplify:

5x − 1,050 = 1,500

Add 1,050:

5x = 2,550

Divide by 5:

x = 510

  1. 64

Advanced Mathematics (radical equations) MEDIUM-HARD

Given:

Multiply by 2:

Substitute :

Square both sides:

2b = 64(2)

Divide by 2:

b = 64

  1. 6

Advanced Mathematics (quadratic equations) HARD

Although this does not look like a quadratic equation, in fact it is.

Original equation:

Multiply by x :

x 2 + 36 = 12x

Subtract 12x :

x 2 − 12x + 36 = 0

Factor:

(x − 6)(x − 6) = 0

Solve using the Zero Product Property:

x − 6 = 0, so x = 6

  1. 12

Additional topics (coordinate geometry) MEDIUM-HARD

The system of inequalities indicates that the line containing the hypotenuse of the triangle is given by the equation 3x + y = k , which has an x -intercept of and a y -intercept of (0, k ). This means that the

triangle has a base of and a height of k .

The area of the triangle is 24:

Simplify:

Multiply by 6:

k 2 = 144

Take the square root:

k = 12

  1. 8.25

Additional Topics (triangles, circles, and squares) HARD

Most students will begin this problem by trying to find the length of the radius of the larger circle. This is a bit of a pain and, as it turns out, completely unnecessary. Instead, start by drawing in the 45°-45°-90° triangle as shown, and notice that one leg of this triangle is the radius of the smaller circle, and the hypotenuse is the radius of the larger circle. This is the key to the relationship between the circles.

If we label the smaller leg r and use either the Pythagorean Theorem or the Reference Information about 45°-45°-90° triangles given at the beginning of the test, we find that the hypotenuse is . Therefore, the area of the smaller circle is πr 2 and the area of the larger circle is . In other words, the larger circle has an area that is twice the area of the smaller circle. Therefore, if the larger circle has area 16.5, the smaller circle has an area of 16.5 ÷ 2 = 8.25.

Section 4: Math (calculator)

  1. B

Algebra (word problems) EASY

For getting 32 questions correct and 40 − 32 = 8 questions wrong, the student earned 32(2) − 0.25(8) = 64 − 2 = 62 points.

  1. D

Problem Solving and Data Analysis (central tendency) EASY

The average of the three numbers is 10:

Multiply by 3:

3 + 5 + m = 30

Simplify:

8 + m = 30

Subtract 8:

m = 22

  1. C

Algebra (linear equations) EASY

Original equation:

3b + 4 = −1

Multiply by 3:

9b + 12 = −3

  1. B

Data Analysis and Problem Solving (graphical analysis) EASY

The graph indicates that Rebecca started her hike at 10:00 a.m. and stopped for lunch at 11:45 a.m. (The horizontal segment indicates when she is not moving, so that represents her 1-hour lunch break.) This first part of her hike, therefore. took 1 hour 45 minutes, or 105 minutes. The return hike began at 12:45 and lasted until 2:00 p.m., for a time of 1 hour 15 minutes, or 75 minutes. The difference is 105 − 75 = 30 minutes.

  1. C

Algebra (ratios) EASY

The win-to-loss ratio of 5:3 is a “part-to-part” ratio, so we can represent each part as a fraction of the whole. We can do this by simply adding 3 + 5 = 8 (the “whole”) and dividing by that total. This gives us a ratio of , which means that the Bombers won of their games and lost of them. Since they played a total of 120 games, they won games and lost games.

  1. C

Advanced Mathematics (adding polynomials) MEDIUM

(3x 3 − 2x 2 + 5) + (5x 2 + x − 10)

Distribute coefficients to eliminate parentheses:

3x 3 − 2x 2 + 5 + 5x 2 + x − 10

Group like terms together:

3x 3 − 2x 2 + 5x 2 + x + 5 − 10

Combine like terms:

3x 3 + 3x 2 + x − 5

  1. A

Additional Topics (triangles) MEDIUM

It helps to mark the measures of the other two interior angles to the triangle as c ° and d °, because we know something about these angles.

Since angles in a triangle must have a sum of 180°:

c + d + 32 = 180

Subtract 32:

c + d = 148

Since angles in a linear pair have a sum of 180°:

a + c = 180

b + d = 180

Add previous two equations:

a + b + c + d = 360

Subtract c 1 d 5 148:

  1. A

Additional Topics (complex numbers) MEDIUM

Multiply by i :

K + i = i (1 − i )

Distribute:

K + i = i − 2i 2

Substitute i 2 = −1:

K + i = i + 2

Subtract i :

K = 2

  1. C

Advanced Mathematics (variation) MEDIUM

To answer this question, it helps to be familiar with the concepts we discussed in Chapter 8 , Lesson 4. The first thing to notice about the ordered pairs is that as the value of x increases, the value of y decreases. More specifically, notice that the product of the two values in each ordered pair is always the same: (2)(10) = 20, (4)(5) = 20, and (10)(2) = 20. A discussed in Chapter 8 , Lesson 4, this is the hallmark of an inverse variation . The equation for this particular relationship is y = 20/x .

  1. B

Problem Solving and Data Analysis (scatterplots) MEDIUM-HARD

Since, by definition, the luminosity of our sun is “1 sun,” we must look for the number 1 on the vertical (“luminosity”) axis. These numbers are given as powers of 10, so we have to remember that 100 5 1. If we trace the horizontal line representing 100 lumens, we can see that it intersects the line of best fit at roughly 6,000°K. The choice that is closest to this value is (B) 5,800°K.

  1. C

Advanced Mathematics (rational equations) MEDIUM-HARD

The line of best fit appears to cross the points (10,000°K, 102 suns) and (5,000°K, 10−1 suns), give or take a small error. This means that a Main Sequence star with a temperature of 10,000°K is about 102 ÷ 10−1 = 103 = 1,000 times as bright as a Main Sequence star with a temperature of 5,000°K. Choice (C) 900 is the only choice within an acceptable margin of error.

  1. D

Problem Solving and Data Analysis (scatterplots) MEDIUM-HARD

First, we need to recognize that 0.0001 = 10-4 . The scat-terplot shows precisely two points below the 10-4 line. Since there are 50 stars represented in the scatterplot (no need to count them—the description of the graph tells us!), these two stars represent 2/50 = 4/100 = 4% of the total.

  1. A

Algebra (linear equations) MEDIUM

Original equation:

Multiply by b + k :

a + k = −2(b + k )

Distribute:

a + k = −2b − 2k

Subtract k :

a = −2b − 3k

Add 2b :

a + 2b = −3k

Divide by −3:

  1. C

Advanced Mathematics (graphing polynomials) MEDIUM

First, notice that the answer choices include the factors x 2 − 1 and x 2 + 1. The first of these is a difference of squares, so it can be factored further: x 2 − 1 = (x + 1)(x − 1). However, x 2 + 1 is a sum of squares, which cannot be factored over the real numbers. This enables us to express each function in completely factored form:

(A) f (x ) = (x 2 + 1) (x 2 + 1) = (x 2 + 1) (x 2 + 1)

(B) f (x ) = (x 2 − 1) (x 2 + 1) =(x + 1)(x − 1)(x 2 + 1)

(C) f (x ) = x 2 (x 2 − 1) = x 2 (x + 1)(x − 1)

(D) f (x ) = x 2 (x 2 + 1) = x 2 (x 2 + 1) = x 2 (x 2 + 1)

Now we can find all of the x -intercepts by setting each factor to 0 and (if possible) solving for x . Notice that if we do this for the factored form of each function, we see that (A) has no x -intercepts, (B) has intercepts at x = −1 and x= 1, (C) has intercepts at x = 0, x = −1, and x = 1, and (D) has an intercept at x = 0. Therefore, the function in choice (C) is the only one that has three x -intercepts.

  1. B

Algebra (quantitative reasoning) MEDIUM-HARD

First, we should notice the fact that n + 8 must be greater than n + 2, no matter the value of n . Next, we should notice that, in order for the product of two numbers to be negative, one of those numbers must be positive and the other one negative. Obviously, the greater number is the positive one, and the lesser one is the negative one.

Therefore:

n + 2 < 0 and n + 8 > 0

Solve each inequality for n :

n < –2 and n > –8

Since n must have an integer value and must satisfy the inequalities above, it can take only the values –7, –6, –5, –4, and –3.

  1. A

Algebra (rates) MEDIUM

Although this problem can be solved by “plugging in” convenient numbers for the unknowns, it is actually much more straightforward to treat this as a conversion problem, as discussed in Chapter 7 , Lesson 4. The question gives us the “initial fact” that Lauren has 6 gallons of gas in her car, and we”d like to “convert” that fact into the number of hours she can travel. Using the rates given in the problem, the conversion should look like this:

Notice that all the units on the left-hand side cancel except for hours, which is the desired unit.

  1. A

Algebra (linear equations) MEDIUM-HARD

Perhaps the best way to find the slope of the line is to get the equation into “slope-intercept form” (Chapter 7 , Lesson 5).

Original equation:

Cross-multiply:

a (2y ) = b (2x + 1)

Simplify:

2ay = 2bx + b

Divide by 2a :

Simplify:

Therefore, the slope of the line is b/a .

  1. A

Advanced Mathematics (exponential functions) MEDIUM

Notice that substituting t = 0 into the function gives us P(0) = 250(2.4)0 = 250(1) = 250. Therefore, the number 250 in the equation means the population of mushrooms on the plot when t = 0.

  1. B

Advanced Mathematics (exponential functions) MEDIUM-HARD

The fastest way to answer this question is to notice that in exponential growth and decay functions in which the exponent is the time variable, t , the base of the exponential (in this case 2.4) must represent 1 + r , where r is the rate of change per time unit. This means that the rate of weekly increase (recall that t is measured in weeks) must be 2.4 – 1 = 1.4 = 140%.

Another way to solve the problem is to calculate the populations at the specified times and then calculate the percent change. At the beginning of the sixth week, 5 weeks have passed, and so the population is 250(2.4)5 = 19,906. At the beginning of the seventh week, the population is 250(2.4)6 = 47,776. To calculate the percent change, we find the difference and divide by the initial amount: (47,776 – 19,906)/19,906 = 1.40 = 140%.

  1. C

Algebra (rewriting expressions) MEDIUM-HARD

Original equation:

Multiply by 2p (the common denominator):

p (x 2 + 1) + 2x = 2p

Distribute:

px 2 + p + 2x = 2p

Subtract 2p and write in descending powers of x :

px 2 + 2xp = 0

Put into quadratic formula with a = p , b = 2, and c = –p :

  1. D

Problem Solving and Data Analysis (variation) MEDIUM-HARD

Recall from Chapter 8 , Lesson 4, that if a is inversely proportional to b , then , where k is some positive constant. If we multiply both sides of this equation by b , we get ab = k , which means that the product ab is a constant.

  1. B

Problem Solving and Data Analysis (quadratics) MEDIUM

The table indicates that a total 500 patients had the antibody and that 20 of these patients nevertheless had a negative test result. Therefore the probability of this result is 20/500 = 0.04.

  1. B

Problem Solving and Data Analysis (data spread) MEDIUM

As we discussed in Chapter 8 , Lesson 3, the standard deviation of a set of numbers tells us how “spread out” the data are from the average—the greater the standard deviation, the greater the spread from the average. It should be clear by inspection that the scores for Class B are more “clustered” than the scores for Class A. For instance, there is only a 5-point difference between the lowest and highest scores in Class B, but a 31-point difference between the lowest and highest scores in Class A. This means that the standard deviation of the scores in Class B is smaller.

  1. C

Algebra (word problems) MEDIUM-HARD

Begin by assuming that there are n students in Mrs. Black”s class. If she gives out 3 candy bars to each student and has 6 left over, she must have 3n + 6 candy bars. If she needs 50 more candy bars in order to give each student 5 candy bars, she must have 5n – 50 candy bars. Since these two expressions both express the total number of candy bars,

3n + 6 = 5n – 50

Add 50 and subtract 3n :

56 = 2n

Divide by 2:

28 = n

  1. D

Problem Solving (rates) MEDIUM

Let”s call the three numbers a , b , and c .

If their sum is 240,

a + b + c = 240

Let”s say the largest number is c . If this is 50% larger than the sum of the others,

c = 1.5(a + b )

Divide by 1.5:

Substitute into the first equation:

Multiply by 1.5:

c + 1.5c = 360

Simplify:

2.5c = 360

Divide by 2.5:

c = 144

  1. A

Algebra (graphs of linear equations) MEDIUM

Recall the slope formula from Chapter 7 , Lesson 5:

Since the slope equals :

Cross-multiply:

7k – 35 = 56

Add 35:

7k = 91

Divide by 7:

k = 13

  1. C

Algebra (word problems/percent change) HARD

The question makes it clear that we will get the same answer regardless of the original price of the motor. Since we are working with percentages, it is convenient to assume the original price of the motor is $100.

After a markdown of 30%, the price becomes $100 – 0.30($100) = $70. If it is marked down an additional p percent, its price becomes . Since this is equivalent to a one-time markdown of 58%, then

Divide by 70:

Subtract 1:

Multiply by –100:

p = 40

  1. B

Additional Topics (trigonometry/coordinate geometry) HARD

If this question gave you trouble, review Chapter 10 , particularly Lessons 3, 4, and 9. Since the question asks about the slope, we should draw in a right triangle to show the “rise” and “run” of the line. If cos A = 0.8, then the adjacent side of this triangle could be 8 and the hypotenuse could be 10 (because o/h = 8/10 = 0.8) as shown below.

We can find the third side of the triangle with the Pythagorean Theorem (82 + x 2 = 102 ), although it”s easier to simply notice that this is a triangle in the 3-4-5 family: 6-8-10. Since the slope of a line is the rise over the run, the slope is 6/8 = 0.75.

  1. C

Problem Solving/Data Analysis (tables/ratios) MEDIUM-HARD

For the males, the ratio of yes votes to no votes is 150:90 = 5:3. If n of the females had shifted their votes from no to yes, then 173 – n would have voted no and 107 + n would have voted yes. If this ratio is then equal to the ratio for males,

Cross-multiply:

321 + 3n = 865 – 5n

Subtract 321 and add 5n :

8n = 544

Divide by 8:

n = 68

  1. A

Problem Solving/Data Analysis (tables/proportions) HARD

The table shows that 150 males voted yes out of a sample population of 520. If this is a representative ratio, and if x represents the total number of males who vote yes out of the entire population, then

Cross-multiply:

(32,760)(15) = 52x

Divide by 52:

  1. .2

Problem Solving and Data Analysis (ratios) EASY

The total weight of the shipment is 6 + 4 + 2 + 3 = 15, and the total weight of oranges is 3, so the fraction of the shipment that is oranges is 3/15 5 .2.

  1. 14

Problem Solving and Data Analysis (proportions) EASY

If x is the number of months it takes until the regions lose 343 acres, then

Cross-multiply:

24.5x = 343

Divide by 24.5:

x = 14

  1. 678

Problem Solving and Data Analysis (data from formula) MEDIUM

The formula indicates how many donuts were sold on each day of the festival. One day 1, the bakery sold 13(1) + 200 = 213 donuts. On day 2, the bakery sold 13(2) + 200 = 226 donuts. On day 3, the bakery sold 13(3) + 200 = 239 donuts, for a total of 213 + 226 + 239 = 678 donuts.

  1. 1450

Problem Solving and Data Analysis (rates) MEDIUM

If Connor wrote x lines of code, then Joachim wrote x + 600 lines of code. Together they wrote (x ) + (x + 600) = 3,500 lines of code:

x + x + 600 = 3,500

Simplify and subtract 600:

2x = 2,900

Divide by 2:

x = 1,450

Therefore Connor wrote 1,450 lines of code.

  1. .05

Problem Solving (analyzing formulas) HARD

If the investment is worth $1,102.50 after 2 quarters,

1,000(1 + k )2 = 1,102.5

Divide by 1000:

(1 + k )2 = 1.1025

Take the square root:

1 + k = 1.05

Subtract 1:

k = .05

  1. 18

Additional Topics (volumes and similarity) HARD

The radius of the inside of the larger bowl is 9 – 3 = 6 centimeters, so the radius of the smaller bowl is 3 centimeters. The volume of the smaller bowl is therefore . Therefore, k = 18.

  1. 300

Problem Solving and Data Analysis (formula analysis) MEDIUM

This requires simply substituting into the formula:

  1. .300

Problem Solving and Data Analysis (formula analysis) HARD

If the value of the investment increases by 69% in 2 years, this means that FV = 1.691 when n = 2. Substituting into the formula gives

Divide by I :

Cross-multiply:

(1 + r )2 = 1.69

Take the square root:

1 + r = 1.3

Subtract 1:

r = 1.3 – 1 = 0.3